You are on page 1of 122

Q1.

A aceleração da gravidade g pode ser medida com razoável precisão usando-se um pêndulo
simples que consiste de um corpo de massa m preso a um fio de massa desprezı́vel e compri-
mento l .
(a) Encontre a expressão do perı́odo do pêndulo em função dos seus parâmetros.
(b) Um grupo de estudantes foi ao laboratório para obter uma medida precisa da aceleração
da gravidade no local. Para isso construiu um pêndulo simples com uma massa metálica presa
ao teto do laboratório por um fio fino. A massa metálica tem a forma de uma esfera de raio
r = 8,00 ± 0,05 cm e massa m = 10,0 ± 0,1 kg presa ao fio de forma que em repouso o centro de
massa da esfera fica a 4,00±0,02 m do teto. A massa do fio é 7,4±0,2 g. O perı́odo de oscilação
foi medido para diferentes deslocamentos iniciais laterais entre 5,0 ± 0,1 e 10,0 ± 0,1 cm. Os
estudantes determinaram que nesse intervalo de deslocamentos laterais o perı́odo não depende
da posição inicial, dentro da incerteza experimental e que o pêndulo realizou 10 oscilações
completas em 40,0 ± 0,5 s. Determine o valor de g encontrado, com a incerteza experimental.
Considere, se necessário, ⇡ 2 = 9,86960.

Q2. Dois corpos, cada um de massa M , estão ligados por uma corda uniforme inextensı́vel de
comprimento l. O corpo A está sobre uma mesa uniforme e o corpo B está pendurado na
lateral, a corda passando por uma polia de raio desprezı́vel sem atrito, como mostrado na
figura. Despreze o atrito entre A e a mesa.

A
M
x
θ

M B

(a) Encontre a aceleração comum dos corpos, se o ângulo ✓ é mantido constante e igual a
zero e a massa da corda é desprezı́vel.
(b) Considere agora o movimento mais geral em que o ângulo ✓ também pode variar. Suponha
que ✓ é sempre menor que ⇡/2 e que o corpo B nunca toca na mesa. Escreva a Lagrangiana
do sistema e as equações de movimento (não tente resolver as equações). Mostre que
recuperamos o resultado do item (a) se fizermos ✓ = 0.
(c) Suponha agora que ✓ é novamente mantido constante e igual a zero, mas a corda tem massa
não desprezı́vel m. Escreva a Lagrangiana do sistema e as equações de movimento. Não é
necessário resolver as equações.

1
Q3. Uma partı́cula com massa de repouso m e energia total relativı́stica igual a duas vezes sua
energia de repouso, colide frontalmente com uma partı́cula idêntica (mesma massa de repouso
m), inicialmente em repouso. Após a colisão forma-se uma única partı́cula possuindo massa
de repouso M (colisão totalmente inelástica). Nos itens abaixo, expresse suas respostas em
termos de c e m.

(a) Calcule o módulo v da velocidade da partı́cula incidente antes da colisão.


(b) Usando conservação de energia-momento,

(i) determine o módulo V da velocidade da partı́cula resultante em termos da velocidade v


da partı́cula incidente. Use o resultado do item (a) para obter o valor numérico de V /c.
(ii) determine a massa M da partı́cula resultante.

(c) Calcule a energia cinética da partı́cula resultante.

Q4. Considere a equação de Schrödinger uni-dimensional independente do tempo para energias


E no intervalo [0,U0 ] (U0 > 0) de uma partı́cula de massa m num potencial de poço quadrado
dado por 8
< U0 , se x < 0,
V (x) = 0, se 0 < x < L,
:
U0 , se x > L.

(a) O espectro de energias E é discreto ou contı́nuo? Por quê?


(b) Escreva a forma geral da função de onda nas 3 regiões do potencial.
(c) Escreva a equação cuja solução dá o espectro de energias. Não é necessário resolver essa
equação.
(d) O que acontece com o espectro de energias no limite em que L ! 0?

Q5. O motor de Stirling é uma máquina térmica cujo ciclo é composto por dois processos isotérmicos
e dois processos isocóricos (isovolumétricos). Considere que 1 mol de um gás monoatômico ideal
(CV = 3R/2) atravesse um ciclo de Stirling formado pelos seguintes processos consecutivos:
(1) compressão isotérmica até 1/3 do volume inicial V0 à temperatura T0 ; (2) aquecimento a
volume constante até o dobro da temperatura inicial T0 ; (3) expansão isotérmica até o volume
inicial V0 ; (4) resfriamento isovolumétrico até a temperatura inicial T0 .

(a) Esboce o ciclo acima num diagrama P x V (pressão por volume).


(b) Determine a variação da energia interna do gás nos processos 1 e 2 em termos de R e T0 .
(c) Determine o trabalho realizado pelo gás no processo 3 em termos de R e T0 .
(d) Determine o rendimento dessa máquina.

2
Q6. Dois aros circulares finos encontram-se no plano xy de um sistema de coordenadas, ambos com
centro na origem. Um aro tem raio b e uma densidade linear de carga elétrica < 0 e o outro
tem raio 2b e uma densidade linear de carga elétrica 2 > 0.

(a) Calcule o potencial eletrostático V (z) no ponto P = (0,0,z).


(b) Calcule o vetor campo elétrico E(z) no ponto P = (0,0,z).
(c) Escreva a equação da segunda lei de Newton para uma partı́cula de carga q > 0 e massa
m, restrita a se mover ao longo do eixo z e sujeita ao campo elétrico do item (b). Além da
força elétrica, nenhuma outra força atua sobre a partı́cula.
(d) Calcule a frequência de pequenas oscilações para a partı́cula do item (c) na vizinhança de
z = 0. Dica: linearize a força em torno de z = 0.

Q7. Uma onda eletromagnética plana monocromática que se propaga no vácuo com polarização
circular é descrita, em notação complexa, pelo campo elétrico Ẽ(r,t) = E0 ei(kz !t) (x̂ + iŷ),
onde ! = ck é a frequência angular,
p c é a velocidade da luz no vácuo, k é o número de onda,
E0 é uma amplitude real e i = 1.

(a) Encontre o campo elétrico real (fı́sico) E(r,t).


(b) Encontre o campo magnético real (fı́sico) B(r,t) usando as equações de Maxwell. Se prefe-
rir, utilize r ! ik.
(c) Calcule a densidade de momento linear da onda eletromagnética g = ✏0 E ⇥ B.
(d) Calcule a densidade de momento angular da onda eletromagnética ` = r ⇥ g. Dica: use
coordenadas cilı́ndricas r = ⇢ˆ
⇢ + z ẑ.

Q8. Um elétron com energia cinética Ecin = 22 eV colide com um átomo de hidrogênio que se
encontra inicialmente no estado fundamental. Apenas uma parte da energia do elétron inci-
dente é transferida para o átomo, que passa para um estado excitado com número quântico n.
Decorrido um intervalo de tempo t após a colisão, o átomo decai para o estado fundamental,
emitindo um fóton com energia igual a 10,2 eV.

(a) Usando a aproximação não relativı́stica, determine o comprimento de onda de de Broglie


do elétron incidente.
(b) Determine o número quântico n do estado excitado do átomo de hidrogênio.
(c) Calcule a incerteza na energia do fóton emitido sabendo que t = 10 8 s.
(d) Justifique a aproximação não relativı́stica utilizada no item (a).

1
Q9. Considere um sistema quântico cujo espaço de Hilbert é gerado por uma base ortonormal
de 3 estados |1i, |2i e |3i. 0
Um1estado genérico do sistema pode ser representado nessa base
x
através de um vetor coluna @y A, onde x, y e z são coeficientes complexos. A Hamiltoniana do
z
sistema, por sua vez, pode ser representada nessa mesma base através de uma matriz quadrada
complexa 0 1
E1 0 0
H = @ 0 E2 M23 A .
0 E3

(a) Qual é o valor do único elemento da matriz H que está faltando? Qual é o valor da parte
imaginária de E3 ?
(b) Um certo observável A atua sobre os estados da base da seguinte forma

A|1i = 2|1i,
A|2i = 2|2i,
A|3i = |3i.

Escreva a matriz que representa A nessa base. Este observável pode ser medido simultanea-
mente com a energia? Justifique.
(c) Quais são os autovalores de energia do sistema?
(d) Suponha que0E1 1 = 1, E2 = E3 = 3 e M23 = 1 e que o sistema seja preparado no instante
0
t = 0 no estado @1A. Encontre o estado para t > 0.
0

Q10. Considere um sistema formado por N ı́ons magnéticos localizados de spin 1 em contato com
um reservatório térmico à temperatura T . O sistema é descrito de forma simplificada pelo
Hamiltoniano
X N N
X
2
H=D i h i,
i=1 i=1

onde i é a projeção z (adimensional) do spin i, que pode assumir os valores 0, + 1 e 1, h > 0


é um campo magnético externo e D > 0 é um termo de anisotropia.

(a) Determine a função de partição do sistema.


(b) Determine a energia livre de Helmholtz por ı́on como função da temperatura.
(c) Determine a energia interna por ı́on como função da temperatura.
(d) Suponha agora que h = 0. Determine o calor especı́fico como função da temperatura.

2
Q1. Um corpo de massa m cai em linha reta a partir do repouso em um fluido. A aceleração da
gravidade ~g pode considerada constante. O corpo é sujeito também a uma força de resistência
proporcional à velocidade: F~r = km~v , onde k é uma constante. A força de empuxo do fluido
é desprezı́vel.
(a) Obtenha o módulo da velocidade do corpo como função do tempo.
(b) Qual é a velocidade terminal do corpo (módulo da velocidade no limite t ! 1)?
(c) Encontre z(t), a posição do corpo como função do tempo (considere z(0) = 0).
(d) Encontre z(v), a posição do corpo como função do módulo da velocidade.
t
Q2. O pêndulo duplo plano consiste de duas partı́culas de massas m1 e m2 e duas hastes rı́gidas
de massas desprezı́veis e comprimentos l1 e l2 , que oscilam, sob a ação da gravidade ~g , em um
mesmo plano vertical fixo, como representado na figura abaixo. Considerando ~g constante e
adotando como coordenadas generalizadas os ângulos ✓1 e ✓2 da figura, obtenha:

l1
θ1

m1
l2
θ2
m2

(a) A energia cinética do sistema.


(b) A energia potencial do sistema.
(c) A Lagrangiana do sistema.
(d) As equações de movimento relativas a ✓1 e ✓2 .

Q3. Considere a dinâmica quântica não relativı́stica de uma partı́cula de massa m num potencial
harmônico tridimensional isotrópico de frequência angular ! dado por
m! 2 2
V (x,y,z) = (x + y 2 + z 2 ).
2
(a) Escreva os auto-estados |n1 ,n2 ,n3 i da Hamiltoniana total Ĥ em termos dos auto-estados
de osciladores harmônicos unidimensionais |ni i (i = 1,2,3) e também as auto-energias de Ĥ.
(b) Uma das auto-energias do sistema é 72 ~!. Qual é a sua degenerescência?
(c) O observável Ĥ é medido quando o sistema se encontra no seguinte estado (considere os
auto-estados |n1 ,n2 ,n3 i normalizados)
1 1 1
| i = p |0,0,1i + |0,1,0i + |0,1,1i.
2 2 2
Que resultados podem ser obtidos e com que probabilidades?
(d) Suponha que a medida do item (c) resultou no valor 52 ~!. Considere t = 0 o instante
imediatamente posterior a essa medida. Determine o estado do sistema | (t)i para t > 0.

1
Q4. A densidade de energia u(T ) da radiação eletromagnética em equilı́brio térmico à temperatura
T pode ser expressa (a partir de argumentos termodinâmicos) como
Z 1 ⇣⌫ ⌘
u(T ) = ⌫ 3f d⌫, (1)
0 T
onde ⌫ é a frequência da radiação.

(a) Apenas usando a Eq. (1), encontre u(T ) a menos de um fator (independente de T ).
Qual é a dimensão deste fator?
(b) Em 1900, Planck descobriu que
⇣⌫ ⌘ 8⇡⌫ 2 h⌫
⌫ 3f = 3 (h⌫/k
,
T c e BT ) 1
onde h é a constante de Planck (que relaciona o quantum de energia e a frequência), c é a
velocidade da luz no vácuo e kB é a constante de Boltzmann.
8⇡⌫ 2
(i) Discuta, sem demonstar, o significado fı́sico do fator c3
d⌫.
(ii) Determine o comportamento da distribuição de energia no limite em que a energia do
fóton é muito menor do que a energia térmica kB T .
(iii) Qual é o significado do resultado do item (ii) no contexto da fı́sica clássica?

(c) As constantes c, ~ = h/(2⇡) e a constante gravitacional G podem ser usadas para definir
unidades absolutas de tempo (tP ), distância (lP ) e massa (mP ). Determine essas grandezas em
termos de produtos de potências de ~, c e G. Determine também a temperatura de Planck TP .
Estime a ordem de grandeza de tP , lP , mP e TP no sistema internacional de unidades.

Q5. Um estudante quer determinar o calor especı́fico cx de uma substância x desconhecida. Para
isso, dispõe de um calorı́metro, que é um dispositivo que idealmente não troca calor com o
ambiente. A capacidade térmica K do calorı́metro é conhecida. O calorı́metro encontra-se
inicialmente à temperatura ambiente Tamb . O procedimento experimental adotado consiste em
colocar uma massa conhecida de água mH2 O à temperatura ambiente Tamb no calorı́metro, adi-
cionar uma massa conhecida da substância x, mx , inicialmente apuma temperatura
p Tx > Tamb
e medir a temperatura final de equilı́brio Teq . Se necessário, use 2 ⇠
= 1,4 e 26 ⇠ = 5,1.
(a) Escreva a equação necessária para obter cx a partir das grandezas fornecidas.
(b) Considerando K = 30,0 cal/o C, o calor especı́fico da água cH2 O = 1,0 cal/(g o C) e utili-
zando mH2 O = 50,0 g, mx = 200 g, Tx = 37,8 o C, Tamb = 25,0 ± 0,1 o C, o estudante determinou
que a temperatura final de equilı́brio do sistema foi de Teq = 27,8±0,1 o C. Calcule cx . Expresse
seu resultado com o erro associado ao valor de cx .

2
Q6. Um anel fino de raio R e carga total Q > 0 uniformemente distribuı́da ao longo de sua
circunferência está fixo no plano xy de um sistema de coordenadas e tem seu centro na origem
O. Seja P um ponto com coordenadas (0,0,z) (ver figura abaixo).

z
dE
P

θ
r
z
y

R
O dQ x

(a) Somando as contribuições de todos os elementos de carga do anel, encontre módulo, direção
~
e sentido do campo elétrico E(z) no ponto P .
(b) Proceda analogamente ao item (a) e calcule o potencial elétrico V (z) no ponto P .
(c) Uma partı́cula pontual de carga q < 0 e massa m parte do repouso de um ponto com
coordenadas (0,0,z0 ), muito distante da origem (ou seja, z0 R) e viaja ao longo do eixo z.
Qual é a sua velocidade quando ela passa pelo centro do anel? Considere desprezı́veis os efeitos
da radiação eletromagnética emitida pela partı́cula no seu trajeto em direção ao centro do anel.

Q7. Um aro quadrado rı́gido de arame com lado L tem resistência elétrica total R. O aro está no
plano xy de um sistema de coordenadas e move-se com velocidade ~v para fora da região onde
há um campo magnético uniforme B ~ (area sombreada na figura abaixo) apontando para fora
da página (sentido de z positivo). Considere o instante empque o vértice da esquerda do aro
está a uma distância s dentro da area sombreada (0 < s < 2L/2).

y v
x L

B
R

(a) Calcule o fluxo do campo magnético através do aro como função de s.


(b) Determine o valor e o sentido de circulação da corrente elétrica induzida no aro.
(c) Calcule a força magnética total (módulo, direção e sentido) sobre o aro quando a corrente
induzida circula nele. Que força adicional à força magnética deve ser aplicada no aro para que
ele se mova com velocidade constante sob a ação exclusiva dessas duas forças?

1
Q8. Considere a dinâmica quântica não relativı́stica de um elétron (massa m e carga e) movendo-
se ao longo do eixo x num potencial de oscilador harmônico unidimensional com frequência
~ = E x̂ ao longo do mesmo eixo.
angular !. O elétron também está sujeito a um campo elétrico E

(a) Escreva a Hamiltoniana total do sistema.


(b) Sejam |ni (n = 0,1,2, . . . ) os auto-estados do oscilador harmônico. Vamos considerar
agora o efeito do campo elétrico como uma pequena perturbação que modifica muito pouco
as auto-energias e os auto-estados do oscilador harmônico. Seja V̂E o termo da Hamiltoniana
devido ao campo elétrico. Nesse caso, a correção da energia do primeiro estado excitado em
ordem linear em E é dada pelo valor médio de V̂E em |1i. Calcule essa correção.
(c) A correção do primeiro auto-estado excitado em ordem linear em E é dada por
r
(1) ~ eE ⇣ p ⌘
| 1 i= |0i 2 |2i . (2)
2m! ~!
A correção da energia do primeiro estado excitado em ordem quadrática em E é dada pelo
elemento de matriz de V̂E entre |1i o estado da Eq. (2). Calcule essa correção.
(d) Calcule as auto-energias exatas do sistema (oscilador harmônico mais campo elétrico)
através de uma transformação de coordenadas. Compare o resultado exato com o cálculo per-
turbativo dos itens (b) e (c). Eles concordam? Por quê?

Q9. Suponha que um planeta extra-solar esteja a uma distância de c T anos-luz da Terra (c é a
velocidade da luz e T é o tempo em anos que esta leva para viajar da Terra até lá). Uma
expedição é planejada para enviar astronautas ao planeta de tal forma que eles envelheçam
3T /4 anos durante a viagem de ida. A viagem é quase toda feita a uma velocidade constante.
Por isso, desconsidere os pequenos trechos com movimento acelerado.
(a) Qual deverá ser o módulo da velocidade constante dos astronautas, em relação à Terra, na
ida?
(b) De acordo com os astronautas, qual será a distância a ser percorrida na ida?
(c) A cada ano (de acordo com o relógio da nave) os astronautas enviam um pulso de luz para
a Terra. Qual é a periodicidade dos pulsos recebidos na Terra?
(d) Na metade da jornada de ida um casal de astronautas decide retornar à Terra em um
módulo espacial. De acordo com os astronautas que permanecem na nave, o módulo retorna
em direção à Terra com velocidade 5c/6. Calcule o tempo total (medido na Terra) que o casal
de astronautas terá ficado fora do nosso planeta.

Q10. Um sistema de N osciladores quânticos unidimensionais, localizados e independentes está em


equilı́brio com um reservatório térmico à temperatura T . As energias de cada oscilador são
dadas por ✓ ◆
1
En = ~!0 n + n = 1,3,5,7, . . .
2
Note que os valores assumidos por n são apenas os naturais ı́mpares.
(a) Obtenha uma expressão para a energia interna por oscilador u como função da temperatura
T . Qual é a expressão para u no limite clássico (~!0 ⌧ kB T )? Esboce um gráfico de u por T .
(b) Obtenha uma expressão para a entropia por oscilador s como função da temperatura T .
Qual é a expressão para s no limite clássico? Esboce um gráfico de s por T .

2
Q1. Uma esfera de bronze sólida de massa m e raio r rola sem deslizar ao longo de um plano incli-
nado após ser solta do repouso de uma altura h. O momento de inércia da esfera em relação
a um eixo que passa pelo seu centro é I = 2mr2 /5 e a aceleração da gravidade é g. O plano
inclinado forma um ângulo ✓ com a horizontal, como mostra a figura.

!
(a) Há atrito entre a esfera e o plano inclinado? Como você chegou a essa conclusão?
(b) Há conservação de energia mecânica? Justifique sua resposta levando em consideração o
respondido no item (a).
(c) Utilizando considerações de energia, determine a velocidade com que a esfera atinge a base
do plano inclinado.
(d) Obtenha a velocidade na base do plano inclinado já calculada no item (c) utilizando agora
considerações de dinâmica (ou seja, aplicando a segunda lei de Newton).

Q2. Considere uma massa m presa à extremidade de uma haste inextensı́vel de massa desprezı́vel
e comprimento l. A outra extremidade da haste está presa a um ponto fixo e o sistema haste-
massa move-se em um plano vertical num local onde a aceleração da gravidade é g.
(a) Escreva a Lagrangiana do sistema.
(b) Obtenha a equação de movimento que descreve o sistema.
(c) Determine os pontos de equilı́brio do sistema e classifique-os quanto à estabilidade, justifi-
cando suas respostas.
(d) Encontre a frequência de pequenas oscilações em torno do ponto de equilı́brio estável.

Q3. No processo Compton de espalhamento relativı́stico, um fóton de energia-momento (E0 , p~0 )


incide sobre um elétron de massa m em repouso. É observado um fóton emergente em uma
direção que forma um ângulo ✓ com a direção de incidência, com energia-momento (E, p~).
(a) Denotando o momento do elétron espalhado por p~e , escreva as equações para a conservação
de energia-momento.
(b) Obtenha a relação
1 1 1
= (1 cos ✓) .
E E0 mc2
(c) Supondo que o comprimento de onda do fóton incidente seja 0 , determine o comprimento
de onda do fóton espalhado quando ✓ = ⇡/2.
(d) Nas mesmas condições do item anterior, qual é a energia cinética do elétron espalhado?
Expresse a resposta em termos de 0 , C ⌘ h/ (mc) e constantes universais.

1
Q4. Considere a dinâmica quântica unidimensional de uma partı́cula de massa m sob a ação de um
potencial harmônico. Seu Hamiltoniano é dado por
✓ ◆
p̂2 1 † 1
Ĥ = + m! x̂ = ~! â â +
2 2
,
2m 2 2

onde ! é a frequência angular do oscilador e


r
m! i
â = x̂ + p p̂.
2~ 2m!~

Os auto-estados |ni (n = 0,1, . . .) do Hamiltoniano são não-degenerados, são auto-estados do


operador número N̂ = ↠â e satisfazem as relações
p p
â |ni = n |n 1i , ↠|ni = n + 1 |n + 1i .

(a) Calcule os elementos de matriz dos operadores x̂ e p̂ na base dos auto-estados do Hamilto-
niano.
(b) Calcule o valor esperado do operador x̂2 para um auto-estado qualquer |ni .
(c) Calcule a razão entre a energia total média e a energia potencial média para um auto-estado
qualquer |ni .
(d) Use a equação de movimento dos operadores na representação de Heisenberg

dÔH (t) h i
i~ = ÔH (t),Ĥ ,
dt

onde ÔH (t) = eiĤt/~ Ôe iĤt/~


, para obter a evolução temporal do operador âH (t).

Q5. Uma máquina térmica de um gás ideal monoatômico funciona de acordo com um ciclo que tem
inicialmente uma expansão adiabática partindo de um estado A de volume V0 até um estado
B cujo volume é rV0 (com r > 1). O processo é seguido por uma contração isotérmica de B
até o estado C, que possui o mesmo volume de A. Finalmente, o ciclo se completa por uma
compressão isovolumétrica de C até A.
(a) Represente no diagrama P V o ciclo realizado por esta máquina térmica.
(b) Calcule (i) o trabalho total realizado pelo gás e (ii) o calor injetado no gás, ambos durante
um ciclo completo. Deixe sua resposta em função de r, ⌘ cP /cV e das temperaturas extremas
Tmax e Tmin , que são, respectivamente, as temperaturas máxima e mı́nima entre as quais o ciclo
opera. Lembre-se de que cP cV = R.
(c) Determine o rendimento do ciclo.
(d) Escreva o rendimento do ciclo apenas em função de Tmax e Tmin (caso já não o tenha feito no
item (c)). Considere o caso em que Tmax = 2Tmin > 0. Determine a razão entre o rendimento
desta máquina e o rendimento de um ciclo de Carnot. Qual tem o maior rendimento? Isso faz
sentido com o que se espera da segunda lei da termodinâmica? Justifique sua resposta.

2
Q6. Uma esfera isolante sólida de raio a tem densidade de carga uniforme ⇢ e carga total Q. Uma
esfera oca condutora não carregada, cujos raios interno e externo são b e c, respectivamente, é
concêntrica à esfera isolante, como mostra a figura abaixo.

(a) Determine a magnitude do campo elétrico nas regiões:


(i) r < a; (ii) a < r < b; (iii) b < r < c e (iv) r > c.
(b) Ache a carga induzida por unidade de área nas superfı́cies interna e externa do condutor.
(c) Esboce o gráfico da magnitude do campo elétrico E versus r. Identifique em seu gráfico
cada uma das regiões citadas no item (a).

Q7. Considere as equações de Maxwell na forma diferencial e resolva cada item abaixo.
(a) Derive, mostrando todos os passos, as equações de onda no vácuo em sua forma vetorial
para os campos elétrico e magnético. Lembre-se:
r⇥(r⇥V) = r(r·V) r2 V e r·(r⇥V) = 0
(b) Escreva a equação de onda para uma função escalar qualquer f (~r,t) e, comparando com as
expressões obtidas no item (a), determine a velocidade de propagação para ambos os campos.
(c) Uma possı́vel solução da equação obtida no item (a) é a solução do tipo onda plana li-
nearmente polarizada. Suponha um campo eletromagnético do tipo onda plana linearmente
polarizada que esteja se propagando na direção ẑ. Considerando que ! é a frequência angular,
k o número de onda, E0 e B0 as amplitudes dos campos elétrico e magnético, respectivamente,
~ eB
escreva explicitamente qual é o módulo e a direção de E ~ em função da posição e do tempo.
(d) Partindo agora das equações de Maxwell na presença de cargas e correntes, derive a equação
que relaciona as densidades de carga e de corrente elétrica (equação da continuidade). Que lei
de conservação é expressa matematicamente por esta equação?

Q8. Considere uma partı́cula de spin 1/2 sob a ação de um campo magnético uniforme B~ = B ẑ.
O Hamiltoniano para este problema é dado por
Ĥ = B Ŝz ,
onde é uma constante. Sejam os estados |+i e | i tais que Ŝz |±i = ±(~/2) |±i .
(a) Quais são os auto-valores do Hamiltoniano? p
(b) No instante t = 0 a partı́cula se encontra no estado | (0)i = [|+i | i ] / 2. Calcule o
estado da partı́cula em um instante t > 0 qualquer.
(c) Calcule a média dos operadores Ŝx , Ŝy e Ŝz para qualquer instante t 0. Lembre-se de
que Ŝx |± i = (~/2) |⌥ i e Ŝy |± i = ±i(~/2) |⌥ i.
(d) Qual é o menor valor de t > 0 para o qual o estado volta a ser igual ao estado inicial?

1
Q9. Se dois eventos no espaço-tempo são separados espacialmente pelo vetor x x̂ + y ŷ + z ẑ
e temporalmente por t, o intervalo invariante entre eles, cujo valor independe do referencial
inercial, é definido como
s2 ⌘ x 2 + y 2 + z 2 c 2 t 2 .
(a) Eventos (1) e (2) ocorrem em posições distintas (x1 ,y1 ,z1 ) e (x2 ,y2 ,z2 ), respectivamente,
de um dado referencial inercial (S) e são tais que o intervalo invariante é positivo. Existe um
referencial inercial onde tais eventos ocorrem em um mesmo ponto do espaço? Justifique.
(b) Nas mesmas condições do item (a), o evento (2) poderia ter sido causado pelo evento (1)?
Justifique sua resposta considerando a propagação de um sinal de (1) para (2) com velocidade
V~ = Vx x̂ + Vy ŷ + Vz ẑ.
(c) Um relógio está em repouso em um referencial (S0 ) que se move com velocidade V~ em
relação a (S).
(i) Qual é o sinal do intervalo invariante entre eventos que caracterizam duas posições
sucessivas dos “ponteiros do relógio” (desconsidere as dimensões espaciais do relógio)?
(ii) Obtenha a relação entre o intervalo de tempo próprio t0 (medido em S0 ) e o intervalo
de tempo t medido em (S).
(d) A separação espacial entre uma fonte F e um detector D de partı́culas é Lx̂, no referencial
do laboratório (referencial S). Considere os eventos EF e ED , de produção e detecção de uma
partı́cula, respectivamente. Suponha que essa partı́cula se mova de F a D com velocidade
constante V~ = V0 x̂ no referencial do laboratório.
(i) Quais são as separações no espaço x e no tempo t entre EF e ED no referencial do
laboratório?
(ii) Seja L0 a distância entre F e D no referencial da partı́cula. Quais são as separações
no espaço x0 e no tempo t0 entre EF e ED no referencial da partı́cula?
(iii) Determine a relação entre L0 e L.

Q10. Num modelo para um cristal sólido podemos supor que os N átomos sejam equivalentes a
3N osciladores harmônicos clássicos, unidimensionais, independentes, de massa m, que oscilam
com a mesma frequência angular ! em torno de sua posição de equilı́brio. A uma distância
x desta posição a energia potencial é dada por U = m! 2 x2 /2. Conhecendo-se alguns dados
experimentais, é possı́vel estimar, em termos da distância inter-atômica a baixas temperaturas
d, a raiz do deslocamento quadrático médio dos átomos quando ocorre a fusão. A resolução
dos itens abaixo permite fazer esta estimativa. Suponha que o sólido se encontre em equilı́brio
térmico a uma temperatura absoluta T .
(a) Considere que o número de estados numa célula do espaço de fase (x,p) seja dado por
(dxdp)/h, onde h é a constante de Planck. Obtenha a função de partição para o oscilador
harmônico, Z(T,!).
(b) Calcule o número médio de osciladores cuja posição se encontra entre x e x + dx.
(c) Obtenha uma expressão para a energia potencial média, hU i por oscilador unidimensional.
Compare o resultado com o valor esperado pelo teorema da equipartição.
(d) Seja x20 o deslocamento quadrático médio em torno do equilı́brio quando o sólido se funde e
seja f = x0 /d. Usando hU i = m! 2 x20 /2, estime f para um dado elemento cuja massa atômica
é m = 1.0 ⇥ 10 25 kg, a temperatura de fusão é TF = 1400 K, d = (10/3) Å = (10/3) ⇥ 10 10 m
e a frequência é tal que ~!/kB = 300 K.

2
Q1. Definindo-se o vetor de Hertz Z~ pelas expressões:
~
~ ·Z
∇ ~ = −φ; ~ = µ0 0 ∂ Z ,
A (1)
∂t
~ são, respectivamente, os potenciais escalar e vetor.
onde φ e A

a) Mostre que os potenciais satisfazem o calibre de Lorentz:

∇ ~ + µ0 0 ∂φ = 0;
~ ·A (2)
∂t

b) Demonstre que para um meio sem fontes (ρ = 0, J~ = 0) e de µ = µ0 o vetor Z


~ satisfaz às
seguintes expressões:
~
1 ∂ 2Z P~ ~ ~
~−
∇2 Z = − ; ~ = 1∇
B ~ × ∂Z ; ~ =∇
E ~ ×∇ ~ − P,
~ ×Z (3)
c2 ∂t2 0 c2 ∂t 0

onde P~ é o vetor de polarização.

Q2. Considere um disco vazado muito fino, com raio interno r1 e raio externo r2 , deitado sobre o
plano xy e com o eixo centrado em z = 0 (conforme ilustrado na figura 1).

Figura 1: Disco vazado.

O anel tem densidade superficial de carga dada por:


σ0
σ(r) = , (4)
r
p
onde r = x2 + y 2 .
~ = y = 0,z) sobre o eixo z;
a) Encontre o campo elétrico E(x
b) Suponha agora que o anel comece a girar com velocidade angular ω0 . Encontre a densidade
de corrente J~s = σ~v , onde ~v é a velocidade linear;
c) Encontre o campo magnético H(x ~ = y = 0,z) sobre o eixo z, gerado pela densidade de
~
corrente Js .

1
Q3. Um pı́on positivo π+ pode decair segundo a reação π+ → µ+ + νµ , ou seja, ele pode decair
em um múon positivo µ+ acompanhado por um neutrino muônico νµ . Desprezando a massa
mν do neutrino e considerando um pı́on inicialmente em repouso num referencial inercial S,
determine, no mesmo referencial, em termos das massas do pı́on (mπ ) e do múon (mµ ):
a) O módulo do momentum linear do múon.
b) A energia total do múon.
c) A velocidade do múon.
d) A distância que, em média, um múon percorre (no vácuo) antes de também decair. Use o
sı́mbolo τ para o tempo de vida médio do múon medido no próprio referencial da partı́cula.

Q4. Considere uma partı́cula não relativı́stica, de massa m, executando um movimento harmônico
simples com frequência ν.
a) Determine, em termos de ν, os nı́veis de energia
H E permitidos para esta partı́cula a partir
da regra de quantização de Bohr-Sommerfeld pq dq = nh.
b) Considere um sistema contendo um grande número destas partı́culas em equilı́brio térmico.
A partir dos nı́veis de energia permitidos para cada partı́cula, determinados no ı́tem anterior,
calcule a energia total média hEi, onde P (En ) = Ae−En /kB T é a função de distribuição.

Q5. Considere uma máquina de Carnot operando com um paramagneto ideal, cuja equação de
estado é dada pela lei de Curie
H
M = D ,
T
sendo M a magnetização, H o campo magnético, T a temperatura e D uma constante. A
variação de energia interna é dada em termos da variação da entropia e da magnetização por
dU = T dS +H dM (o termo HdM é análogo ao termo −P dV para o gás ideal), e vale também
que dU = CM dT , com CM constante.

a) Determine a relação que vincula os valores iniciais da magnetização e da temperatura Mi ,


Ti aos valores finais Mf , Tf em uma transformação adiabática, em termos de CM e D.
b) Represente o ciclo, composto por duas transformações adiabáticas e duas transformações
isotérmicas, em um diagrama H-M . As isotermas correspondem respectivamente a uma
temperatura mais alta, TQ , e outra mais baixa, TF . Indique os quatro estados nos vértices
do diagrama como (M1 ,H1 ) (inı́cio do ciclo, no valor mais alto para a magnetização e à
temperatura TQ ), (M2 ,H2 ), (M3 ,H3 ), (M4 ,H4 ).
c) Calcule o trabalho total realizado no ciclo, em função de M1 , M2 , TQ , TF e da constante
D.
d) Obtenha a eficiência do ciclo, dada pela razão entre o trabalho total realizado e o calor
absorvido (à temperatura TQ ).

2
Q6. Um disco de raio R é composto por duas metades cada uma com densidades superficiais de
massa respectivas de 1ρ e de 2ρ.

a) Qual é o momento de inércia em relação ao eixo (perpendicular ao plano do disco) que passa
pelo seu centro geométrico G?
b) Encontre as coordenadas x1 e x2 do centro de massa do disco.
c) Qual é o momento de inércia em relação ao eixo (perpendicular ao plano do disco) que passa
pelo seu centro de massa?
d) Considere o movimento em linha reta do disco sobre um plano horizontal perpendicular ao
plano do disco, sem deslizar. Encontre λ(θ), implicitamente definido por

v(t) = λ(θ) R ,
dt
onde θ é o ângulo entre o eixo vertical e a reta que passa pelo centro geométrico e o centro de
massa (veja a figura), v(t) é o módulo da velocidade do centro de massa, e dθ dt
é o módulo da
velocidade de rotação do disco.

Q7. Considere um objeto de massa M que se desloca sob ação de uma força central do tipo cou-
lombiana modificada por uma força proporcional ao inverso de r3 ,
k q
F (r) = − 2
− 3,
r r
onde r é a coordenada radial, e k e q são constantes positivas. Considere que a energia total
do sistema é descrita por
M 2 M 2 2 k q
E= ṙ + r θ̇ − − 2 ,
2 2 r 2r
e que o momento angular, do sistema é dado por L = M r2 θ̇.

1
a) Para o caso em que o objeto descreva uma órbita circular (de equilı́brio) encontre o raio da
órbita em função dos parâmetros k, q, M e L, do sistema.

b) Para as mesmas condições do item a), encontre a energia total, E, em função dos parâmetros
k, q, M e L, do sistema.

c) Ao identificar o potencial efetivo para o movimento radial como

L2 k q
Vef (r) = 2
− − 2,
2mr r 2r
verifique sob quais condições sobre as constantes q, L e M , a coordenada radial da órbita
circular obedece uma configuração de equilı́brio estável.

d) No caso da coordenada radial da partı́cula se deslocar da condição de equilı́brio (estável) e


passar a oscilar de forma aproximadamente harmônica (em torno do raio da órbita circular),
encontre a relação entre o perı́odo de oscilação radial e o perı́odo de revolução (movimento
angular) em função das constantes q, M e L.

Q8. Seja um sistema composto por um par A e B de spins 1/2 descrito pelo estado
|ψi = α|A+ i ⊗ |B− i + β|A− i ⊗ |B+ i + γ|A− i ⊗ |B− i + δ|A+ i ⊗ |B+ i

(com α,β,γ,δ ∈ C) pertencente ao espaço de Hilbert HA ⊗ HB , onde o estado |A± i satisfaz


hA± |A± i = 1, hA± |A∓ i = 0 e

~
ŜzA |A± i = ± |A± i, Ŝ∓A |A± i = ~|A∓ i, Ŝ±A |A± i = 0.
2
E analogamente para |B± i. Lembrando que

Ŝz ≡ ŜzA ⊗ IˆB + IˆA ⊗ ŜzB

assim como
Ŝx ≡ ŜxA ⊗ IˆB + IˆA ⊗ ŜxB , Ŝy ≡ ŜyA ⊗ IˆB + IˆA ⊗ ŜyB
com I A ,I B sendo operadores identidade atuando nos respectivos espaços de Hilbert, responda:
a) Qual é a dimensão do espaço de Hilbert HA ⊗ HB do par de spins A e B?
b) Seja o estado |ψi com α = β = γ = 0. Qual é o valor de δ ∈ C mais geral que normaliza
|ψi.

c) Seja o estado |ψi com α = −β = 1/ 2 e γ = δ = 0. Qual é o valor esperado do operador
Ŝz nesse estado?

d) Seja o estado |ψi com α = β = 1/ 2 e γ = δ = 0. Determine se |ψi é um auto-estado do
operador de spin Ŝ 2 ≡ Ŝx2 + Ŝy2 + Ŝz2 . Se for, qual é o auto-valor correspondente? (Sugestão:
h i
lembrar que Ŝ± = Ŝx ± iŜy e que Ŝx ,Ŝy = i~Ŝz .)

2
Q9. Seja um oscilador harmônico com frequência ω, massa m e com hamiltoniana
Ĥ = (1/2 + n̂)~ω, (5)

onde n̂ ≡ ↠â com n̂|ni = n|ni e lembramos que os operadores de abaixamento e levantamento
satisfazem √
â|ni = n|n − 1i

↠|ni = n + 1|n + 1i
Supondo que o oscilador esteja em um estado coerente |zi definido por

â|zi = z|zi,

responda
a) Qual é o valor de hz|n̂|zi para z = 21 exp(iπ/4), supondo que |zi esteja normalizado?
b) Supondo que em t = 0 o oscilador esteja no estado fundamental |0i, calcule a forma do
estado no instante t = 1/10 s para ω = 5π s−1 .
P+∞
c) Quanto vale cn (como função de n e z) para que o estado coerente |zi = n=0 cn |ni
(expandido na base de auto-estados |ni do operador número n̂) esteja normalizado? (Lembre-
se que ex = +∞ n
P
n=0 x /n!)
d) Use o resultado do item anterior e calcule o valor numérico de |hz 0 |zi|2 para z = 1/2 exp(iπ/4)
e z 0 = 1/4 exp(iπ/4).

Q10. Considere um sistema de N spins 1/2 não-interagentes, com momento de dipolo magnético
de módulo µ, na presença de um campo magnético uniforme B.

a) Escreva a hamiltoniana do sistema.


b) Considerando o sistema em equilı́brio térmico a temperatura inversa β = 1/kB T , calcule
a função de partição Z(β,B).
c) Calcule a magnetização M como função de T e B.
d) Obtenha a expressão para M no limite de altas temperaturas e campo magnético fraco.

3
Q1. Uma espira condutora retangular (comprimento a, largura b e resistência R) situa-se nas vizi-
nhanças de um fio reto infinitamente longo que é percorrido por uma corrente i para a direita,
conforme a figura. A espira afasta-se do fio com uma velocidade constante ~v , de forma que a
distância do centro da espira ao fio é dada por s(t) = s0 + vt. Calcule:
a) o módulo do campo magnético produzido pela corrente num ponto situado a uma distância
r do fio. Indique a direção e o sentido do campo na região delimitada pela espira;
b) o fluxo magnético na região delimitada pela espira para um dado valor de s(t);
c) a força eletromotriz induzida na espira para uma certa distância s(t);
d) a corrente induzida na espira, iind . Indique o sentido da mesma.

Q2. Um meio condutor tem condutividade elétrica σ, permeabilidade magnética µ0 e permissividade


elétrica  = K0 , em que K é a constante dielétrica real. A equação de onda para o campo
~ − K 12 ∂ 2 E2~ − σ 12 ∂ E~ = 0, sendo 12 = µ0 0 .
elétrico neste meio é dada por ∇2 E c ∂t 0 c ∂t c
~
a) Mostre que a função de onda plana monocromática E(z,t) =E~ 0 ei(ωt−q̃z) é solução da equação
diferencial acima. Encontre a relação entre o número de onda complexo, q̃, e a frequência
~
angular, ω, para que E(z,t) seja solução. Mostre também que q̃ se torna real no caso de um
meio isolante.
b) Encontre a constante dielétrica complexa, K̃, usando a relação entre o número de onda e a
2
constante dielétrica, q̃ 2 = K̃ ωc2 . Verifique que a parte real de K̃ é igual a K , como esperado,
e explicite a parte imaginária de K̃.
c) Faça a aproximação para baixas frequências na expressão
pda constante dielétrica complexa
do item (b) e calcule o ı́ndice de refração complexo, ñ = K̃ . Mostre que as partes real e
imaginária de ñ são iguais neste caso.
d) A profundidade de penetração da onda no meio condutor, δ, é dada pelo inverso da parte
imaginária do número de onda, qi , ou seja, δ = 1/qi . Lembre-se de que q̃ = ñ ωc e calcule a
ω
profundidade de penetração para a prata (Ag) na região de micro-ondas (f = 2π = 10GHz),
para a qual vale a aproximação do item (c). A condutividade da prata nesta faixa de frequências
é σAg = 3 × 10+7 (Ωm)−1 . Aproxime o resultado do cálculo e obtenha a ordem de grandeza de
δAg (1 m, 10 cm, 1 cm ...).

1
Q3. Considere 2 fótons que se propagam, ao longo do eixo x, em sentidos opostos. As energias dos
fótons são 5 MeV e 2 MeV, respectivamente.
a) Calcule a velocidade relativa entre os fótons.
b) Qual é o valor da energia total do sistema?
c) Qual é momento total do sistema?
d) Calcule a energia de repouso do sistema.

Q4. Um fóton de raio-X com comprimento de onda λ = 10−10 m, é retroespalhado em um experi-


mento Compton, ou seja, o ângulo de espalhamento é de 180o em relação ao eixo de incidência.
a) Calcule a frequência do fóton retroespalhado.
b) Quais são a direção e o sentido do momento do elétron ejetado no espalhamento, em relação
à do fóton incidente?
c) Qual é o módulo da velocidade do elétron ejetado no espalhamento?

Q5. Um recipiente cilı́ndrico de seção reta circular A e base fixa foi posicionado verticalmente sobre
uma superfı́cie plana e preenchido com um gás ideal. Sobre sua extremidade superior, aberta,
foi perfeitamente ajustado um êmbolo circular móvel de massa M . Suponha que o êmbolo
permaneça orientado horizontalmente e só deslize para cima e para baixo, sem atrito, em
contato com a parede interna do cilindro. Considere dada a razão γ entre os calores especı́ficos
do gás a pressão constante e a volume constante.

a) Calcule a pressão de equilı́brio para o gás no recipiente, sendo p0 a pressão atmosférica.


b) Escreva a expressão para a variação da pressão p em termos da variação do volume
V decorrente de um pequeno deslocamento do êmbolo. Suponha que, para pequenos
deslocamentos do êmbolo, os estados do gás sejam descritos por um processo quase-
estático adiabático.
c) Determine a força adicional exercida sobre o êmbolo quando o mesmo tiver um desloca-
mento dx em relação à posição de equilı́brio.
d) Obtenha a frequência angular para pequenas oscilações do êmbolo a partir da posição de
equilı́brio, em termos de V , A, M , p0 e γ.

2
Q6. Uma partı́cula de massa m está submetida2
a uma força central conservativa cuja energia
potencial é dada por U (r) = k r2 − a2 e−b r , em que r é a coordenada radial esférica, e k, a


e b são constantes reais e positivas.


a) Determine as unidades das constante k, a e b no SI (Sistema Internacional de Unidades).
b) Esboce um gráfico da função U (r), determinando seus pontos de máximo e mı́nimo em
função dos parâmetros dados.
c) Determine as faixas de energia E da partı́cula para as quais (i) a partı́cula está em órbitas
ligadas e (ii) não ligadas. (iii) Determine as condições, se existem, para a existência de órbitas
com raio constante.
d) Determine a força que age sobre a partı́cula, diga quais as situações de equilı́brio, se existi-
rem, e, em caso afirmativo determine a frequência de oscilação da partı́cula para movimentos
radiais próximos do(s) ponto(s) de equilı́brio estável.

Q7. Uma partı́cula de massa m está confinada sobre uma superfı́cie esférica de raio fixo a, e nenhuma
força externa age sobre a mesma.
a) Determine a lagrangiana da partı́cula usando coordenadas apropriadas no espaço tridimen-
sional (R3 ) e estabeleça a equação de vı́nculo.
b) Usando o método dos multiplicadores de Lagrange, encontre as equações de movimento
e determine a força de vı́nculo, i.e., determine o multiplicador de Lagrange e interprete o
resultado.
c) Estabeleça as constantes do movimento da partı́cula.
d) Supondo, agora, que o raio da esfera varia no tempo com a função a(t) = a0 (1 + cos ωt),
com a0 e ω constantes, determine as constantes de movimento da partı́cula.

Q8. Seja uma partı́cula livre de massa m confinada a uma circunferência de perı́metro L.
a) Escreva a equação de Schroedinger correspondente.
b) Calcule a função de onda normalizada ψ = ψ(t,x), onde x é a posição da partı́cula (0 ≤ x <
L), supondo que ela tenha valores bem definidos de momento e energia: p e E, respectivamente.
c) Supondo que a partı́cula esteja em um auto-estado de energia, quais são os dois menores
autovalores correspondentes (não nulos)?
d) Seja uma partı́cula em um auto-estado de energia com o menor valor não nulo de energia.
Escreva sua função de onda para que tenha uma densidade de probabilidade de ser encontrada
entre x e x + δx igual a (2/L)[cos(2πx/L)]2 . (Lembrar que (cos x)2 = (cos 2x + 1)/2.)

1
Q9. Seja um sistema composto por um par A e B de spins 1/2 descrito pelo estado
A B A B

|ψi = α |z+ i ⊗ |z− i − |z− i ⊗ |z+ i

onde
~ A
Ŝx |xA A A
± i = ± |x± i, hx± |x± i = 1, (1)
2
A ~ A A A
Ŝy |y± i = ± |y± i, hy± |y± i = 1, (2)
2
A ~ A A A
Ŝz |z± i = ± |z± i, hz± |z± i = 1, (3)
2
(e analogamente para B) e onde escrevemos os operadores de spin como
     
~ 0 1 ~ 0 −i ~ 1 0
Ŝx = , Ŝy = , Ŝz = , (4)
2 1 0 2 i 0 2 0 −1

na base de auto-estados de Ŝz :


   
1 0
|z+ i = , |z− i = (5)
0 1

Responda:
a) Qual é o valor de α ∈ R para que |ψi esteja normalizado?
b) Qual é a probabilidade de se medir na direção z: −~/2 para o spin A e +~/2 para o spin
B?
c) Qual é a probabilidade de se medir na direção x: +~/2 para o spin A e −~/2 para o spin
B?
d) Qual é a probabilidade de se medir na direção z −~/2 para o spin A e na direção x +~/2
para o spin B?

Q10. Considere um sistema composto por um número grande N de moléculas distinguı́veis, que
não interagem entre si. Cada molécula tem dois estados de energia possı́veis: 0 e  > 0.

a) Obtenha a densidade de entropia S/N do sistema como função apenas da energia média
por molécula E/N , de  e da constante de Boltzmann kB .
b) Considerando o sistema em equilı́brio térmico à temperatura inversa β = 1/kB T , calcule
E/N .
c) Qual o valor máximo para E/N no caso acima? Compare com o valor máximo dessa
grandeza caso fosse possı́vel que todos os elementos do sistema estivessem no estado de
energia máxima.

2
Q1. a) Um cilı́ndrico dielétrico maciço, de comprimento infinito e raio a, possui uma densidade
de carga volumétrica uniforme e positiva ρ. Uma casca cilı́ndrica, também dielétrica, de raio
b > a, com eixo comum ao cilindro, tem uma densidade de carga superficial uniforme e negativa
σ, de forma que a carga total do cilidro mais casca, em certo comprimento, é zero, e portanto
~
σ = −ρa2 /2b. Calcule o campo elétrico E(r) para as regiões r < a, a < r < b e b < r sendo r
a distância ao eixo do cilindro.
b) Considere em seguida que o conjunto cilindro mais casca se move para a direita com ve-
locidade ~v . O movimento dá origem a uma corrente elétrica I = πa2 ρv no cilindro maciço,
para a direita e uniformemente distribuida na seção reta, de forma que a densidade de corrente
fica sendo dada por J~ = ρ~v . Da mesma forma, a casca em movimento dá origem a uma cor-
rente de mesma intensidade I, mas em sentido contrário (para a esquerda). Calcule a indução
magnética B ~ para as regiões r < a, a < r < b e b < r.

a
ρ
b

Q2. O campo elétrico de uma onda plana monocromática no vácuo é dado por
~
E(z,t) = (E1 x̂ + E2 ŷ)ei(kz−ωt) .

onde x̂ e ŷ são versores cartesianos nas direções x e y, respectivamente, e E1 e E2 são constantes.

~
a) Encontre a indução magnética B(z,t).
b) Mostre que o campo elétrico e a indução magnética são ortogonais entre si.
c) Encontre o vetor de Poynting da onda.

Q3. Considere um gás de moléculas diatômicas com frequência de oscilação ω e momento de inércia
I. À temperatura ambiente, as energias dos estados moleculares vibracionais são muito maiores
do que kB T . Portanto, a maioria das moléculas se encontra no estado vibracional de menor
energia. Por outro lado, a energia caracterı́stica dos estados rotacionais é muito menor do
que kB T . A energia rotacional-vibracional E(n,`) do estado de uma molécula diatômica é
caraterizada pelo número quântico n, para a energia vibracional, e pelo número quântico `,
para a energia rotacional.

a) Escreva E(n,`) para n = 0 e ` qualquer.


b) Suponha que uma molécula sofra uma transição de um estado inicial com n = 0 e ` qual-
quer para um estado excitado com n = 1. Determine as duas energias totais permitidas
para a molécula após a transição, lembrando que a regra de seleção impõe ∆` = ±1.
Calcule a diferença de energia entre esses dois estados permitidos e o estado inicial, bem
como as respectivas frequências de transição.
c) Considere o estado da molécula no qual n = 0 e ` qualquer. Sabendo que a degenerescência
do estado é 2` + 1, determine a população do estado rotacional-vibracional, N (E), como
função de E, a partir da distribuição de Boltzmann.

1
d) Para n = 0, o estado ` = 0 não é o estado mais populado à temperatura ambiente. Para
pequenos valores de `, a população do estado aumenta ligeiramente em relação a ` = 0
por causa do aumento da densidade de estados. Para grandes valores de `, a população
diminui por causa do fator de Boltzmann. Determine o valor de ` para o qual a população
é máxima.

Q4. Suponha que um fóton encontre um elétron que está inicialmente em repouso no referencial
S, como na figura 1A. Na maioria das vezes, o fóton é simplesmente desviado da trajetória
original, mas, ocasionalmente, o evento resulta no desaparecimento do fóton e na criação de
um par elétron-pósitron, na presença do elétron original. Suponha que os detalhes da interação
que produziu o par sejam tais que as três partı́culas resultantes se movam para direita, como
na figura 1B, com a mesma velocidade u, isto é, que estejam todas em repouso no referencial
S0 , que está se movendo para a direita com velocidade u em relação a S.

a) Escreva as leis de conservação de energia e momento antes e depois da criação do par.


b) Usando a conservação da energia-momento no caso relativı́stico, obtenha no sistema S0
a energia do fóton para que seja criado um par de partı́culas com energia equivalente à
energia de repouso de 2 elétrons.
c) Utilize a relação m20 c4 = E 2 − p2 c2 para obter a relação u/c = pc/E.
d) Determine a partir do item (c) a velocidade u com a qual as três partı́culas se movem no
referencial S.

(A) (B)

e−
u
e+
foton e− e− u
u

Figura 1: (A) Situação anterior à colisão, no referencial S. (B) Situação após a colisão, no referencial S.

Q5. Um gás ideal contido num recipiente, inicialmente com volume VA e pressão pA (estado A),
sofre expansão isobárica até atingir o volume VB (estado B). O gás sofre então uma expansão
adiabática, até que sua pressão seja pC (estado C), de forma que uma contração isobárica (até
o estado D) seguida de uma compressão adiabática levem o gás novamente à situação inicial
(estado A). Considere dada a razão γ entre os calores especı́ficos do gás a pressão constante e
a volume constante.

a) Represente as transformações descritas acima em um diagrama p−V , indicando os estados


A, B, C e D.
(b) Calcule o calor trocado em cada trecho do ciclo, em termos de pA , VA , VB , pC e γ.
(c) Determine a eficiência do ciclo, isto é, a razão entre o trabalho realizado pelo gás e o calor
absorvido por ele.

2
Q6. É possı́vel construir armadilhas capazes de confinar ı́ons de
massa m e carga q. Em particular, a armadilha pode res-
tringir o movimento dos ı́ons a apenas uma dada direção x0 x0
espacial, x. Assim, considere dois ı́ons de cálcio uma vez
ionizado (Ca+ ), submetidos a um potencial confinante ex-
terno harmônico U (x) = mω 2 x2 /2. Esses ı́ons interagem +e 0 +e
!x1 !x2
adicionalmente através da repulsão coulombiana,

e2
FC =
(x1 − x2 )2
onde x1 e x2 são as posições dos ı́ons de cálcio e, por simplicidade, foi definido: e2 = q 2 /(4πε0 ).
A figura acima define um sistema de coordenadas conveniente e representa os ı́ons na posição
de equilı́brio em que −x1 = x2 = x0 . O objetivo deste problema é estudar os modos normais
dessa cadeia unidimensional constituida pelos dois ı́ons de cálcio.

a) Obtenha a posição de equilibrio x0 em termos de e, m e ω.


b) Escreva as equações de Newton para o movimento de cada ı́on e obtenha a frequência de
oscilação do sistema quando a separação entre os ı́ons for constante. Este é o primeiro
modo normal de oscilação dessa cadeia.
c) O segundo modo normal corresponde a um movimento antissimétrico dos ı́ons, em cujo
caso o centro de massa está parado em x = 0. Obtenha esse segundo modo normal
no limite de pequenas oscilações. Obtenha a razão entre as frequências dos dois modos
normais de oscilação do sistema.
d) As figuras a) e b) abaixo representam os modos normais de oscilação desse sistema de
dois ı́ons. Identifique o primeiro e o segundo modo normal obtidos, respectivamente, nos
itens b e c acima. Qual deles tem menor energia?

Q7. Um satélite artificial de massa m está em órbita elı́ptica em torno da Terra. Admita que a
Terra seja uma esfera de densidade uniforme com raio R e massa M , e denote por G a constante
de gravitação universal. Considere conhecidos d e D, as distâncias entre o centro da Terra e o
satélite nos pontos de menor e maior afastamento, respectivamente. Uma partı́cula de massa
m0 menor do que m, choca-se centralmente e de forma completamente inelástica com o satélite
no ponto de menor afastamento da Terra. No instante da colisão, o satélite e a partı́cula tinham
velocidades iguais em módulo, mas com sentidos opostos.

a) Obtenha a velocidade vS do sistema satélite-partı́cula imediatamente após a colisão em


termos de vp , a velocidade no ponto de menor afastamento.
b) Expresse o momento angular do satélite nos pontos de mı́nimo e máximo afastamento
em termos de vp e de va (a velocidade no ponto de maior afastamento), respectivamente,
antes da colisão.
c) Obtenha a velocidade vp , antes da colisão, em termos de M , d, D e G.

1
d) Obtenha a energia ES e o momento angular LS do sistema satélite-partı́cula, depois da
colisão, em termos de m0 e das grandezas que caracterizam o movimento do satélite antes
da colisão.

Q8. Seja o estado do spin de um elétron dado por


√ !
2
|ψi = α |z+ i − |z− i
2
   
1 0
|z+ i = , |z− i = .
0 1
Lembrando que os operadores de spin Ŝx , Ŝy , Ŝz podem ser escritos em termos das matrizes
de Pauli como Ŝ = ~ ~σ /2 (veja formulário), onde
~ ~
Ŝx |x+ i = + |x+ i, Ŝx |x− i = − |x− i,
2 2
~ ~
Ŝy |y+ i = + |y+ i, Ŝy |y− i = − |y− i,
2 2
~ ~
Ŝz |z+ i = + |z+ i, Ŝz |z− i = − |z− i,
2 2
a) Qual é o valor de α ∈ R para que |ψi fique normalizado?
b) Qual é a probabilidade de se medir −~/2 para o spin na direção z?
c) Qual é a probabilidade de se medir +~/2 para o spin na direção x?
d) Qual é o valor esperado do spin no plano y = 0 em uma direção de 450 entre os eixos x e
z?

Q9. Seja o operador  associado a um certo observável fı́sico A de um sistema satisfazendo


[Â,Ĥ] 6= 0, onde Ĥ é um operador hamiltoniano independente do tempo. Sejam agora os
autovetores normalizados, φ+ , φ− , e autovalores correspondentes, a+ , a− (a+ 6= a− ) de Â:
Âφ+ = a+ φ+ , Âφ− = a− φ− ,
com
u+ + u− u+ − u−
φ+ = √ , φ− = √
2 2
onde
Ĥu+ = E+ u+ , Ĥu− = E− u−

a) Calcule o valor esperado de  no estado φ+ .


b) Calcule a projeção de Ĥu+ no estado u− .
c) Admitindo que o sistema esteja inicialmente em um estado arbitrário, ψ(0) escreva quanto
valerá o estado ψ(t) em um instante posterior como função de Ĥ.
d) Calcule o valor esperado do observável A no instante t = ~π/[3(E+ − E− )] admitindo que
o sistema esteja inicialmente no estado ψ(0) = φ+ e E+ 6= E− .

Q10. Considere N osciladores harmônicos tridimensionais clássicos não-interagentes, de massa m e


frequência angular ω, em contato com um reservatório térmico à temperatura T .

a) Escreva a hamiltoniana do sistema e obtenha a função de partição canônica.


b) Obtenha o valor médio da energia por oscilador. Qual a capacidade térmica do sistema?

2
Q1. Um capacitor esférico é composto por uma esfera condutora de raio R1 , concêntrica com uma
casca condutora esférica de raio R2 e espessura desprezı́vel, com R1 < R2 . O condutor interno
possui carga +Q e o externo possui carga −Q.

(a) Calcule o campo elétrico e a densidade de energia em função de r, onde r é a distância


radial a partir do centro dos condutores, para qualquer r.
(b) Determine a capacitância C do capacitor.
(c) Calcule a energia do campo elétrico armazenada em uma casca esférica de raio r, espessura
dr e volume 4πr2 dr, localizada entre os condutores. Integre a expressão obtida para
encontrar a energia total armazenada entre os condutores. Dê sua resposta em termos da
carga Q e da capacitância C.

Q2. Duas bobinas idênticas, compostas cada uma por um anel de raio R e espessura desprezı́vel,
são montadas com seus eixos coincidentes com o eixo-z, conforme se vê na figura abaixo. Seus
centros estão separados por uma distância d, com o ponto médio P coincidindo com a origem
do eixo-z. Cada bobina transporta uma corrente elétrica total de intensidade I. Ambas as
correntes têm o mesmo sentido anti-horário.

(a) Utilize a lei de Biot-Savart para determinar o campo magnético de uma única bobina ao
longo de seu eixo de simetria.
(b) A partir do resultado anterior, obtenha o campo magnético B(z) ao longo do eixo-z das
duas bobinas.
(c) Admitindo que o espaçamento d seja igual ao raio R das bobinas, mostre que, no ponto
P, as seguintes igualdades são válidas: dB/dz = 0 e d2 B/dz 2 = 0.
(d) Considerando os gráficos abaixo, de B (em unidades arbitrárias) versus z, qual curva
descreve o campo magnético ao longo do eixo-z na configuração do item (b)? Justifique.
(e) Supondo que a corrente na bobina superior tenha seu sentido invertido, calcule o novo
valor do campo magnético no ponto P.

z
I
3
R B
2

P d
I 0
1
R

−1.5 0 1.5
z /d

Q3. A lei de radiação de Planck permite obter a seguinte densidade de energia do espectro de corpo
negro de uma cavidade à temperatura T :

8πν 2 dν
ρ(ν)dν = 3 hν/kT
.
c e −1

1
(a) Expresse a densidade de energia em função do comprimento de onda λ = c/ν no lugar da
frequência ν.
(b) Mostre que para comprimentos de onda longos e altas temperaturas, o resultado anterior
se reduz à lei clássica de Rayleigh-Jeans.
(c) Obtenha a lei de Stefan-Boltzmann a partir da lei de radiação de Planck. Note que a
radiância R(λ), que é o fluxo de energia por unidade de área em uma pequena abertura
da cavidade, é dada por R(λ) = cρ(λ)/4.

Q4. Considere uma colisão relativı́stica frontal completamente inelástica de duas partı́culas que se
movem ao longo do eixo-x. Ambas as partı́culas possuem massa m. Antes da colisão, um
observador A, em um referencial inercial, nota que elas se movem com velocidades constantes
de mesma magnitude mas em direção opostas, isto é, a partı́cula 1 se move com velocidade v
e a partı́cula 2 se move com velocidade −v. De acordo com outro observador B, entretanto, a
partı́cula 1 está em repouso antes da colisão.

(a) Determine a velocidade vx′ da partı́cula 2 medida pelo observador B antes da colisão.
(b) Ache as velocidades vA e vB′ da partı́cula resultante após a colisão, medidas, respectiva-
mente, pelos observadores A e B.
(c) Utilize a conservação relativı́stica massa-energia para calcular a massa M da partı́cula
resultante após a colisão.

Q5. A pressão p de um gás se comporta, como função da temperatura T e do volume molar v, de


acordo com a seguinte equação de estado
RT a
p= − 2,
v v
onde a é uma constante positiva e R é a constante universal dos gases.

(a) Utilize a identidade    


∂u ∂p
=T −p
∂v T ∂T v
para determinar a energia molar u como função de v.
(b) Admitindo que cv = (∂u/∂T )v seja constante e igual a c, ache u como função de T e v.
(c) Numa expansão livre do gás, a temperatura cresce ou decresce? Leve em conta que numa
expansão livre u permanece invariante e v cresce.
(d) Demonstre a identidade do item (a).

2
Q6. Um pêndulo simples é constituı́do por uma partı́cula de
massa m suspensa por um fio inextensı́vel de comprimento
a e massa desprezı́vel. Seu ponto de suspensão é conectado x
a um suporte que se movimenta horizontalmente sem atrito
como mostrado na figura. Suponha que o suporte seja muito
pequeno e que o pêndulo se movimente apenas no plano ver-
tical. Usando como coordenadas generalizadas x e θ, onde g θ a
x é a posição horizontal do suporte e θ o deslocamento an-
gular do pêndulo, conforme se vê na figura, o movimento do m
sistema é descrito pela lagrangiana:
m 2 m 2 2
L= ẋ + (a θ̇ + 2aẋθ̇ cos θ) + mga cos θ.
2 2
(a) Obtenha a equação de movimento para a coordenada θ.
(b) Admitindo que os deslocamentos angulares sejam pequenos e que o suporte esteja sujeito
a um movimento harmônico forçado de frequência ω, isto é, descrito por x(t) = x0 cos ωt,
obtenha a solução geral θ(t) da equação do movimento para a coordenada θ.
(c) No caso do item anterior, obtenha a frequência de ressonância ωR .
(d) Escreva a solução geral para θ(t), quando as condições iniciais forem θ(0) = 0 e θ̇(0) = 0
e o suporte movimentar-se com frequência ω < ωR .

Q7. Um átomo de trı́tio pode ser descrito classicamente como um núcleo com carga elétrica +e,
composto por um próton e dois nêutrons, circundado por um elétron orbital de carga −e, o
qual percorre uma órbita circular de raio r0 . Em um processo conhecido como decaimento
beta, o núcleo de trı́tio se transforma em um núcleo de hélio, composto por dois prótons e um
nêutron, emitindo um par de partı́culas que rapidamente escapa do sistema atômico. Como
consequência desse processo, o átomo de hélio fica ionizado uma vez, e o elétron orbital passa
subitamente para uma nova situação, orbitando agora em torno de um núcleo de carga +2e.

(a) Supondo que o par de partı́culas que escapa do átomo tenha momento linear total de
módulo p, obtenha a velocidade de recuo do átomo de hélio de massa M .
(b) Obtenha a energia Ea do elétron orbital antes do decaimento beta.
(c) Calcule a energia Ed do elétron orbital depois do decaimento beta e obtenha a razão
ρ = Ea /Ed .
(d) Determine o momento angular total do elétron em função de r0 e da massa m do elétron.
Calcule a maior e a menor distância entre o elétron e o núcleo na nova órbita em termos
de r0 .

Q8. Considere os dois estados |1i e |2i da molécula de amônia N


ilustrados ao lado. Suponha que eles estão ortonormalizados,
H
hi|ji = δij e que apenas esses dois estados sejam acessı́veis H
ao sistema, de forma que podemos descrevê-lo usando a base H |1>
formada por |1i e |2i. Nessa base, o hamiltoniano H do
sistema é dado por H
H
 
E0 −E1 H |2>
H=
−E1 E0 N

1
(a) Se inicialmente o sistema estiver no estado |1i, ele permanecerá no estado |1i em um
instante posterior? E se estiver no estado |2i, ele permanecerá no estado |2i?
(b) Ache os autovalores EI e EII e os respectivos autovetores |Ii e |IIi de H, expressando-os
em termos de |1i e |2i.
(c) Baseado no resultado acima, podemos prever pelo menos uma frequência de emissão de
radiação eletromagnética possı́vel para uma molécula de amônia. Qual é essa frequência?
(d) Ache a probabilidade de medirmos uma energia EI no seguinte estado
1 2
|ψi = √ |1i − √ |2i.
5 5

Q9. Uma partı́cula quântica de massa m está sujeita a um potencial


1
V = mω 2 (x2 + y 2 + z 2 ).
2
(a) Obtenha os nı́veis de energia dessa partı́cula. Isto é, determine os autovalores de

~2 2
− ∇ ψ + V ψ = Eψ.
2m

(b) Considere o estado fundamental e os dois primeiros nı́veis excitados. Monte uma tabela
mostrando para cada um desses três nı́veis, o valor da energia, a degenerescência e os
respectivos estados em termos dos números quânticos.
(c) Utilizando
L2
   
2 1 ∂ 2 ∂ψ
∇ψ= 2 r − 2 2ψ
r ∂r ∂r ~r
e lembrado que L2 Yℓm = ~2 ℓ(ℓ + 1)Yℓm , escreva a equação diferencial do item (a) para
a parte radial da função de onda (não é preciso resolvê-la). Identifique nessa equação o
potencial efetivo Vef (r).
(d) Resolva a equação diferencial do item anterior para o caso em que ℓ = 0 e determine o
2
autovalor correspondente. Para isso, admita uma solução do tipo e−αr e determine α.

Q10. Considere um gás monoatômico clássico constituı́do por N átomos não interagentes de massa
m confinados num recipiente de volume V , à temperatura T . A hamiltoniana corespondente a
um átomo é dada por H = (p2x + p2y + p2z )/2m.

(a) Mostre que a função de partição canônica atômica é ζ = V /λ3 , onde λ = h/ 2πmkB T é
o comprimento de onda térmico de de Broglie.
(b) Utilizando ζ do item anterior, obtenha a função de partição Z do sistema e a energia
livre de Helmholtz F . Obtenha, também, a energia livre por átomo f = F/N no limite
termodinâmico N → ∞, V → ∞, v = V /N fixo.
(c) Obtenha a energia interna U e a pressão p do gás.
(d) Calcule a entropia por átomo no limite termodinâmico.

2
Q1. Considere um condutor macroscópico de forma arbitrária, cuja superfı́cie é fechada e suave.
Partindo da lei de Gauss e considerando que o rotacional do campo eletrostático é nulo:

(a) Calcule o campo elétrico no interior do condutor;


(b) Obtenha a componente normal do campo elétrico na superfı́cie externa do condutor em
termos da densidade superficial de carga;
(c) Obtenha a componente tangencial do campo elétrico na superfı́cie do condutor.

Q2. Considere um conjunto de soluções de ondas planas eletromagnéticas no vácuo, cujos campos
~ i(~k·~x−ωt) , onde ~k é o
(elétrico e magnético) são descritos pela parte real de funções: ~u(~x,t) = Ae
vetor de onda, que determina a direção de propagação da onda, e ω é a frequência angular, que

se relaciona com o vetor de onda por ω = v|~k|, onde v = 1/ ǫµ é a velocidade de propagação
das ondas.

~ · ~u = i~k · ~u;
(a) Mostre que o divergente de ~u(~x,t) satisfaz: ∇
~ × ~u = i~k × ~u;
(b) Mostre que o rotacional de ~u(~x,t) satisfaz: ∇
~ B
(c) Demonstre que as ondas são transversais e que os vetores E, ~ e ~k são mutuamente
perpendiculares.

Q3. Em 1913, Niels Bohr introduziu seu modelo atômico através da adaptação do modelo de Ruther-
ford às ideias de quantização propostas na época. Em homenagem a esse evento, aborde os
itens abaixo em termos de grandezas fundamentais.

(a) Use a regra de quantização para o momento angular, L = ~n, para encontrar uma ex-
pressão para os raios das órbitas permitidas de um elétron ao redor de um átomo de
número atômico Z .
(b) Segundo o modelo de Bohr, a transição entre diferentes órbitas é acompanhada pela
emissão/absorção de um fóton. Determine a energia do fóton emitido como resultado
da transição entre o primeiro estado excitado e o estado fundamental de um átomo de
hidrogênio.
(c) Considere um elétron preso em um poço unidimensional quadrado infinito de largura
a. Determine uma expressão paraH os nı́veis de energia eletrônicos usando a regra de
quantização de Bohr-Sommerfeld pdx = hn.
(d) Determine a largura a desse poço, em termos do raio de Bohr, para que a energia de um
fóton emitido devido à transição entre o primeiro estado excitado e o estado fundamental
seja igual àquela obtida no item (b).

Q4. Os raios-γ produzidos por aniquilação de pares apresentam um espalhamento Compton con-
siderável. Considere que um fóton com energia m0 c2 seja produzido pela aniquilação de um
elétron e um pósitron, onde m0 é a massa de repouso do elétron e c é a velocidade da luz.
Suponha que esse fóton seja espalhado por um elétron livre e que o ângulo de espalhamento
seja θ.

(a) Encontre a máxima energia cinética possı́vel do elétron em recuo nesse espalhamento.
(b) Se o ângulo de espalhamento for θ = 120◦ , determine a energia do fóton e a energia
cinética do elétron após o espalhamento.

1
(c) Se θ = 120◦ , qual é a direção de movimento do elétron após o espalhamento, em relação
à direção do fóton incidente?

Q5. Um mol de um gás ideal simples está contido em um recipiente de volume inicial v0 e pressão
p0 . O gás ideal é descrito pelas equações pv = RT e u = cRT , onde p é a pressão, v é o volume
molar, T a temperatura absoluta, u é a energia molar; R e c são constantes. O gás se expande
a partir desse estado inicial até o estado correspondente a um volume final 2v0 através de um
dado processo. Determine o trabalho W realizado pelo gás e o calor Q recebido pelo gás para
cada um dos processos listados abaixo. As respostas finais devem ser dadas apenas em termos
de (v0 ,p0 ) e de c.

(a) Expansão livre. Determine também a variação de temperatura ∆T .


(b) Expansão quase-estática isentrópica. Obtenha também a pressão final pf , utilizando o
fato de que, nesse processo para um gás ideal, pv γ = constante, onde γ = (c + 1)/c.
(c) Expansão quase-estática isobárica.
(d) Expansão quase-estática isotérmica.

2
Q6. Duas partı́culas, A e B, de massas m e M (m 6= M ), respectivamente, estão conectadas às
extremidades de um fio inextensı́vel de comprimento ℓ e de massa massa desprezı́vel que passa
por um orifı́cio em uma mesa horizontal, como mostrado na figura abaixo. A partı́cula A
move-se sem atrito sobre a mesa enquanto a outra o faz verticalmente sob a ação conjunta da
gravidade, de aceleração ~g , e da tração do fio (desconsidere também o atrito entre o fio e o
orifı́cio).

(a) Supondo que a posição inicial de A seja r = r0 , que velocidade inicial deve ser conferida
a ela para que B permaneça em repouso abaixo da superfı́cie da mesa?
(b) Obtenha as equações do movimento, admitindo que a lagrangiana que descreve um mo-
vimento arbitrário desse sistema é dada por
1 1
L = (m + M )ṙ2 + mr2 θ̇2 − M g(r − ℓ).
2 2
(c) Obtenha as grandezas conservadas e dê o significado de cada uma delas.
(d) Se B for ligeiramente e verticalmente deslocada da sua posição, ocorrerão pequenas os-
cilações no sistema. Obtenha o perı́odo dessas oscilações em termos do raio de equilı́brio
req e das demais grandezas que caracterizam o sistema (m, M e g).

(Q6)
r
θ A
0.5 V/ka 2 (Q7)

g −2 −1 1 2 x/a
B
−0.5

Q7. Uma partı́cula de massa m está sujeita ao potencial unidimensional


1 k
V (x) = kx2 − 2 x4 ,
2 4a
mostrado na figura acima, onde k e a são constantes positivas.

(a) Determine a força F (x) e obtenha os pontos de equilı́brio, determinando sua natureza
(b) Calcule o perı́odo das pequenas oscilações que ocorrem em torno do ponto de equı́librio
estável.
(c) Admita que a partı́cula esteja em repouso no ponto x = 0 e que receba um impulso que lhe
confere, instantaneamente, uma velocidade dep módulo v na direção
p de x positivo. Discuta
o que ocorre nos seguintes casos: 0 < v ≤ a k/2m e v > a k/2m.
(d) Esboce o diagrama de fase do sistema (ẋ versus x para energia constante) para os di-
versos tipos de movimento. Indique claramente a curva que corresponde à transição de
movimento periódico para não periódico, bem como o valor da energia correspondente.

1
Q8. Considere o problema de uma partı́cula de massa m cujo movimento ao longo do eixo-x está
restrito ao intervalo 0 ≤ x ≤ a, isto é, ela encontra-se confinada em uma caixa com paredes
colocadas nas posições x = 0 e x = a.

(a) Determine a função de onda e a energia do estado fundamental.


(b) Suponha que a partı́cula seja descrita pela seguinte função de onda:
 
πx 2πx
ψ(x) = A sin − 3i sin ,
a a
onde A é uma constante de normalização. Determine A e calcule a probabilidade de obter
o resultado 2π 2 ~2 /ma2 para a medida da energia.
(c) Suponha agora que a partı́cula esteja no estado fundamental. Qual é a distribuição de
probabilidades do momento da partı́cula nesse estado?
(d) Considerando novamente que a partı́cula esteja no estado fundamental, suponha que as
paredes sejam removidas, de forma instantânea, deixando a partı́cula livre (Ĥ = p̂2 /2m).
Qual é a energia dessa partı́cula livre?
~ = γ S,
Q9. Considere uma partı́cula de spin 1/2, cujo momento magnético é M ~ onde γ é uma
constante. Podemos descrever o estado quântico dessa partı́cula utilizando o espaço gerado
pelos autovetores |+i e |−i do operador Ŝz , que mede a projeção do spin na direção z,
~ ~
Ŝz |+i = |+i, Ŝz |−i = − |−i.
2 2
~ = B ŷ, orientado ao longo
A partı́cula encontra-se sujeita a um campo magnético uniforme B
do eixo y, de forma que que o hamiltoniano é dado por
~ ·B
Ĥ = −M ~ = −γB Ŝy .
Inicialmente, no instante t = 0, ela está no estado |ψ(0)i = |+i.

(a) Quais são as possı́veis valores da projeção do spin no eixo-y?


(b) Encontre os autovetores de Ŝy .
(c) Obtenha |ψ(t)i para t > 0 em termos de |+i e |−i definidos acima.
(d) Obtenha os valores médios dos observáveis Sx , Sy e Sz em função do tempo.
Q10. Um determinado material magnético é composto por N átomos
magnéticos não-interagentes, cujos momentos magnéticos µ po- y
dem apontar em três direções possı́veis, conforme mostra a fi-
gura ao lado: µ0 = µŷ, µ1 = µx̂ e µ2 = −µx̂. O sistema µ0 H
encontra-se em equilı́brio térmico a temperatura T e na presença
de um campo magnético uniforme orientado ao longo da direção µ2 µ1 x
y, H = H ŷ, de modo que os nı́veis de energia correspondentes
a um único átomo são ǫ0 = −µH, ǫ1 = 0 e ǫ2 = 0.
(a) Obtenha a função de partição canônica z de um átomo, a função de partição canônica Z
do sistema e a energia livre de Helmholtz f por átomo.
(b) Determine a energia média u = hǫn i e a entropia s por átomo.
(c) Obtenha a magnetização por átomo m = mx x̂ + my ŷ = hµn i.
(d) Verifique que a susceptibilidade isotérmica χT = (∂my /∂H)T a campo nulo obedece à lei
de Curie, χT ∝ 1/T .

2
Q1. Considere um fio infinitamente longo disposto paralelamente ao eixo z, interceptando o plano
z = 0 em x = a e y = 0, conforme mostra a figura. O fio está carregado com densidade linear
de carga elétrica λ uniforme.

a
x y

(a) Determine o potencial elétrico V (x,y,z) em todo o espaço, de forma que o potencial seja
zero no eixo z. Sugestão: pode-se calcular o potencial a partir do campo elétrico do fio
longo, que é obtido de forma simples usando a lei de Gauss.
(b) Considere agora, além do fio, um condutor plano infinito (aterrado) ocupando o plano
x = 0. Calcule V (x,y,z) para a região x > 0 do espaço. Sugestão: utilize o método das
imagens.
(c) Qual a densidade superficial de carga σ(y,z) induzida no condutor plano em x = 0?
∫∞
(d) Calcule a integral −∞ σ(y,z) dy e discuta o resultado obtido.

Q2. Um fio carregado com densidade linear de carga elétrica λ > 0 está colado (formando um anel)
na borda de um disco isolante de raio a, que pode girar ao redor de seu eixo vertical sem atrito.
O comprimento do fio é exatamente 2πa. Apenas na região central do disco, até um raio b < a,
age um campo magnético uniforme B0 vertical para cima.

z
B0

b
a

(a) O campo magnético é agora desligado. Obtenha a expressão para o torque devido à força
eletromotriz induzida no fio, em termos da variação temporal do campo magnético, dB/dt.
A partir deste resultado, calcule o momento angular final do disco (módulo e direção).
(b) Considerando como dado o momento de inércia I do sistema disco+fio, calcule o campo
magnético (módulo e direção) produzido no centro do disco pelo anel de carga na situação
final acima.

1
Q3. Um feixe de luz com comprimento de onda 480 nm no vácuo e de intensidade 10 W/m2 incide
sobre um catodo de 1 cm2 de área no interior de uma célula fotoelétrica. A função trabalho
do metal é 2,2 eV. As respostas devem ser dadas com dois algarismos significativos.

(a) Calcule a energia dos fótons incidentes em Joules e em elétron-volts.


(b) Calcule o número de fótons por segundo incidentes na placa metálica.
(c) Se a eficiência da conversão fotoelétrica é de 20% (apenas 20% dos fótons arrancam
elétrons do metal), calcule a corrente elétrica máxima, através da célula, quando uma ddp
é aplicada entre o catodo e o anodo.
(d) Calcule o comprimento de onda máximo dos fótons incidentes acima do qual não ocorre
o efeito fotoelétrico.

Q4. Uma partı́cula de massa m executa oscilações harmônicas, em uma dimensão, num potencial
U (x) = mω 2 x2 /2. Considere a partı́cula num estado cuja função de onda é ψ(x) = Ae−bx ,
2

onde A e b são constantes.

(a) Escreva a equação de Schrödinger independente do tempo para este potencial.


(b) Determine o valor de b para que ψ(x) seja solução desta equação de Schrödinger, e o valor
da energia associada a esta função de onda.
(c) Calcule a constante de normalização A.
(d) Classicamente, esta partı́cula oscilaria dentro do intervalo simétrico [−xmax ,xmax ], onde
xmax = [~/mω]1/2 . Calcule, usando a Mecânica Quântica, a probabilidade de se encontrar
esta partı́cula no intervalo [−xmax ,xmax ]. Compare este resultado com o esperado pela
Mecânica Clássica.

Q5. Um cilindro de paredes externas impermeáveis, rı́gidas e adiabáticas, fechado em ambas as


extremidades, é munido de uma parede de separação interna impermeável, móvel, adiabática e
ideal (sem fricção), que o divide em dois compartimentos (A e B). Cada um deles é preenchido
com um mol de um gás ideal monoatômico. Inicialmente a pressão, o volume e a temperatura
(P0 ,V0 ,T0 ) são idênticos em ambos os lados da parede interna. Uma certa quantidade de calor
é introduzida de forma quase-estática no compartimento A até que sua pressão atinja o valor
PA = 32P0 .

(a) A partir das equações de estado do gás ideal monoatômico U = 32 N RT = 23 P V e de sua


entropia S/N = 32 R ln T + R ln V + constante, demonstre que, ao longo de um processo
isentrópico em um sistema fechado, P 3 V 5 = constante.
(b) Obtenha os volumes finais VA e VB dos dois compartimentos em termos do volume inicial
V0 .
(c) Obtenha as temperaturas finais TA e TB dos dois compartimentos em termos da tempe-
ratura inicial T0 , verificando que TA = 15TB .
(d) Obtenha as variações de entropia do gás nos dois compartimentos, ∆SA e ∆SB . Qual é o
sinal da variação da entropia total do sistema?

2
Q6. Uma partı́cula de massa m move-se com velocidade ⃗v1 no semi-plano superior até ser desviada
ao atingir o semi-plano inferior, onde passa a se propagar com velocidade ⃗v2 , conforme ilustrado
na figura abaixo. Observa-se experimentalmente as seguintes caracterı́sticas: i) a partı́cula
passa do meio 1 ao meio 2 desde que v1 > vmin ; ii) a partı́cula se move de modo retilı́neo
e uniforme em cada um dos semi-planos; iii) o ângulo de saı́da θ2 é diferente do ângulo de
entrada θ1 , o que nos faz presumir que em cada meio a partı́cula esteja sob ação de diferentes
potenciais U1 e U2 .

(a) Com base no experimento, esboce o gráfico do potencial U em função de y para x fixo
(justificando o gráfico).
(b) Determine v2 em termos de v1 , de m e dos potenciais U1 e U2 . Qual é a velocidade vmin
acima da qual observa-se a passagem da partı́cula do meio 1 para o meio 2?
(c) Determine o ı́ndice de refração sen θ1 /sen θ2 em termos de m, v1 e dos potenciais em cada
meio.

Q7. Uma partı́cula de massa m desenvolve movimento unidimensional sob ação do potencial abaixo
(c é uma constante)
1
U (x) = x4 − cx2 .
2
(a) Esboce os gráficos de U (x) e dos respectivos espaços de fase (ẋ versus x para todas as
energias possı́veis) nos seguintes casos : i) c > 0, ii) c = 0 e iii) c < 0.
(b) Por meio da energia total E, identifique todos os movimentos periódicos possı́veis e seus
respectivos pontos de inversão (onde a velocidade é nula) para cada um dos casos do item
(a).
(c) Determine a dependência do perı́odo de oscilações com a energia total E para c = 0.

Q8. Uma partı́cula de massa m está num potencial tal que a equação de Schrödinger (com ~ = 1)
no espaço dos momentos é
( )
p⃗ 2 ∂
− a∇p ψ̄(⃗p,t) = i ψ̄(⃗p,t)
2
2m ∂t

onde
∂2 ∂2 ∂2
∇2p = + + .
∂p2x ∂p2y ∂p2z

(a) Escreva a equação de Schrödinger no espaço das coordenadas.


(b) Qual é o potencial V (r), r = |⃗r|?
(c) Qual é a força, F⃗ (⃗r), sobre a partı́cula?

1
Q9. Os operadores de spin de uma partı́cula de spin-1 (um tripleto) podem ser representados no
espaço complexo C 3 pelas matrizes
     
0 1 0 0 −i 0 1 0 0
~ ~
Ŝx = √  1 0 1  , Ŝy = √  i 0 −i  , Ŝz = ~  0 0 0  .
2 0 1 0 2 0 i 0 0 0 −1

(a) Mostre que as relações de comutação [Ŝx ,Ŝy ] = i~Ŝz , e permutações cı́clicas em x,y,z, são
satisfeitas.
(b) Se uma medida da componente z do spin é feita, quais são os possı́veis resultados? En-
contre os respectivos autovetores.
(c) Se o estado da partı́cula é dado pelo vetor
 
1
|ϕ⟩ =  i  ,
−2

quais são as probabilidades de se obter cada um dos resultados possı́veis das medidas do
spin ao longo do eixo-z?
(d) A partir do resultado do item c), qual é a probabilidade de se encontrar a partı́cula em
qualquer um desses estados?

Q10. Considere um oscilador harmônico unidimensional modificado, definido pela função hamilto-
niana
p2
H = + V (x),
2m
onde V (x) = 12 mω 2 x2 para x ≥ 0, V (x) = ∞ para x < 0. Ele encontra-se em equilı́brio térmico
com um reservatório de calor a temperatura T .

(a) Justifique, em termos da paridade das autofunções do problema quântico, por que, devido
às condições impostas, apenas os valores inteiros ı́mpares de n são permitidos para as
autoenergias deste oscilador, ϵn = (n + 1/2)~ω.
(b) Para a versão quântica, obtenha a função de partição canônica z deste oscilador e a energia
livre de Helmholtz associada f .
(c) Obtenha a energia interna média deste oscilador a partir de u = −∂ ln z/∂β.
(d) A partir da definição da energia interna média no ensemble canônico, u ≡ ⟨ϵn ⟩, demonstre
a expressão u = −∂ ln z/∂β.
(e) Mostre que a função de partição canônica clássica deste oscilador é dada por zclass =
(2β~ω)−1 . Determine a energia interna média clássica associada, uclass ≡ ⟨H ⟩class .

2
Q1. Considere uma esfera sólida, uniformemente carregada, de carga Q e raio R.
(a) Determine o vetor campo elétrico E ~ em um ponto à distância r do centro da esfera, nos
casos r > R e r ≤ R.
(b) Obtenha a força dF~ sobre um elemento de volume dV da esfera, localizado na posição ~r.
(c) Determine agora, por integração, a força total F~ que age sobre o hemisfério superior da
esfera.

Q2. Um capacitor de placas planas paralelas é formado por dois discos circulares de raio a, separados
entre si de uma distância d  a, no vácuo. As placas estão ligadas a um gerador de corrente
alternada de frequência ω, que produz uma carga uniforme na placa do capacitor, dada por
q(t) = q0 sin(ωt). São desprezados efeitos de borda. Supondo baixas frequências, de forma que
√ ~ entre as placas pode
ωa/c  1 (onde c = 1/ µ0 0 é a velocidade da luz), o campo elétrico E
ser considerado uniforme. Considere um sistema de coordenadas cilı́ndricas, (r,θ,z), com eixo
z passando pelo centro das placas, conforme indicado na figura.

(a) ~ entre as placas.


Calcule a expressão para o campo elétrico E
(b) Calcule o campo magnético B,~ em função do raio r, na região entre as placas do capacitor.
(c) Calcule o vetor de Poynting S ~ = (E
~ × B)/µ
~ 0.
(d) Usando a aproximação de baixas frequências, mostre que é satisfeita a conservação de
~ + ∂u/∂t = 0, onde u = 1 (0 E
energia, expressa pela condição ∇ · S ~2 + B~ 2 /µ0 ) é a
2
densidade de energia contida no campo eletromagnético.

Q3. Uma partı́cula de massa m confinada em um poço de potencial unidimensional possui função
de onda dada por:


 0 para x < −L/2
3πx

ψ(x) = A cos para − L/2 < x < L/2

 L
0 para x > L/2

(a) Calcule a constante de normalização A.


(b) Calcule a probabilidade de encontrar a partı́cula no intervalo entre −L/4 < x < L/4 .
(c) Através da solução da equação de Schrödinger independente do tempo para esta partı́cula
no referido poço de potencial, ache a energia correspondente à função de onda, em termos
de m, L e h.
(d) Calcule o comprimento de onda do fóton emitido na transição desta partı́cula para o
estado fundamental, em termos de m, L e h.

1
Q4. O decaimento dos múons obedece à seguinte equação diferencial

dN (t)
= −RN (t)
dt
onde N (t) é o número de múons presentes no instante de tempo t e dN (t)/dt representa a taxa
de variação de múons no mesmo instante de tempo t. A constante de proporcionalidade R é
chamada de constante de decaimento. O tempo de vida médio do múon é t = 2µs, isto é, neste
intervalo de tempo N (t)/N (0) = 1/e ≈ 1/2,73. Sendo a velocidade dos múons na direção da
superfı́cie da Terra igual a 0,998c, responda:

(a) No sistema inercial de referência do múon, qual o valor de R para o decaimento de múons?
(b) Sem considerar correções relativı́sticas, estime quantos múons seriam detectados ao nı́vel
do mar, correspondentes a 108 múons detectados a 9 km de altitude.
(c) Considere agora a previsão relativı́stica e repita a estimativa do item (b).

Q5. Um gás ideal monoatômico de N moléculas de massa m está em equilı́brio térmico a uma
temperatura absoluta T . O gás está contido em uma caixa cúbica de aresta L, cujos lados de
baixo e de cima estão paralelos à superfı́cie da Terra. Considere o efeito do campo gravitacional
sobre as moléculas. A aceleração da gravidade é g. Determine:

(a) a função de partição de uma molécula do gás;


(b) a energia cinética média de uma molécula do gás;
(c) a energia potencial média de uma molécula do gás;
(d) a energia potencial média de uma molécula do gás no caso em que mgL/kB T  1. Faça
o cálculo até 2ª ordem da razão mgL/kB T .

2
Q6. Um equilibrista de massa m está inicialmente parado na extremidade de uma barra larga,
horizontal, homogênea, de comprimento D e massa M = 3m. A barra gira em torno de um
eixo vertical que passa pelo seu centro. O equilibrista começa então a caminhar sobre a barra,
em direção ao eixo de rotação, com velocidade constante. Considere o perı́odo inicial de rotação
do sistema igual a T0 .

(a) Determine o torque das forças que atuam sobre o equilibrista em relação ao centro da
barra.
(b) Determine o momento angular do sistema quando o equilibrista atinge o centro da barra.
Determine o perı́odo de rotação do sistema nessa situação.
(c) Determine as energias nas posições inicial e final do sistema. Nesse movimento, a energia
do sistema variou?

Considere o equilibrista como uma massa puntiforme.


1
Dado: ICM (barra) = M D2
12
Q7. Uma partı́cula de massa m se encontra no interior de um cano oco, liso, estreito e longo que
gira num plano horizontal com velocidade angular ω constante. O eixo fixo de rotação passa
por uma das extremidades do cano, como mostra a figura.

(a) Escreva a Lagrangiana da partı́cula.


(b) Obtenha as equações de Lagrange do movimento da partı́cula.
(c) Determine o movimento da partı́cula, considerando que inicialmente ela é lançada do
centro de rotação com velocidade ~v0 .
(d) Obtenha a função Hamiltoniana (H) do movimento dessa partı́cula e as equações de
Hamilton do movimento.
(e) Dentre as grandezas fı́sicas H e E (energia), quais são conservadas? Justifique sua res-
posta.

1
Q8. Considere um oscilador harmônico (em umaPdimensão, x) de massa m e frequência ω. No ins-
tante t = 0, o estado do oscilador é |ψ(0)i = n cn |φn i onde os |φn i são os estados estacionários,
isto é, H|φn i = En |φn i, sendo H a hamiltoniana e En = (n + 1/2)~ω com n = 0,1,2, . . .

(a) Considerando que os estados |φn i são normalizados, determine a condição que os coefi-
cientes cn devem satisfazer para que o estado |ψ(0)i seja também normalizado. Calcule,
então, a probabilidade P de que uma medida da energia do oscilador, feita num instante
t posterior, dê um resultado maior que 2~ω.
(b) Se apenas c0 e c1 são diferentes de zero, dê a expressão para o valor esperado da energia,
hHi, em termos de c0 e c1 . Com a condição hHi = ~ω, calcule |c0 |2 e |c1 |2 .
(c) O vetor de estado |ψ(0)i está definido a menos de um fator de fase global, o que nos
permite escolher c0 real e positivo. Fazendo isso, escrevendo
q c1 = |c1 |eiθ1 e mantendo a
1 ~
condição hHi = ~ω, determine θ1 de modo que hXi = 2 mω
.
Observação: Lembremos que o efeito do operador posição X sobre os estados esta-
cionários do oscilador harmônico é
r
~ h√ √ i
X|φn i = n + 1 |φn+1 i + n |φn−1 i
2mω
entendendo-se que o segundo termo acima é nulo para n = 0.
(d) Com |ψ(0)i determinado conforme o item anterior, escreva |ψ(t)i para t > 0 e calcule
hXi(t).

~ R
Q9. Sejam L, ~ e P~ os operadores do momento angular, da posição e do momento linear, respecti-
vamente.
P
(a) Usando as relações de comutação [Li ,Lj ] = i~ k ijk Lk , mostre que

~ ×L
L ~ = i~L
~

~ =R
(b) Com a definição L ~ × P~ e usando [Ri ,Pj ] = i~δij , mostre que
X
[Li ,Rj ] = i~ ijk Rk
k

~ e P~ são hermitianos, isto é, R† = Ri e P † = Pi , mostre que


(c) Sabendo que os operadores R i i
as componentes do operador do momento angular L ~ = R ~ × P~ também são operadores
hermitianos.
Observação: Nas expressões acima, ijk é o tensor completamente antissimétrico, isto é:

0
 se houver ı́ndices iguais;
ijk = +1 se ijk for 123, 231 ou 312;

−1 nos demais casos.

X
Se precisar, use a identidade ijk ijl = 2δkl .
i,j

2
Q10. A radiação em uma cavidade ressonante pode ser vista como um gás de fótons cuja pressão
sobre as paredes de uma cavidade de volume V é dada por

aT 4
P = ,
3
onde a é uma constante. A energia interna desse gás é dada pela equação U = aT 4 V . Considere
que inicialmente a temperatura da cavidade seja T0 e seu volume V0 .

(a) Determine o trabalho realizado em um processo isotérmico no qual o volume da cavidade


é duplicado. Forneça a resposta em termos de T0 , V0 e da constante a apenas.
(b) Determine o calor fornecido em um processo isotérmico no qual o volume da cavidade é
duplicado. Forneça a resposta em termos de T0 , V0 e da constante a apenas.
(c) Usando a relação     
∂U ∂U
dQ = dT + + P dV ,
∂T V ∂V T
determine a equação que descreve um processo adiabático em termos das variáveis V e T .
(d) Determine o trabalho realizado em um processo adiabático no qual o volume da cavidade
é duplicado. Expresse o resultado em termos de T0 , V0 e da constante a apenas.

3
Q1. Um cilindro de altura h e raio externo b é feito de um material com condutividade elétrica σ
e permissividade elétrica ε. O cilindro é furado ao longo de seu eixo de forma que seu raio
interno é a. Um material de alta condutividade elétrica preenche o furo central do cilindro e
forma também uma casca cilíndrica em torno da sua borda externa, formando os contatos
elétricos do cilindro, conforme ilustra a figura abaixo. Considere h >> b, de modo que os
efeitos de borda podem ser desprezados. Aplica-se uma diferença de potencial elétrico V0
entre esses contatos (tome V = 0 na superfície externa do cilindro).

∂ρ
(a) Mostre que, no regime estacionário ( = 0 ), a densidade de carga no interior do meio
∂t
condutor homogêneo é nula.
(b) Mostre que, nesse caso, o potencial elétrico obedece à equação de Laplace e obtenha o
G G
vetor campo elétrico E ( r ) no interior do cilindro.
(c) Calcule a carga livre total acumulada na superfície do contato interno (raio a) e a
capacitância entre os dois contatos elétricos.
(d) Calcule a resistência elétrica entre esses dois contatos elétricos.

b
a
V0

Q2. Um cilindro condutor muito longo de raio a conduz uma corrente I ao longo de seu eixo z. A
G
densidade de corrente J no interior do cilindro varia de acordo com a expressão abaixo:
G J
( )
J ( r, ϕ, z ) = ˆz 0 sen
r
πr
a
,

onde r é a distância radial entre o ponto considerado e o eixo do cilindro.

(a) Determine a constante J0 em termos de I e a.


G
(b) Calcule o campo magnético B fora do cilindro condutor (r > a) e expresse seu resultado
em termos de I e a.
G
(c) Calcule o campo magnético B no interior do cilindro condutor (r < a) e expresse seu
resultado em termos de I e a.
(d) Esboce um gráfico qualitativo do módulo do campo magnético, B(r), indicando seu
comportamento em r = 0 e r = a.

1
Q3. (a) Utilize a relação de de Broglie para o comprimento de onda associado a uma partícula e
obtenha a relação de quantização do momento angular de um elétron em movimento
orbital atômico, no modelo de Bohr (L = nћ, com n=1, 2, 3, ...).
(b) Use a expressão acima para mostrar que as energias associadas aos estados eletrônicos
permitidos em um átomo de hidrogênio são dadas por
mee 4
En = − ,
8ε02 h 2n 2

onde e e me são a carga e a massa do elétron, respectivamente.


(c) Calcule a energia de ionização do Lítio duplamente ionizado (Z = 3) sabendo que a
energia de ionização do hidrogênio é 13,6 eV.
(d) Em espectroscopia, a série de Balmer está associada a um subconjunto de transições nas
quais o elétron do átomo de H vai de um estado excitado ao estado final caracterizado
por nf = 2. Nesta série, a linha denominada por Hβ corresponde a transição a partir do
estado com ni = 4. Estime o comprimento de onda da linha Hβ e situe a mesma em
alguma região do espectro eletromagnético.

Q4. Em um experimento de efeito fotoelétrico, a Figura 1 abaixo mostra um possível gráfico da


corrente fotoelétrica em função da diferença de potencial V entre o coletor de elétrons e um
alvo de sódio. As curvas (a) e (b) correspondem a diferentes intensidades da luz incidente e
V0 é o chamado “potencial de corte” ou “potencial limite”. Já a Figura 2 mostra medidas do
potencial limite em função da frequência da luz incidente. Utilizando esses gráficos:

(a) estime o valor da constante de Planck em eVs, indicando o procedimento utilizado;


(b) estime o valor da “função trabalho” para o sódio;
(c) estime o valor da energia cinética do mais rápido fotoelétron emitido quando o alvo de
sódio é atingido por luz de frequência 1015 Hz;
(d) cite uma característica do efeito fotoelétrico que pode ser explicada classicamente e
outra que não se pode explicar utilizando a teoria ondulatória do eletromagnetismo.

Figura 1 Figura 2

Ib 3.0
(b)
Corrente detectada, I

2.5
Potencial Limite (V)

2.0
Ia 1.5
(a)
1.0
0.5

V0 - 0 + 0.0
0 1 2 3 4 5 6 7 8 9 10 11 12
Diferença de potencial aplicada, V
Frequência (1014Hz)

2
Q5. Dois corpos idênticos de capacidade térmica constante CP (finita) estão nas temperaturas T1
e T2, respectivamente, sendo T2 > T1. Considere que nos processos descritos abaixo os
corpos permanecem a pressão constante e não sofrem mudança de fase.

(a) Se os corpos forem colocados em contato, mas isolados termicamente do resto do


universo, determine a temperatura de equilíbrio.
(b) Determine a variação de entropia do sistema no processo descrito no item (a).

Considere agora que os corpos funcionem como fontes quente e fria para uma pequena
máquina térmica, a qual irá funcionar até que os dois corpos atinjam o equilíbrio térmico.

(c) Supondo que esse processo seja reversível, determine a temperatura final de equilíbrio
neste caso.
(d) Calcule a quantidade de trabalho produzida pela máquina térmica no processo descrito
no item (c).

3
Q6. Um corpo celeste de massa m se aproxima do Sol (massa M >> m) seguindo uma trajetória
hiperbólica e quando está a uma distância r0 dele, a sua velocidade é v0 e faz um ângulo de
30° com o raio vetor ao Sol.

(a) Calcule o momento angular L e a energia E desse corpo celeste.


(b) Determine a distância rp de máxima aproximação do corpo celeste ao Sol, expressando o
seu resultado em termos de L e E.
(c) Quando o corpo celeste atinge a distância rp de máxima aproximação, sofre um choque
com um pequeno asteróide de tal maneira que sua massa não varia, porém ele passa a
descrever órbita circular de raio rp no mesmo plano da órbita anterior. Calcule a nova
energia e o novo momento angular do corpo celeste após a colisão, expressando o seu
resultado em termos de rp.

Q7. Uma bola de massa m = 450 g está presa a uma mola cuja energia potencial em função da
elongação x está mostrada na figura abaixo (linha sólida). Expresse as respostas no SI.

(a) Determine a constante elástica da mola, para pequenos deslocamentos.


(b) Esboce um gráfico da força que atua sobre essa bola em função da elongação da mola.

Sabendo que o movimento da bola é unidimensional e sua elongação máxima é de 3 cm:

(c) determine sua velocidade máxima;


(d) determine a energia cinética da bola nesse movimento para a elongação da mola
x = − 2 cm;
(e) Determine a posição (x < 0) em que a bola deve ser solta a partir do repouso para
atingir o ponto x = 5 cm com velocidade nula.

18
16
14
12
10
U(mJ)

8
6
4
2
0
-2
-4 -2 0 2 4
x (cm)

1
Q8. Considere o problema unidimensional quântico de uma partícula de massa m sujeita ao
potencial


⎪ +∞ ,x <0


V (x ) = ⎨ 0 ,0 <x <a .


⎪ +∞
⎩ ,x >a

(a) Escreva a equação de Schrödinger independente do tempo para este problema.


(b) Resolva a equação, achando todas as soluções aceitáveis independentes. Isto é: determine
todos os valores possíveis para a energia, En , e as funções de onda normalizadas
correspondentes, ψn(x) .

Suponha agora que, na verdade, o potencial total tenha a forma Vtotal (x) = V(x) + W(x) ,
sendo W(x) uma pequena correção dada por


⎪ 0 ,x <0


W ( x ) = ⎨W0 sen ( πx a ) ,0 <x <a



⎩ 0 ,x >a

(c) Usando teoria de perturbações de primeira ordem, calcule a correção para a energia do
estado fundamental obtida no item anterior.

G = G
Q9. Para uma partícula de spin ½ o operador de spin é dado por S = σ , onde
2

⎛ 0 1 ⎞⎟ ⎛ 0 −i ⎞⎟ ⎛ 1 0 ⎞⎟
σx = ⎜⎜ σy = ⎜⎜ σz = ⎜⎜
⎝ 1 0 ⎠⎟⎟ ⎝ i 0 ⎟⎟⎠ ⎝ 0 −1 ⎟⎟⎠
, ,

são as matrizes de Pauli. Seja n̂ o vetor unitário na direção de ângulos (θ, φ), conforme
ilustra a figura abaixo.

θ n̂

y
φ
x
(a) Calculando o produto escalar, mostre explicitamente que o operador que representa a
G
componente do spin nessa direção, Sn = n
ˆ·S , é dado por

= ⎛ cos θ
⎜⎜ e −i φ sen θ ⎞⎟
Sn =
⎜⎝e iφ sen θ − cos θ ⎠⎟⎟
.
2

2
(b) Mostre que os únicos valores que podem ser obtidos numa medida de Sn são + = 2 e
− = 2 , qualquer que seja a direção n̂ .
(c) Obtenha o vetor coluna normalizado que representa o estado no qual uma medida de Sn
produz necessariamente o valor + = 2 . Simplifique a resposta final expressando a
dependência em θ em termos de sen(θ 2) e cos(θ 2) .
(d) Suponha, agora, que θ = 60° e φ = 45°. Se a partícula for preparada de tal forma que a
componente z do spin, Sz , tenha o valor bem definido + = 2 , qual é a probabilidade de
obter-se esse mesmo valor numa medida de Sn ? Dê a resposta numérica.

Q10. Considere um gás composto por N partículas ultrarrelativísticas (de forma que sua energia
ε possa ser escrita como ε = c p , onde p é o seu momento linear) confinado em um
recipiente de volume V e a temperatura T. Suponha que as partículas sejam indistinguíveis
e não interagentes, e que sua energia térmica seja suficientemente alta para desprezar
efeitos quânticos.
N
( 8πV )
(a) Mostre que a função de partição do gás é Z = , onde h é a constante
N ! ( hc kBT )3N
de Planck, c é a velocidade da luz no vácuo e kB é a constante de Boltzmann.
(b) Determine a pressão do gás.
(c) Calcule a entropia do gás.
(d) Determine a energia interna do gás.

3
Q1. Duas esferas ocas, ambas de massa M e raio R, que estão girando em torno do centro de
massa (CM) do sistema com um período inicial T0, são mantidas distantes d0 = 8R uma da
outra por um fio ideal que passa pelos respectivos centros, conforme ilustra a figura abaixo.
Num dado instante um motor, colocado dentro de uma das esferas, começa a enrolar o fio
lentamente, aproximando uma esfera da outra. Considere que o momento de inércia do
motor seja desprezível quando comparado ao das esferas. Desconsidere efeitos da gravidade e
expresse todos os seus resultados em termos de M, R e T0. Dado: o momento de inércia da
2
casca esférica em relação a um eixo que passa pelo seu centro é MR 2 .
3

(a) Determine o momento angular desse sistema em relação ao seu centro de massa, antes
do motor ser ligado.
(b) Calcule a velocidade angular de rotação, ω f , no instante em que uma esfera encosta-se à
outra.
(c) Calcule a variação da energia cinética do sistema até esse instante.
(d) Qual foi o trabalho realizado pelo motor para fazer com que as esferas se encostem?

Q2. Um pêndulo simples consiste de uma massa m pendurada a partir de um ponto fixo por uma
barra estreita de massa desprezível, inextensível, de comprimento l. Seja g a aceleração da
gravidade local e θ o ângulo entre o pêndulo e a direção vertical. No que segue, faça sempre
a aproximação de pequenos ângulos.

(a) Escreva a equação de movimento desprezando o atrito. Obtenha a frequência natural ω


do pêndulo.

(b) Determine θ ( t ) para as seguintes condições iniciais: θ ( 0 ) = 0 e (0) = Ω .
dt
(c) Escreva a equação do movimento do pêndulo na presença de uma força de atrito viscoso

dada por FR = 2m gl .
dt
(d) Na situação do item (c), determine θ ( t ) para as seguintes condições iniciais: θ ( 0 ) = θ0

e (0) = 0 .
dt

1
Q3. Parte I – Na tentativa de observar o efeito fotoelétrico, um cientista do final do século XIX
realiza um experimento onde utiliza pulsos (1 ms de duração) de luz monocromática, com
comprimento de onda 414 nm e três diferentes potências, dadas respectivamente por P0 , 3P0
e 5P0 , onde P0 = 300 keV/s . Ele escolhe para seu experimento três superfícies metálicas
cujas funções trabalho são conhecidas: Li (2,3 eV), Be (3,9 eV) e Hg (4,5 eV).

(a) Determine para quais superfícies metálicas e potências poderá ocorrer a emissão de
fotoelétrons.
(b) Calcule o número máximo de fotoelétrons que poderia ser emitido pelo pulso de potência
3P0 em cada superfície.

Parte II – Para preencher com elétrons as subcamadas de um átomo usa-se a seguinte regra:
as subcamadas que têm o menor valor de n + l são preenchidas antes; se duas subcamadas
têm o mesmo valor de n + l, preenche-se antes a subcamada com menor valor de n.

(c) Use esta regra para escrever a configuração eletrônica do Sc, que é o átomo com número
atômico mais baixo que apresenta um elétron em uma subcamada d.
(d) Quais são os valores possíveis do momento angular orbital e de sua componente z para
um elétron na subcamada d do Sc?

Q4. Considere um elétron que se encontra confinado dentro de um poço de potencial


unidimensional V ( x ) dado por

⎧⎪ +∞ ,x <0

V ( x ) = ⎪⎨ 0 ,0 <x <d .
⎪⎪
⎩⎪ +∞ ,x >d

(a) Escreva a equação de Schrödinger para este elétron e as condições de contorno que
devem ser satisfeitas pelas funções de onda.
(b) Obtenha as funções de onda normalizadas e determine os valores das energias permitidas
para este elétron.

Admita agora que este elétron se encontre no estado quântico cuja função de onda dentro do
poço é dada por

ψ (x ) =
2
d ( ).
sen
3πx
d

(c) Determine o número quântico n do estado ocupado por este elétron e seu comprimento
de onda nesse estado.
(d) Determine a probabilidade de encontrar este elétron entre x = 0 e x = d/6.

2
Q5. Considere um sistema formado por duas partículas distinguíveis, 1 e 2. Cada uma delas deve
estar em um de dois compartimentos, A e B. A energia de uma partícula é zero quando ela
se encontra no compartimento A, e ε quando no compartimento B. Quando as duas
partículas estão no mesmo compartimento, há um custo energético adicional ∆. O sistema
está em equilíbrio com um banho térmico à temperatura T.

(a) Quais são as possíveis configurações do sistema? Determine a energia de cada uma delas.
(b) Calcule a função de partição Z.
(c) Qual é a probabilidade de cada configuração?
(d) Calcule a energia média do sistema.
(e) Obtenha a entropia do sistema em termos de Z.

3
Q6. Um cabo coaxial é composto por um longo cilindro reto condutor de raio a e uma fina casca
cilíndrica condutora de raio b e concêntrica ao cabo interno. Os dois condutores transportam
correntes iguais e opostas de intensidade i.

(a) Determine o módulo do campo magnético na região entre os dois condutores


( a < r < b ).
(b) Determine o módulo do campo magnético na região externa ao cabo coaxial ( r > b ).
(c) Encontre o módulo do campo magnético no interior do cilindro interno (r < a ) se a
corrente está distribuída uniformemente na seção transversal do mesmo.
(d) Calcule a energia armazenada no campo magnético por unidade de comprimento do
cabo.

Q7. Um capacitor esférico isolado possui carga +Q sobre o condutor interno (raio ra) e carga –Q
sobre o condutor externo (raio rb). A seguir, a metade inferior do volume entre os dois
condutores é preenchida por um líquido de constante dielétrica relativa K, conforme
indicado na seção reta da figura abaixo.

(a) Calcule o módulo do campo elétrico no volume entre os dois condutores em função da
distância r ao centro do capacitor. Forneça respostas para a metade superior e para a
metade inferior desse volume.
(b) Determine a densidade superficial de cargas livres sobre o condutor interno e sobre o
condutor externo.
(c) Calcule a densidade superficial de cargas de polarização sobre as superfícies interna (ra)
e externa (rb) do dielétrico.
(d) Qual é a densidade de carga de polarização sobre a superfície plana do dielétrico?
Explique.
(e) Determine a capacitância do sistema.

rb

ra

1
Q8. A equação de Schrödinger independente do tempo para o problema unidimensional de uma
partícula de massa m sujeita a um potencial de oscilador harmônico é

⎛ =2 d 2 1 2 2⎞
⎜⎜⎝⎜ − 2m dx 2 + 2 m ω x ⎠⎟⎟⎟ ψn(x) = Enψn(x), n = 0,1,2,…

onde ω é a frequência angular do oscilador. Um método para se resolver essa equação


consiste em expressá-la em termos do operador

1 ⎛⎜ m ω = d ⎞⎟
a= ⎜ x + ⎟
2 ⎜⎝ = m ω dx ⎠⎟

e de seu conjugado hermitiano.

(a) A função de onda do estado fundamental do oscilador satisfaz a equação diferencial


a ψ0 ( x ) = 0 . Resolva esta última equação e determine ψ0 ( x ) a menos de uma
constante multiplicativa.
(b) Calcule essa constante normalizando ψ0 ( x ) .
(c) Obtenha o valor da energia do estado fundamental desse oscilador.
(d) Suponha, agora, que o oscilador seja perturbado pelo potencial
V ( x ) = V0 exp ( − x 2 b 2 ) ,

onde V0 e b são constantes reais. Usando teoria de perturbações de primeira ordem,


calcule o deslocamento de energia do estado fundamental.

G G
Q9. Uma partícula de spin ½ tem momento de dipolo magnético µ = γS , onde γ é uma
G = G
constante real e S = σ é o operador de spin, sendo
2

⎛ 0 1 ⎞⎟ ⎛ 0 −i ⎞⎟ ⎛ 1 0 ⎞⎟
σx = ⎜⎜ σy = ⎜⎜ σz = ⎜⎜
⎝ 1 0 ⎠⎟⎟ ⎝ i 0 ⎟⎟⎠ ⎝ 0 −1 ⎟⎟⎠
, ,

G
as matrizes de Pauli. Se essa partícula está imersa num campo magnético uniforme B , o
G G
hamiltoniano que governa a dinâmica do spin é H = −µ ⋅ B . No que segue, suponha que o
campo magnético esteja na direção do eixo Oz.

(a) Dê a forma explícita do operador hamiltoniano como uma matriz 2 x 2, em termos de


γ , = e B.
(b) Escreva as expressões para os estados estacionários como vetores-coluna
normalizados e indique suas respectivas energias.

2
(c) No instante inicial, t = 0, a partícula é preparada no estado de spin
1 ⎛ 1 ⎞
χ(0) = ⎜ i α ⎟⎟ (com α real).
2 ⎜⎝e ⎠⎟

Qual será o estado de spin, χ (t ) , num instante t posterior?


(d) Nesse instante posterior é feita uma medida de Sx, a componente do spin segundo o eixo
Ox. Qual a probabilidade P+ (t ) de se obter o valor += 2 ?

Q10. Considere um mol (n = 1) de um gás ideal monoatômico, inicialmente no estado de


equilíbrio térmico especificado pela pressão P0 e pelo volume V0. Esse gás sofre uma
compressão adiabática reversível que o leva a ocupar um volume V0/2. Determine:

(a) a variação de energia interna desse gás devido a essa compressão;


(b) a variação de entropia do gás nessa compressão.

Após essa compressão adiabática, o gás, sempre isolado do resto do universo por paredes
adiabáticas, sofre uma expansão completamente livre até o volume original V0. Determine:

(c) a variação de temperatura do gás devido à expansão livre;


(d) a variação de entropia do gás nessa expansão livre.

3
Q1. Considere um corpo de massa M de seção transversal circular de raio R que rola sem desliza-
mento sobre um plano que possui um ângulo de inclinação θ em relação à horizontal, conforme
mostra a figura abaixo. O coeficiente de atrito estático entre o corpo e o plano é µe . O mo-
mento de inércia do corpo em relação a um eixo passando pelo ponto O é I e a aceleração da
gravidade é g.
y
R x
O
h

(a) Desenhe o diagrama de forças para o corpo. Escreva a equação que relaciona a velocidade
angular, ϕ̇, de rolamento do corpo e a velocidade de translação, ẋ, que caracteriza um
rolamento sem deslizamento.
(b) Determine a aceleração ẍ, associada à translação do corpo ao longo do plano inclinado,
em termos dos parâmetros que constam no enunciado.
(c) Assuma que o corpo inicia o seu movimento a partir do repouso na origem do sistema
de coordenadas cartesianas indicado na figura. Calcule a energia mecânica no inı́cio e no
final do movimento. A energia mecânica do sistema é conservada?
(d) Calcule o momento de inércia I considerando que o corpo seja (i) um anel e (ii) um disco.
Assuma que as massas dos corpos estão uniformemente distribuı́das. Suponha agora que
o ângulo θ possa ser variado. A partir de qual θ cessa o movimento de rolamento puro e
o corpo começa a deslizar, nos casos (i) e (ii) acima? Deixe a resposta em termos de µe .
Q2. Considere o pêndulo invertido da figura abaixo, composto por uma barra de massa M e mo-
mento de inércia I0 em relação ao seu centro de massa, cujas coordenadas são (X,Y). A barra
pode girar livremente no plano xy em torno de um eixo de rotação que passa pela posição
(xp ,yp ), a uma distância ` do centro de massa. A aceleração da gravidade é g.
y

q (X,Y)
(xp,yp) l
g
x
(a) Escreva as equações para a energia cinética e potencial do sistema em termos de X, Y e θ.
Para os itens (b), (c) e (d) assuma que um agente externo faz o eixo de rotação oscilar hori-
zontalmente com frequência angular ω, ou seja, tem-se yp (t) = 0 e xp (t) = A cos(ωt).
(b) Escreva a lagrangiana do sistema em termos da coordenada generalizada θ.
(c) Escreva a equação de movimento para a lagrangiana do item (b).
(d) Considere que o sistema executa pequenas oscilações (θ pequeno). Mostre que neste caso,
θ(t) = α cos(ωt) + β sen(ωt) é uma solução para o problema. Determine α e β.

1
Q3. Para os itens (a), (b) e (c), admita que no modelo de Bohr para uma partı́cula de massa m se
movendo numa órbita circular de raio r e velocidade v, a força Coulombiana fosse substituı́da
por uma força central atrativa de intensidade k r (sendo k uma constante). Admita que os
postulados de Bohr sejam válidos para este sistema. Para esta situação:

(a) Deduza a expressão para os raios rn das órbitas de Bohr permitidas neste modelo em
função do número quântico n e das constantes k, ~ e m. Diga quais os valores possı́veis
de n neste caso.
(b) Lembrando que para o caso desta força central, a energia potencial correspondente é
V (r) = kr2 /2, deduza a expressão para as energias En das órbitas permitidas em função
do número quântico n e das constantes k, ~ e m. Determine a frequência irradiada quando
a partı́cula faz uma transição de uma órbita para outra adjacente.
(c) Calcule o comprimento de onda de de Broglie associado à partı́cula em um estado de
energia correspondente ao número quântico n = 2 em função de k, ~ e m.

Para o item (d), considere um feixe de raios X, contendo radiação de dois comprimentos de onda
distintos, difratados por um cristal cuja distância entre planos de difração é 1 nm (10−9 m). A
figura abaixo apresenta o espectro de intensidade na região de pequenos ângulos (medidos em
relação à direção do feixe incidente).

(d) Determine os comprimentos de onda dos raios X presentes no feixe. Utilize π = 3.

2
Q4. Numa experiência de efeito fotoelétrico, luz de comprimento de onda 414 nm e intensidade I0
incide sobre uma superfı́cie limpa de um metal cuja função trabalho é φ = 2,5 eV.

(a) Calcule a energia cinética máxima dos fotoelétrons.


(b) Se a intensidade de luz incidente for duplicada, o que ocorre com a energia cinética dos
fotoelétrons?

Considere agora a experiência de espalhamento Compton em que um elétron de massa m0 em


repouso espalha um fóton de comprimento de onda λ = 2λc ≡ 2h/(m0 c). Após o espalhamento,
o fóton perde metade de sua energia.

(c) Calcule o comprimento de onda do fóton espalhado (expresse seu resultado apenas em
função de λc ) e determine o seu ângulo de espalhamento.
(d) Calcule a energia total e o momento linear do elétron após a colisão (expresse seu resultado
em função de m0 e c).

Q5. Imagine que um material magnético unidimensional possa ser modelado como uma cadeia
linear de N + 1 spins. Cada spin interage com os seus primeiros vizinhos de tal maneira que a
energia do sistema seja E = n², onde n é o número de paredes de domı́nio separando regiões de
spin ↑ das regiões de spin ↓, como representado na figura abaixo, sendo as paredes de domı́nio
indicadas por linhas tracejadas. A energia por parede de domı́nio é ². Considere N À 1 e
n À 1.

(a) Determine de quantas maneiras as n paredes de domı́nio podem ser arranjadas.


(b) Determine a entropia S(E) do sistema contendo n paredes de domı́nio.
(c) Determine a energia interna E como função da temperatura, E(T ). Expresse seu resultado
em termos de N , ², T e constantes fı́sicas apenas.
(d) Esboçe a função E(T ), indicando os valores de E para T = 0 e T → ∞.

3
Q6. Coloca-se uma esfera metálica descarregada, de raio R, numa região do espaço inicialmente
~ i = E0 k̂. Escolha a origem do sistema de
preenchida por um campo elétrico dado por E
coordenadas no centro da esfera.

(a) Esboce as linhas do campo elétrico em toda a região do espaço. Justifique o esboço
utilizando argumentos fı́sicos.
(b) Determine o campo elétrico E ~ f (~r) em toda a região do espaço. Em particular, encontre
os campos para os pontos em que |~r| À R e |~r| ≈ R e verifique se eles são consistentes
com o esboço no item (a).
(c) Ache a densidade de carga na esfera. Se a esfera possuir raio igual a 10 cm e E0 = 100
N/C, calcule as cargas acumuladas nos hemisférios norte e sul da esfera.
(d) Suponha que a esfera metálica seja substituı́da por uma esfera dielétrica. Discuta qua-
litativamente o que ocorre neste caso e esboce as linhas do campo elétrico em toda a
região do espaço.

Q7. Considere o arranjo hipotético ilustrado na figura abaixo, em que um fio sólido de raio a
estendido ao longo do eixo z conduz uma corrente elétrica I, uniformemente distribuı́da sobre
a sua seção transversal, que é mantida constante. A pequena lacuna no fio, de largura w ¿ a,
forma um capacitor de placas paralelas. A carga no capacitor é zero no instante t = 0.

(a) Encontre o vetor campo elétrico na lacuna em função da distância ρ a partir do eixo z e
do tempo t, além dos parâmetros I,w e a. Despreze os efeitos de borda.
(b) Encontre o vetor campo magnético na lacuna em função de ρ e t e dos parâmetros I,w e
a.
(c) Calcule a densidade de energia eletromagnética uem e o vetor de Poynting na lacuna,
indicando explicitamente a sua direção e o seu sentido.
(d) Determine a energia total Uem na lacuna em função do tempo. Compare a taxa de variação
de Uem com o tempo e o fluxo de energia por unidade de tempo (fluxo de potência), obtido
fazendo-se a integral de superfı́cie do vetor de Poynting.

a +σ −σ z
I I
••• •••

1
Q8. Considere uma partı́cula de massa m na presença de um potencial harmônico V (x) = 12 mω2 x2 ,
onde ω é a frequência angular do oscilador e x é a coordenada da partı́cula (1-dim).
(a) São dadas as funções de onda estacionárias correspondentes ao estado fundamental ψ0 e
ao primeiro estado excitado ψ1 :
³ mω ´ ³ mω ´
2
ψ0 (x) = A exp − x , ψ1 (x) = B x exp − x2 ,
2~ 2~
onde A e B são constantes de normalização. Calcule A e B supondo que as funções de
onda sejam reais.
(b) Seja E0 a energia do estado fundamental. Sabemos que E1 = E0 + ~ω para o primeiro
estado excitado, já que o quantum de energia do oscilador é ~ω. Usando a equação de
Schrödinger, encontre a energia E0 .
(c) Para os estados estacionários, o valor médio da posição hxi é sempre nulo. Construa uma
função de onda não estacionária como combinação linear de ψ0 e ψ1 com coeficientes reais,
tal que o valor médio hxi seja o maior possı́vel. Em outras palavras, considere o estado
normalizado p
ψ (x) = 1 − β 2 ψ0 (x) + β ψ1 (x) ,
com 0 ≤ β 2 ≤ 1 e determine o coeficiente β que maximiza o valor de hxi.
(d) Suponha que a função de onda construı́da no item anterior descreva o estado do oscilador
harmônico no tempo t = 0. Escreva a função de onda do estado para um tempo t > 0
arbitrário, supondo que nenhuma medição foi feita sobre o sistema. Para esse estado,
avalie o valor médio da posição hxi(t) em função do tempo.
Q9. Seja uma partı́cula com momento angular l = 1.
(a) Na representação onde as matrizes de L2 e Lz são diagonais, obtenha a matriz da compo-
nente Lx . Lembre que a matriz de Lx deve representar um operador hermitiano. Sugeri-
mos usar os operadores escada L± .
(b) Calcule os autovalores de Lx .
(c) Encontre o autovetor de Lx com o maior autovalor.
(d) Suponha agora que você encontrou o maior autovalor numa medição de Lx . Calcule as
probabilidades de medir respectivamente +~, 0 e −~ numa medição posterior de Lz .
Q10. Um mol de um gás ideal monoatômico se encontra na temperatura T e ocupa um volume V .
A energia interna por mol de um gás ideal é dada por u = cV T , onde cV é o calor especı́fico
molar, que é considerado constante. Responda as questões abaixo:
(a) Considere a situação em que o gás se encontra em contato com um reservatório térmico
na temperatura T e sofre uma expansão quase-estática reversı́vel na qual o seu volume
passa de V para 2V . Calcule o trabalho realizado pelo gás durante a sua expansão.
(b) Ainda com relação ao processo fı́sico descrito no item (a), determine o calor trocado pelo
gás com o reservatório térmico.
(c) Determine as variações de entropia do gás e do reservatório térmico no processo descrito
no item (a).
(d) Considere agora a situação em que o gás está isolado e sofre uma expansão livre na qual o
seu volume passa de V para 2V . Determine as variações de entropia do gás e do universo
durante o processo de expansão livre.

2
Q1. A interação entre dois átomos de massas m1 e m2 , que formam uma molécula, pode ser descrita
pelo potencial de Lennard-Jones dado por
"µ ¶ µ ¶6 #
12
b b
V (x) = A −2 ,
x x

onde A e b são parâmetros positivos e x a separação interatômica. Trate o problema classica-


mente e despreze qualquer tipo de rotação da molécula.

(a) Determine a posição de equilı́brio em função de A e b.


(b) Calcule a menor energia para dissociar a molécula.
(c) Mostre que o equilı́brio é estável e calcule a frequência de pequenas oscilações em torno
da posição de equilı́brio.
(d) Desenhe um gráfico do potencial de Lenard Jones indicando os parâmetros obtidos nos
itens (a) e (b).

Q2. Atualmente, a totalidade dos atletas de alto nı́vel de salto em altura utiliza uma técnica para
o salto batizada de “Salto Fosbury”. Suponha que nesse salto o atleta possa ser aproximado
por uma barra rı́gida de comprimento `, inclinada por um ângulo θ e movendo-se com uma
velocidade ν0 para a direita conforme mostra a figura abaixo. No momento do “salto” essa
barra começa a girar em torno do ponto P. A barra possui uma massa m homogeneamente
distribuı́da.

v0 v
q q
P P

(a) Calcule o momento de inércia da barra em relação à sua extremidade.


(b) A conservação de uma grandeza fı́sica permite que a barra obtenha uma componente
vertical para a velocidade do seu centro de massa. Qual é essa grandeza fı́sica?
(c) Calcule a componente vertical νv da velocidade do seu centro de massa imediatemente
após atingir o ponto P.
(d) Qual é a altura máxima atingida pelo seu centro de massa em relação ao solo.

1
Q3. Uma fonte produz um feixe de nêutrons com energia cinética média de 0,0133 eV e incerteza
relativa na velocidade, ∆v/v, de 1 × 10−4 . Num determinado instante, a função de onda
unidimensional de um nêutron é descrita por um pacote de ondas dado por
µ ¶
x2
Ψ(x) = A exp − exp(ik0 x) .
2(∆x)2

Nesta expressão, A é uma constante, ∆x é a incerteza padrão na posição, e ~k0 é o momento


linear médio.

(a) Estime o comprimento de onda de de Broglie do nêutron e identifique a região do espectro


eletromagnético correspondente a esse comprimento de onda.
(b) Estime a temperatura associada a essa fonte de nêutrons.
(c) Determine a constante A, expressando-a em termos de ∆x.
(d) Com um pacote de ondas desse tipo, o produto das incertezas na posição e no momento
é o mı́nimo permitido pelo princı́pio da incerteza. Estime ∆x neste caso.

Q4. Átomos muônicos são formados por um núcleo de carga Ze, com o muon negativo orbitando
ao redor do núcleo. O muon possui carga igual à do elétron, mas massa 207 vezes maior que a
deste. Para um átomo muônico cujo núcleo é formado por apenas um próton (mp = 1836 me ),
estime:

(a) a massa reduzida do sistema, em termos da massa do elétron me .


(b) o raio da primeira órbita de Bohr desse átomo muônico,
(c) o comprimento de onda da primeira linha da série de Lyman, sabendo que
µ ¶
1 1
= Rµ · 1 − 2 ,
λ ni

onde Rµ é a constante de Rydberg para o átomo muônico.


(d) Qual é a região do espectro eletromagnético que permite estudar a emissão da série de
Lyman desse átomo muônico?

Q5. A função de partição de um gás monoatômico de N partı́culas interagentes pode ser escrita
como: µ ¶N µ ¶ 3N µ ¶
V − Nb mkB T 2 N 2a
Z= exp ,
N 2π~2 V kB T
onde a e b são constantes positivas e m a massa da partı́cula.

(a) Determine a energia livre de Helmholtz do gás.


(b) Determine a equação de estado desse gás, em termos da pressão, do volume especı́fico
v = V /N , da temperatura e de constantes.
(c) Determine a energia interna especı́fica u = U/N do gás.
(d) Considere que o gás sofra um processo de expansão livre no qual seu volume inicial V é
duplicado no interior de um recipiente feito de paredes adiabáticas. Calcule a variação da
temperatura absoluta que ocorre no processo.

2
Q6. Um cabo coaxial é constituı́do por um fio sólido de raio a envolto por uma casca cilı́ndrica
concêntrica de raio b, com comprimento L >> b. Ele é usado como linha de transmissão
entre uma bateria de fem V e uma resistência R, como indicado na figura abaixo. Despreze a
resistência do cabo.

(a) Calcule o vetor campo elétrico no interior do cabo coaxial (a < r < b).
(b) Calcule o vetor campo magnético no interior do cabo coaxial (a < r < b).
~ B
(c) Calcule o vetor de Poynting, indicando esquematicamente os vetores E, ~ eS~ com relação
à seção transversal do cabo coaxial. O que aconteceria se os pólos da bateria fossem
invertidos?
(d) Usando o vetor de Poynting, calcule a potência que flui da bateria para o resistor e explique
por que este resultado é esperado.

Observação: Indique claramente as superfı́cies gaussianas e/ou caminhos de integração utiliza-


dos nos cálculos acima.

Q7. Considere uma carga puntiforme Q > 0 a uma distância D de uma placa infinita, condutora e
aterrada, como ilustrada abaixo.
(a) Desenhe as linhas de campo elétrico e as equipotenciais. Justifique seu desenho.
(b) Calcule as componentes x̂ e ŷ do vetor campo elétrico, em todo o espaço à esquerda da
placa, em termos das componentes do ponto P ilustrado na figura abaixo.
(c) Qual a densidade de carga na placa?
(d) Determine a força exercida pela placa sobre a carga Q.

P P
y y
x
Q Q
x
D

1
Q8. Uma partı́cula de massa m encontra-se inicialmente em um poço de potencial unidimensional
dado por 
 ∞, x ≤ − L2 ,
V (x) = 0, − L2 < x < L2 , (1)

∞, x ≥ L2 .

(a) Calcule as autofunções e as autoenergias do estado fundamental e do primeiro estado


excitado.
(b) Considere agora que o potencial expande-se instantaneamente para

 ∞, x ≤ −L,
V (x) = 0, −L < x < L, . (2)

∞, x ≥ L.

Calcule a probabilidade da partı́cula realizar uma transição do estado fundamental do


potencial (1) para o primeiro estado excitado do potencial (2).
(c) Calcule a probabilidade da partı́cula continuar no estado fundamental após a expansão.
(d) Considere que a partı́cula se encontre no estado fundamental após a expansão. Calcule a
probabilidade da partı́cula ser encontrada fora da região −L/2 < x < L/2.

Q9. As matrizes de Pauli, σx , σy e σz , são extremamente importantes quando se considera uma


partı́cula de spin 1/2.

(a) Utilize explicitamente a representação matricial dos operadores de Pauli e encontre seus
autovalores e autovetores, bem como o comutador [σy ,σx ].
(b) Considere um estado arbitrário para uma partı́cula de spin 1/2 dado por |ψi = a|−i+b|+i,
com |a|2 + |b|2 = 1, sendo {|−i,|+i} autovetores de σz . Mostre como este estado é
transformado sob a ação de cada um dos operadores σx , σy e σz , independentemente.
(c) Mostre como o operador exp (iασx ) atua sobre o estado |ψi.
(d) Quais imposições devem ser consideradas sobre α para que o operador do item (c) seja
hermitiano? e para que seja unitário?

Q10. Um corpo de capacidade térmica a pressão constante, CP (independente da temperatura), que


se encontra inicialmente na temperatura T1 , é colocado em contato térmico com um reservatório
térmico na temperatura T2 , sendo x ≡ T1 /T2 < 1. Após o equilı́brio térmico ter sido atingido,
determine:

(a) A variação da entropia do corpo.


(b) A variação da entropia do reservatório.
(c) A variação da entropia do Universo.
(d) Verifique se o resultado obtido no item (c) está de acordo com a 2a lei da termodinâmica.

2
Q1. Uma partı́cula de massa m colide com uma barra fina e homogênea inicialmente em repouso,
de momento de inércia I = M l2 /12 relativo ao seu centro de massa, sendo M a sua massa e
l o seu comprimento. Antes da colisão, a partı́cula move-se perpendicularmente à barra com
velocidade v0 . A partı́cula colide elasticamente com a extremidade da barra, conforme ilustra
a figura ao lado

(a) Escreva as equações que expressam as grandezas


fı́sicas conservadas na colisão.
(b) Determine o vetor velocidade de translação do cen-
tro de massa da barra imediatamente após a co-
lisão.
(c) Determine o vetor velocidade angular de rotação da
barra imediatamente após a colisão.
(d) Determine o vetor velocidade da partı́cula imedia-
tamente após a colisão.

Q2. Uma partı́cula de massa m pode se mover sem atrito num aro de raio R, como mostrado na
figura abaixo. O aro gira com velocidade angular constante ω em torno do eixo vertical, como
mostra a figura abaixo. Considere a aceleração da gravidade g.

(a) Determine a energia cinética da partı́cula em função


de θ, θ̇, R, m, e ω.
(b) Determine a lagrangiana da partı́cula, adotando
energia potencial nula no ponto correspondente a
θ = 0.
(c) Determine a equação de movimento da partı́cula.
(d) Determine os pontos de equilı́brio.

Q3. Um experimento de efeito Compton, como ilustrado na figura abaixo, foi planejado para ser
executado no Laboratório Nacional de Luz Sı́ncrotron (LNLS), cujo espectro de emissão é
mostrado abaixo à direita.
Foi escolhida a energia de 10 keV para realizar o experimento. Para essa energia:

(a) estime o fluxo de fótons do feixe escolhido, nas unidades do gráfico abaixo.
(b) determine o comprimento de onda desse feixe de fótons.

1
Fendas em um anteparo de chumbo (Pb) foram colocadas na frente do feixe de raios X
espalhados pelo alvo de grafite, a fim de selecionar o ângulo θ. Abaixo são fornecidas as
seções de choque do Pb como função da energia para os processos de espalhamento (σS ),
de efeito fotoelétrico (σP E ) e de produção de pares (σP R ), bem como o valor total (σ).

2
(c) Na energia escolhida para o experimento, qual processo de absorção do feixe pelo chumbo
tem a maior contribuição na atenuação?
(d) Para que valores aproximados de energia os efeitos de espalhamento predominam sobre
os outros processos de absorção no chumbo?
(e) Estime a espessura do anteparo de chumbo para que ele atenue a intensidade do feixe
incidente de um fator igual a e−3 . Para esse cálculo considere que o chumbo possui uma
densidade aproximada de 3 ×1022 átomos/cm3 .
(f) Um monocristal de silı́cio com distância interplanar de aproximadamente 0,31 nm é es-
colhido como espectrômetro do experimento. Determine o menor ângulo que o feixe
espalhado pelo alvo de grafite deve fazer com a superfı́cie do monocristal para que o feixe
seja difratado em direção ao detetor.

Q4. Utilizando o modelo de Bohr:


(a) Deduza a expressão para os nı́veis de energia do ı́on He+ (Z = 2, MHe+ >> me ) e calcule
os valores das enegias até n = 5.
Com os resultados deste item, determine:
(b) a energia de ionização do He+ ,
(c) o comprimento de onda de uma linha de emissão do He+ na região do espectro visı́vel,
(d) Dois ı́ons de He+ no estado fundamental e com mesma energia cinética colidem frontal-
mente. Cada qual emite um fóton de comprimento de onda 120 nm e fica com energia
cinética final nula, no estado fundamental. Qual é a velocidade dos ı́ons antes da colisão?

Q5. Um gás ideal de moléculas diatômicas polares, cada uma com momento de dipolo elétrico

→ −

µ , encontra-se a uma temperatura T e está sujeito a um campo elétrico E . As orientações
dos dipolos são definidas pelos ângulos θ (0 ≤ θ ≤ π) e φ (0 ≤ φ ≤ 2π) de um sistema de
coordenadas esféricas cujo eixo-z é paralelo ao campo elétrico. A probabilidade de encontrar
uma molécula com orientação do dipolo dentro do elemento dθdφ vale ρdθdφ onde a densidade
de probabilidade ρ(θ,φ) é dada por
1
ρ(θ,φ) = sin θe−βE ,
A
R
e está normalizada de acordo com ρ(θ,φ)dθdφ = 1. A constante A é um fator de normalização,
β = 1/kB T , kB é a constante de Boltzmann e E é a energia de interação do momento de dipolo


com o campo, dada por E = −− →
µ . E = −µE cos θ.

(a) Determine A como função de T , E e µ.


(b) O momento de dipolo médio por molécula é definido pela média P = µhcos θi. Determinar
P como função de T e E.
(c) Esboce o gráfico de P versus E para T constante.
(d) A susceptibilidade elétrica é definida por χ = ∂P/∂E. Determine χ a campo nulo e mostre
que ela é inversamente proporcional à temperatura T . Notar que para pequenos valores
de x vale coth x ≈ 1/x + x/3.

3
Q6. Considere um fio infinitamente longo, carregado uniformemente com carga negativa de den-
sidade λ, ao longo do eixo x. Suponha que acima deste fio, na posição ~r = y1 ̂, exista uma
carga puntiforme q positiva. O fio e a carga estão em repouso no referencial S. Um segundo
referencial, S 0 , está se movendo para à direita, com uma velocidade relativı́stica de módulo v,
como mostra a figura abaixo. Tome a velocidade da luz como sendo c.

(a) Calcule a força resultante, F~res , atuando na carga q no referencial S.


(b) Encontre a densidade de carga λ0 no referencial S 0 . Note que nesse referencial, o fio car-
regado está em movimento, o que implica na existência de uma corrente elétrica. Calcule
essa corrente, indicando o sentido dela.
(c) Qual a força resultante, F~res
0
, no referencial S 0 ? Compare com F~res , obtida no item (a).
Quais as direções e sentidos dessas duas forças?
(d) A relação entre as forças eletromagnéticas F~res e F~res
0
, obtidas nos itens (a) e (c), são
consistentes com os resultados da teoria da relatividade? Justifique a sua resposta.
Dica: Pela teoria da relatividade restrita, as transformações entre F~⊥ e F~⊥0 e entre Fk
e Fk0 , onde ⊥ e k indicam as direções perpendiculares e paralela ao eixo x (direção do
movimento de S 0 ), respectivamente, podem ser obtidas sabendo-se que (i) a energia e
momento (E,~p) nos referenciais S e S 0 se transformam como o tempo e espaço (t,~r) e que
(ii) a segunda lei de Newton, F~ = d~p/dt é válida também na relatividade restrita. Faça a
transformação somente na direção de F~res e F~res
0
.
Q7. Um condutor esférico maciço, de raio a e carregado com carga Q > 0, está envolto por um
material dielétrico esférico, de constante dielétrica ²r = ²/²0 e raio externo b, conforme mostra
a figura abaixo.

(a) Determine o campo elétrico em todo o espaço e esboce um graf́ico de seu módulo E(r).
(b) Determine o potencial no centro das esferas, tomando-se como zero o potencial no infinito.
(c) Encontre as distribuições das cargas livre e ligada (de polarização) nas esferas condu-
tora e dielétrica. Faça uma figura mostrando onde as densidades de cargas se localizam,
indicando se são positivas ou negativas.
(d) Calcule a energia eletrostática do sistema.

1
Q8. Um elétron de massa m está confinado numa esfera de raio a, isto é, submetido ao potencial
V (r) = 0 para r < a e V (r) = ∞ para r > a.
(a) Escreva a equação de Schrödinger independente do tempo para a função u(r) = rR(r),
sendo ψ(r,θ,φ) = R(r) Yl,m (θ,φ) a função de onda completa desse elétron.
(b) Imponha a devida condição de contorno e encontre, para o estado fundamental, ψ(r,θ,φ)
e a respectiva energia.
(c) Escreva a energia do estado fundamental em termos do volume da esfera, massa do elétron
e constantes fundamentais.
(d) Encontre a pressão exercida por esse elétron na superfı́cie da esfera. Expresse em termos
da massa m, raio a e constantes universais.
Q9. Duas partı́culas com spin 1/2 se aproximam e interagem segundo o hamiltoniano
4a(t) ~ ~
H= S1 · S2 ,
~
sendo a(t) = a0 , constante, para 0 < t < τ e a(t) = 0 para t < 0 e t > τ . Em t = −∞ o estado
do sistema é | + ,−i, sendo |±i autovetores do operador Si,z com autovalores ±~/2.
(a) Escreva a matriz H na base dos autovetores de S1,z e S2,z .
(b) Determine os autovalores e autovetores de H.
(c) Qual é o estado |Ψ(t)i do sistema para 0 < t < τ ?
(d) Qual é o estado |Ψ(t)i do sistema para t > τ qualquer?
(e) Qual a probabilidade de uma medida de S1,z fornecer o valor ~/2 para t > τ ?
(f) Após ∆t segundos dessa medida, qual a probabilidade de uma medida de S2,z dar o valor
−~/2?
Q10. Um mol de uma determinada substância percorre o ciclo formado pelos trechos A→B, B→C,
C→D e D→A conforme mostrado no diagrama temperatura T versus entropia S da figura.
T
B C

A D

São dados TA , SA e as razoẽs α = TB /TA e r = SC /SA . Determine em função dos dados do


problema:
(a) o calor trocado em cada um dos trechos e o trabalho total realizado no ciclo;
(b) o rendimento η de um motor que opera de acordo com esse ciclo;
(c) o trabalho em cada um dos trechos do ciclo, considerando que a substância seja um gás
ideal de capacidade térmica a volume constante CV . Sugestão: utilize os resultados do
item (a).
(d) Esboce o ciclo no diagrama P − V para a substância considerada no item anterior orien-
tando e identificando o tipo de processo termodinâmico associado a cada um dos trechos.

2
Q1. Um disco uniforme, de seção reta circular de raio R, massa M e momento de inércia I (com
relação ao eixo perpendicular ao plano do disco e que passa pelo seu centro), encontra-se preso
a uma mola de constante k, massa desprezı́vel e um certo comprimento de repouso, como é
mostrado na figura abaixo. O disco rola sobre a superfı́cie sem deslizar e seu movimento está
confinado ao plano da figura.

1. Escreva a equação para a energia mecânica do sistema em função da velocidade do centro


de massa e da distensão da mola.
2. Obtenha a equação de movimento para o centro de massa do disco.
3. Determine a frequência angular de oscilação do centro de massa do disco.

Q2. Uma partı́cula de massa m move-se em um potencial V (r) = −C/(3r3 ), sendo C uma constante
positiva. Considere que a partı́cula possua momento angular L diferente de zero.

1. Escreva a equação para a energia mecânica da partı́cula em termos da distância r à origem,


da sua derivada temporal ṙ, do momento angular L, da massa m e da constante C.
2. Considerando os termos que só dependem de r na energia mecânica como um potencial
efetivo Vef (r), esboce o graf́ico de Vef (r).
3. Existem órbitas circulares para essa partı́cula? Em caso afirmativo, determine o raio de
cada uma dessas possı́veis órbitas e discuta a estabilidade das mesmas.
4. Calcule a energia mecânica mı́nima, Emin , acima da qual a partı́cula vinda do infinito é
capturada pelo potencial, ou seja, não retorna mais para o infinito.

Q3. (a) As seguintes afirmações referem-se ao efeito fotoelétrico. Responda Verdadeiro (V) ou Falso
(F) e justifique brevemente a sua resposta (máximo de três linhas). Respostas sem justificativas
ou com justificativas erradas não serão consideradas. Responda na folha de Respostas.

1. Incide-se luz num material fotoelétrico e não se observa a emissão de elétrons. Para que
ocorra a emissão de elétrons no mesmo material basta que se aumente suficientemente a
intensidade da luz incidente.
2. Incide-se luz num material fotoelétrico e não se observa a emissão de elétrons. Para que
ocorra a emissão de elétrons no mesmo material basta que se aumente suficientemente a
frequência da luz incidente.
3. No contexto do efeito fotoelétrico, o potencial de corte é a tensão necessária para deter
os elétrons que escapam do metal com a menor velocidade possı́vel.

1
4. Quando luz azul incide sobre uma placa de zinco, ela não produz efeito fotoelétrico, mas
quando iluminada com luz vermelha ocorre emissão de elétrons.
5. Quanto maior for a frequência da luz incidente, maior será a energia cinética dos elétrons
emitidos.

(b) Considere o efeito fotoelétrico inverso, ou seja, a emissão de fótons em consequência do


bombardeio de um material com elétrons de alta velocidade. Calcule a frequência máxima que
podem ter os fótons emitidos se a superfı́cie é bombardeada com elétrons com velocidade c/2,
onde c é a velocidade da luz.

Q4. A energia da radiação de corpo negro, por unidade de volume e por unidade de intervalo de
frequência, é dada por:

8πh ν3
uν (ν) = ,
c3 ehν/kB T − 1
onde ν representa a frequência do fóton e T a temperatura da radiação.

1. Deduza a expressão para a energia total E de um gás de fótons em um volume V . Qual


é a dependência de E com a temperatura?
2. Esboce gráficos de uν (ν) para duas temperaturas T1 e T2 , sendo T1 < T2 .
3. Escreva as formas assintóticas de uν (ν) no caso de frequências muito altas (lei de radiação
de Wien) e no caso de frequências muito baixas (lei de radiação de Rayleigh-Jeans).
4. Imagine que o Universo seja uma cavidade esférica de paredes impenetráveis e raio 1028 cm,
contendo um gás de fótons em equilı́brio térmico. Se a temperatura dentro da cavidade
for de 3 K, estime a quantidade de energia contida nessa cavidade.
5. Supondo que o Universo se expanda adiabaticamente, calcule a temperatura que ele terá
quando o seu volume for o dobro do valor atual (a entropia do gás de fótons é S ∝ V T 3 ).

Q5. Considere um sistema de N átomos localizados e não interagentes. Cada átomo pode estar em
um dos três estados quânticos rotulados pelo número quântico k, com k = −1, 0, 1. Um átomo
tem a mesma energia ε1 > 0 no estado k = 1 ou no estado k = −1. Um átomo no estado k = 0
tem energia ε0 = 0. Determine:

1. A função de partição do sistema.


2. A probabilidade p0 de um átomo se encontrar no estado com energia 0. Determine o
comportamento de p0 nos limites de altas e baixas temperaturas e esboce o gráfico de p0
versus T .
3. As expressões para a energia interna e para a entropia como função da temperatura T .
Determine os valores assintóticos da energia e da entropia nos limites de altas e baixas
temperaturas. A terceira lei da termodinâmica é observada?
4. Esboce o gráfico da entropia como função da temperatura.

2
Q6. No instante inicial t = 0, uma partı́cula de massa m e carga q encontra-se na posição x0 x̂ e com
velocidade v0 ŷ. Os campos de força agindo sobre a partı́cula são devidos somente ao potencial
~ dados por
elétrico φ e ao potencial vetor A,

Φ(~r) = α0 x + aα1 ,
~ r) = β0 (ẑ × ~r) + a β1 û eû·~r/a ,
A(~
2
onde α0 , α1 , a, β0 e β1 são constantes reais e û é um versor constante e real.
~ r).
1. Calcule o vetor campo elétrico em todo o espaço, E(~
~ r).
2. Determine o vetor indução magnética em todo o espaço, B(~
3. Existe algum valor para a velocidade inicial v0 tal que a trajetória da partı́cula seja uma
reta? Em caso afirmativo, calcule-o.

Q7. Durante uma tempestade, uma nuvem cobre a cidade de São Paulo a uma altura h = 500 m em
relação ao solo. Vamos supor que a largura da nuvem seja bem maior que essa altura h. Um
balão meteorológico equipado com um sensor de campo elétrico é então lançado verticalmente
a partir do solo. Os dados coletados pelo sensor estão ilustrados na figura abaixo, onde E(z) é
o módulo do campo elétrico em função da altitude (z = 0 no solo). A espessura da nuvem na
direção vertical é igual a 1200 m e sabe-se que a densidade de carga elétrica é sempre negativa
no seu interior.

1. Indique, em um diagrama, a direção e sentido do campo elétrico nas regiões abaixo, dentro
e acima da nuvem.
2. Calcule a densidade volumétrica de carga na atmosfera em função da altitude, ρ(z), e
esboce o seu gráfico.
3. Para quais valores de z o potencial elétrico é máximo ou mı́nimo? Calcule o potencial
elétrico nesses pontos? Tome V = 0 no solo.

E (V/m)

80

40

500 1300 1700 z (m)

1
Q8. Responda as questões abaixo o mais detalhadamente possı́vel. Não deixe nada indicado. Con-
clua.
Considere um operador hermitiano H e mostre que:
1) os autovalores de H são necessariamente reais;
2) os autovetores de H correspondentes a autovalores diferentes são ortogonais.
Um operador A, que corresponde ao observável a, tem dois autoestados normalizados, |ϕ1> e
|ϕ2>, com autovalores a1 e a2 , respectivamente, e a1 6= a2 . Um outro operador B, que corres-
ponde ao observável b, tem dois autoestados normalizados, |χ1> e |χ2>, com autovalores b1 e
b2 , respectivamente, e b1 6= b2 . Os dois conjuntos de autoestados (ou bases) estão relacionados
por:

1 1
|ϕ1>= √ (|χ1> +3|χ2>) e |ϕ2>= √ (3|χ1> −|χ2>).
10 10
3) Encontre a relação inversa entre as bases, ou seja, os |χ> s em termos dos |ϕ> s.
Sobre esse sistema, podem ser feitas medidas em sequência. Calcule as probabilidades pedidas
nos casos abaixo:

4) a é medido e é encontrado o autovalor a1 . Imediatamente após, b é medido e é encontrado


o autovalor b1 . Em seguida, a é medido novamente. Qual é a probabilidade de se obter
novamente o autovalor a1 nessa última medida?
5) a é medido e é encontrado o autovalor a1 . Após essa medida de a, mede–se b e novamente a,
nessa ordem. Qual é a probabilidade de se obter nessa sequência de medidas os autovalores
b1 (na medida de b) e a1 (na medida de a)?

Q9. Sendo a energia potencial de um sistema quântico unidimensional dada por um poço quadrado
infinito, ½
0, para 0 ≤ x ≤ L ,
V (x) = (1)
∞, em outro caso ,

1) encontre os autovalores da energia e suas respectivas autofunções, indicando as condições


de contorno que estas devem obedecer. OBS.: Não é necessário normalizar as autofunções;
suponha que a constante de normalização de cada estado (n) é conhecida e vale Nn .

A esse sistema é acrescentada uma perturbação da forma:


∆V (x) = a δ(x − L/2) , (2)
onde δ(x − x0 ) é a função delta de Dirac e a uma constante real.

2) Todos os nı́veis de energia são afetados por essa perturbação? Se a resposta for negativa,
o que caracteriza os nı́veis que são e os que não são afetados? Como diferenciá–los?
Explique.
3) Calcule a correção aos nı́veis de energia em primeira ordem em teoria de perturbação.

Q10. Um mol de um gás ideal percorre um ciclo formado por uma expansão adiabática (1 → 2),
uma transformação isobárica (2 → 3) e uma transformação isocórica (3 → 1). Considere dados
V1 , V2 , P3 , CV , γ e R. Em uma transformação adiabática não há troca de calor; em uma
transformação isobárica a pressão P é mantida constante e em uma transformação isocórica o
volume V é mantido constante.

2
1. Esboce o ciclo no diagrama P − V .
2. Determine o calor trocado e o trabalho realizado em cada trecho do ciclo.
3. Ache o rendimento η de um motor que opera segundo esse ciclo em termos de V1 e V2 .
4. Encontre a variação de entropia em cada trecho do ciclo.

3
 

   
 
Q1. Considere uma partı́cula de massa m movendo-se sob a ação do potencial V (x) =
kx2 /2 − kx4 /(4a2 ) onde k e a são constantes positivas. Suponha que o movimento
seja unidimensional e despreze as forças de atrito.

a) Escreva a equação de movimento.


b) Faça um esboço do gráfico de V (x) e descreva os tipos de movimentos possı́veis.
c) Mostre que a função h(x,ẋ) = mẋ2 /2 + V (x) é uma constante do movimento.
d) Encontre a solução x(t) para o caso h = ka2 /4 e x(0) = 0.

Q2. Considere um pêndulo plano formado for uma haste inextensı́vel de comprimento l
e massa desprezı́vel tendo na sua extremidade uma partı́cula pontual de massa m.

a) Escreva as equações de movimento da partı́cula em coordenadas polares r e θ.


b) Suponha que o pêndulo seja lançado de θ(0) = θ0 com θ̇(0) = 0. Calcule o
valor máximo que a tensão na haste atinge durante o movimento.
c) Encontre θ(t) na aproximação de pequenas oscilações supondo θ(0) = θ0 e
θ̇(0) = 0.
d) Esboce um gráfico mostrando como o perı́odo do movimento da partı́cula varia
com a sua energia.

Q3. Considere um oscilador harmônico unidimensional, cujo operador Hamiltoniano é


dado por
1 2 1
H= p + mω 2 x 2 .
2m 2
No instante de tempo t = 0, o sistema se encontra no seguinte estado
1 £ ¤
Ψ(x,0) = √ ψ0 (x) + eiϕ ψ1 (x) ,
2
com ψ0 (x) e ψ1 (x) sendo respectivamente o estado fundamental e o primeiro estado
excitado do Hamiltoniano,
³ mω ´1/4 mω
x2
ψ0 (x) = e− 2~ ,
π~
µ ¶1/4
4m3 ω 3 mω
x2
ψ1 (x) = x e− 2~ ,
π~3
onde ϕ é uma fase.

a) Determine ϕ para que o valor médio da posição seja zero para o estado Ψ(x,0).
b) Determine o valor mais provável da posição no estado Ψ(x,0), empregando o
valor de ϕ determinado acima.
c) Determine o valor médio do momento num instante de tempo qualquer t,
empregando o valor de ϕ determinado acima.

1
Q4. O Hamiltoniano
w¡ 2 ¢
H= Lx − L2y
~
oferece uma boa aproximação para descrever os estados quânticos de um sistema
com momento angular l = 1 colocado num gradiente de campo elétrico. Na ex-
pressão do Hamiltoniano, Lx e Ly são as componentes x e y do operador momento
~ e ω é uma constante real. Os autoestados | − 1i, |0i e | + 1i de Lz
angular orbital L,
com autovalores −~, 0 e +~ formam uma base do espaço de estados desse sistema.

a) Escreva a matriz que representa H na base de Lz citada acima.


b) Encontre os autovalores de H e os correspondentes autovetores na base de Lz
citada acima.
c) Suponha que no instante t = 0 o sistema se encontre no estado
1
|ψ(0)i = √ (| + 1i − | − 1i) .
2
Qual é a probabilidade de se encontrar o valor ~ numa medida de Lz num
instante de tempo posterior t ?

Q5. Uma sonda interestelar afasta-se da Terra com velocidade v = c/3 e a cada um ano-
luz (medido no referencial da sonda) ela emite um sinal luminoso de comprimento
de onda λ0 em direção à Terra. Deseja-se saber,

a) com que periodicidade os sinais chegam à Terra?


b) quanto tempo após o lançamento o primeiro sinal luminoso chega à Terra?
c) onde estará a sonda quando esse sinal chegar à Terra?
d) com que comprimento de onda os sinais são recebidos na Terra?

2
Q6. Um cilindro muito longo de raio R é fabricado com um material isolante cuja cons-
tante dielétrica é K (= ²/²o ) e que possui uma densidade de carga livre cilindrica-
mente simétrica, mas não uniforme ρ(r).

a) Determine ρ(r) tal que o campo elétrico dentro do cilindro seja radial apon-
tando para fora do mesmo e com módulo constante Eo ;
~
b) para a densidade de carga determinada em (a), calcule o campo elétrico E(r)
fora do cilindro;
c) se o cilindro for então envolvido por uma casca cilı́ndrica condutora neutra
de raio interno a (a > R) e raio externo b (b > a), concêntrica ao mesmo,
determine as densidades de carga induzidas nas superfı́cies da casca condutora;
d) para a situação do item (c), esboce um gráfico do módulo do campo elétrico
E(r) em função da distância ao eixo do cilindro, em todo o espaço.

Q7. Uma barra metálica uniforme de massa M pode deslizar com atrito desprezı́vel
ao longo de um par de trilhos horizontais fixos separados por uma distância d,
conforme mostra a figura abaixo. Os trilhos e a ligação transversal da esquerda são
altamente condutores, de modo que suas contribuições para a resistência elétrica do
circuito retangular são desprezı́veis. A barra livre e os contatos com os trilhos fixos
têm resistência elétrica total R. Há um campo magnético uniforme e estacionário
aplicado externamente, de módulo B, orientado verticalmente e apontando para
cima.

a) Determine a corrente i induzida no circuito em termos de d,R,B e v, a veloci-


dade instantânea da barra. Considere como o sentido positivo da corrente na
barra aquele indicado na figura. Ao determinar a corrente induzida, despreze
o campo magnético produzido pela própria corrente;
b) suponha que em t = 0 a barra esteja numa posição xo e com velocidade vo .
Determine x(t) e v(t);
c) obtenha expressões numéricas para x(t), v(t) e i(t) usando os seguintes parâmetros:
M = 0,10 kg, d = 1,0 m, R = 1,0 Ω, B = 0,2 T, xo = 3,0 m e vo = 10 m/s.
Qual a posição final da barra quando ela estiver em repouso?
d) É justificável desprezar no item (a) o campo magnético produzido pela corrente
induzida? Para responder esse item, calcule a razão entre o maior valor do
campo magnético produzido pela corrente induzida (Bi ) e o valor do campo
aplicado (B). Estime Bi calculando o campo magnético na superfı́cie da barra
livre, assumindo que ela é muito longa e tem seção transversal circular com
raio a = 3,0 mm.

d i

x(t)

1
Q8. Considere o seguinte ciclo ABC, para um mol de gás ideal monoatômico: expansão
isotérmica, de pressão inicial p0 para pressão p0 /3, contração isobárica e aqueci-
mento isovolumétrico. No estado inicial, o volume é V0 .

a) Obtenha expressões para a pressão , volume e temperatura, para cada um dos


três estados, A, B e C, em função de p0 e V0 . Obtenha expressões para o
trabalho realizado pelo gás e calor recebido pelo mesmo, nos três processos.
b) Obtenha expressões para o calor especı́fico a volume constante, cV , e para o
calor especı́fico a pressão constante, cp . Explique a origem fı́sica da diferença
entre as duas grandezas.
c) Obtenha uma expressão para a entropia do gás, S, como função da temperatura
e do volume do mesmo. Justifique.
d) Esboce um diagrama temperatura X entropia para o ciclo acima, utilizando o
resultado anterior. Justifique.

Q9. Considere N partı́culas de massa m, não interagentes, em equilı́brio térmico à tem-


peratura T , em movimento unidimensional em uma caixa de comprimento L.

a) Escreva a função de partição clássica para este sistema, Z(T,L,N ). Justifique.


b) Obtenha a energia livre de Helmholtz F (T,L,N ) e a entropia S(T,L,N ) do
sistema.
c) Demonstre, em geral, que a transformada de Legendre da energia interna U (S)
com relação à entropia S é dada por F = U − T S. Utilize esta relação para
obter uma expressão para a energia interna U do sistema em estudo, como
função da temperatura T . Comente o resultado.
d) Esboce gráficos de energia interna U e da entropia S em função da tempe-
ratura T . Obtenha o calor especı́fico a volume ”L”constante, cL , e discuta a
compatibilidade do resultado obtido com os dois gráficos esboçados, de U (T )
e de S(T ).

Q10. Quando Bohr desenvolveu sua teoria atômica procurou-se dar respaldo a mesma
achando-se situações em que ela concordava com resultados experimentais. Consi-
deraremos três dessas aqui, para um átomo monoeletrônico com massa nuclear M
finita e número atômico Z. Para isso,

a) deduza a expressão (em função das constantes e, m, h, etc) da energia En dos


nı́veis quantizados, que sabemos reproduz as linhas principais do espectro de
átomos monoeletrônicos;
b) calcule a razão En (He+ )/En (H), entre as energias dos nı́veis eletrônicos do
hélio uma vez ionizado (He+, Z=2 , M=4 u.a.) para aquelas do hidrogênio
(H, Z=1, M=1 u.a.) e
c) mostre que para um número quântico n muito grande, a frequência da luz
emitida na transição n → n − 1 coincide com a frequência clássica de revolução
do elétron no n-ésimo estado. Isso mostra que a teoria obedece o princı́pio de
correspondência.

2
T1. Um mol de um gás ideal, desde um estado inicial a temperatura T0 e pressão [4,0]
P0 , é submetido a (i) uma compressão adiabática, seguida de (ii) um resfriamento
isocórico (volume constante). Ao final das transformações, a temperatura é a mesma
do estado inicial, Tf = T0 , e a pressão foi aumentada por um fator f > 1, Pf = f P0 .
Trate os dois processos como quase estáticos e considere conhecidos os parâmetros
e relações do gás ideal dados no formulário.

a) Represente esquematicamente as transformações num diagrama P V . Deter-


mine os valores da pressão, volume e temperatura do gás nos pontos terminais
dos processos (i) e (ii).
b) Calcule o calor trocado e o trabalho realizado sobre gás no processo.
c) Determine a variação da entropia do gás. Represente as transformações num
diagrama T S.

T2. Para os três sistemas listados a seguir o termo dominante da capacidade térmica [3,0]
molar, em torno da temperatura ambiente, tem a forma

C = AT b .

Para cada um dos três sistemas seguintes:

a) um gás ideal diatômico (clássico),


b) um sólido isolante e
c) um metal,

i. explique sucintamente a origem fı́sica desta capacidade térmica e dê os valores


da constante A e do expoente b;
ii. indique a forma que o termo dominante da capacidade térmica assume no
limite de baixas temperaturas, explicando as alterações, se houver.

T3. Um sistema é constituı́do de N partı́culas localizadas e não interagentes. [3,0]


Cada partı́cula pode ser encontrada em três estados 1, 2 e 3, com energias 1 = 0,
2 = 3 = ∆ > 0. O sistema está em equilı́brio a uma temperatura absoluta T .

a) Determine a função de partição do sistema de N partı́culas.


b) Calcule a energia do sistema por partı́cula. Faça um gráfico do resultado em
função da temperatura indicando os valores limites para T → 0 e T → ∞.
c) Obtenha a entropia do sistema por partı́cula. Faça um gráfico do resultado
em função da temperatura indicando os valores limites para T → 0 e T → ∞.
d) Interprete os resultados limite dos ı́tens b e c em termos das probabilidades de
ocupação dos estados.

1
C1. A massa m de um pêndulo está presa por um fio ideal de comprimento l a um [3,5]
ponto de sustentação. Esse ponto se move para a frente e para trás ao longo de um
eixo horizontal, de acordo com a equação x = a cos ωt. Suponha que o pêndulo só
oscile no plano vertical que contém o eixo x. Considere que a posição do pêndulo
seja descrita por um ângulo θ que o fio faz com uma linha vertical.

a) Escreva a Lagrangeana e obtenha as equações de Lagrange.


b) Mostre que, para valores pequenos de θ, a equação de movimento se reduz à
equação de movimento de um oscilador harmônico forçado.
c) Determine o movimento para o estado estacionário correspondente ao ı́tem (b)
encontrando a amplitude de oscilações do estado estacionário.

C2. Uma partı́cula de massa m se move num poço de potencial U (x) = a ln(x) + b/x2 , [3,5]
onde x é a distância da partı́cula ao centro de forças e a e b são constantes positivas.
Considere apenas x > 0.

a) Qual a força que age sobre a partı́cula? Esboce os gráficos de F (x) e de U (x).
b) Quais os pontos de equilı́brio e quais as caracterı́sticas desses pontos? Quais
os possı́veis movimentos da partı́cula?
c) Se houver pontos de equilı́brio estável, calcule o perı́odo de pequenas oscilações
em torno desses pontos.

C3. Um cometa de massa m descreve uma órbita hiperbólica em torno do Sol (massa [3,0]
M ) e quando está a uma distância r0 se aproximando do Sol, a sua velocidade é v0
e faz um ângulo de 30◦ com o raio vetor ao Sol.

a) Calcule o momento angular e a energia desse cometa.


b) Determine a distância rp de máxima aproximação do cometa ao Sol.
c) Quando o cometa atinge a distância rp de máxima aproximação, sofre um
choque com um pequeno asteróide de tal maneira que sua massa não varia
porém ele passa a descrever órbita circular de raio rp no mesmo plano da
órbita anterior. Calcule a nova energia e o novo momento angular após a
colisão.

2
M1. A figura abaixo corresponde à radiância espectral de dois corpos mantidos a tem- [2,5]
peraturas T1 e T2 . Vamos supor que a emissão de radiação desses corpos é igual a
de corpos negros nessas temperaturas.
5

a) Sabendo que a temperatura do corpo


1 é T1 = 2898 K, determine a tempe- T

Radiância (unidades arbitrárias)


4 2

ratura do corpo 2.
b) Qual é a razão entre as potências 3

totais irradiadas pelos corpos, ou


seja, quanto vale R2 /R1 , onde R é a 2

radiância total?
c) Segundo a teoria clássica da emissão 1
T 1

de radiação do corpo negro, que as-


pectos das curvas ao lado não podem 0
0 500 1000 1500 2000 2500 3000
ser explicados? Como Planck resol-
comprimento de onda (nm)
veu esse problema?
M2. Uma espaçonave de comprimento próprio L = 300 m está passando por uma estação [2,5]
transmissora fixa na Terra, a uma velocidade v = 0,6c. No instante em que o nariz
da espaçonave passa pelo transmissor, dois relógios, um no transmissor e outro no
nariz da espaçonave são sincronizados para t = t0 = 0. No momento em que a
cauda da espaçonave passa pelo transmissor, um sinal é enviado pelo transmissor e
subsequentemente detectado por um receptor no nariz da espaçonave.

a) Em que instante o sinal foi enviado, segundo o relógio no nariz da espaçonave?


b) Em que instante o sinal foi recebido, segundo o relógio de bordo?
c) Em que instante o sinal foi recebido na espaçonave, segundo o relógio do trans-
missor?
d) Onde estava o nariz da espaçonave, no instante em que o sinal foi recebido, do
ponto de vista de um observador fixo na posição do transmissor.

3
M3. Um átomo de hidrogênio que se encontra no estado fundamental (isto é, de menor [2,5]
energia) é excitado até o estado com n = 4.

a) Calcule a energia absorvida pelo átomo.


b) Calcule as energias dos diferentes fótons que podem ser emitidos quando o
átomo voltar para o estado fundamental. Faça um diagrama de nı́veis de
energia, indicando essas transições.
c) Calcule a velocidade de recuo do átomo de hidrogênio, ao fazer a transição do
estado n = 4 diretamente para o estado fundamental. Suponha o átomo em
repouso, antes da transição ocorrer.

M4. A função de distribuição de velocidades de um grupo de N partı́culas é dada por [2,5]


dNv = av dv onde dNv é o número de partı́culas que tem velocidades entre v e
v + dv, e a é uma constante. Nenhuma partı́cula tem velocidade maior que V ,
sendo que as velocidades podem variar entre 0 e esse valor máximo, V .

a) Esboce o gráfico da função de distribuição, ou seja g(v) = dNv /dv em função


de v.
b) Calcule o valor da constante a em termos de N e V .
c) Calcule a velocidade média, a velocidade quadrática média e a velocidade mais
provável em termos de V .
d) Qual porcentagem das partı́culas tem velocidades entre a velocidade média e
V ? E entre a velocidade quadrática média e V ?

Q1. Uma partı́cula de massa m sujeita a um potencial unidimensional [3,0]


(
0, 0<x<L
V (x) =
∞, outros pontos

está no instante t = 0 no estado


(
1+i
q2 q
2 L
sen πx
L
+ √1
2
2
L
sen 2πx
L
, 0<x<L
ψ(x) =
0, outros pontos.

a) Determine o estado ψ(x,t) no instante de tempo t.


b) Determine o valor médio da energia hEi.
c) Determine a freqüência de variação do valor médio da coordenada hxi.

Q2. Considere a perturbação Ĥ1 = bx4 ao oscilador harmônico simples, cujo hamilto- [4,0]
niano não perturbado é
p̂2x 1
Ĥ0 = + mω 2 x2 .
2m 2
O hamiltoniano Ĥ0 + Ĥ1 descreve um oscilador com uma força restauradora não
linear.

a) Expresse Ĥ1 em termos dos operadores â e ↠.


b) Use as propriedades dos operadores â e ↠para obter a correção de primeira
(1)
ordem para o n-ésimo nı́vel de energia, En .
(1) (0)
c) Calcule En /En no limite n → ∞.

1
Q3. A função de onda de uma partı́cula sujeita a um potencial com simetria esférica é [3,0]
dada por
ψ(~r) = (x + y + 3z)f (r).

a) Expresse ψ(r) em termos de harmônicos esféricos.


b) ψ é auto-função de L2 ? Em caso afirmativo, qual é o valor de l? Em caso
negativo, quais são os possı́veis valores de l quando L2 é medido?
c) Quais são as probabilidades de se encontrar a partı́cula com os possı́veis valores
de ml ?

E1. Considere uma distribuição esférica cargas, com densidade volumétrica de cargas [3,0]
constante, de raio Re e carga total e.

a) Determine o vetor campo elétrico E em todo o espaço (dentro e fora da distri-


buição esférica).
b) Calcule a energia total associada a esta distribuição de cargas e mostre que:

3 e2
U= .
5 4π0 Re
c) Supor que a energia calculada no ı́tem anterior corresponde à massa (me ) de
repouso do elétron. Calcule a expressão que fornece o raio do elétron e faça
uma estimativa do seu valor, em metros.

E2. Considere um fio reto e infinito, pelo qual flui uma corrente I constante. Calcule [3,0]
o vetor campo magnético B em um ponto P , à distância ρ do fio:
a) utilizando a lei de Ampère:
I
B · d~` = µ0 I; z

b) utilizando a lei de Biot-Savart:

µ0 I d~` × êr
dB = ;
4π r2 I
P
c) através do cálculo do potencial vetor: ρ
Z
µ0 JdV
A= .
4π r

Lembre-se que JdV ↔ Id~`.


Sugestão: Suponha um fio de comprimento L muito
grande, de forma que L  ρ.

2
E3. Foi idealizado um experimento para o estudo da polarização de ondas eletro- [4,0]
magnéticas por reflexão em uma superfı́cie dielétrica plana. Uma onda não po-
larizada pode ser considerada como tendo componentes do campo elétrico paralela
(componente p) e não paralela (componente s) à superfı́cie do dielétrico, de forma
que os coeficientes de Fresnel correspondentes são:

n1 cos θ1 − n2 cos θ2 sen(θ2 − θ1 )


r12p = = ;
n1 cos θ1 + n2 cos θ2 sen(θ2 + θ1 )

n2 cos θ1 − n1 cos θ2 tg(θ1 − θ2 )


r12s = = .
n2 cos θ1 + n1 cos θ2 tg(θ1 + θ2 )

a) Notando que a polarização ocorre quando θ1 (θ1 = θB ) satisfaz a condição


θ1 + θ2 = π/2, mostre que tg θB = n2 /n1 e que, nesta condição, o coeficiente
de Fresnel da componente da onda que não se anula pode ser escrito como:

1 − n22
r12 = .
1 + n22

b) No experimento, um painel de polieti-


leno e um conjunto emissor/receptor de
micro-ondas foi utilizado, conforme mos-
trado na figura ao lado. O ângulo de
incidência foi sendo variado e as inten-
sidades relativas Ip /I0 e Is /I0 da onda
correspondentes às componentes p e s,
foram registradas na tabela abaixo.
Ângulo Leituras dos medidores Ângulo Leituras dos medidores
(θ1 ) Is /I0 Ip /I0 (θ1 ) Is /I0 Ip /I0
20◦ 0,41 ± 0,04 0,36 ± 0,04 50◦ 0,10 ± 0,04 0,17 ± 0,04
25◦ 0,28 ± 0,04 0,34 ± 0,04 55◦ 0,05 ± 0,04 0,14 ± 0,04
30◦ 0,26 ± 0,04 0,28 ± 0,04 60◦ 0,03 ± 0,04 0,19 ± 0,04
35◦ 0,24 ± 0,04 0,21 ± 0,04 65◦ 0,04 ± 0,04 0,23 ± 0,04
40◦ 0,15 ± 0,04 0,19 ± 0,04 70◦ 0,19 ± 0,04 0,48 ± 0,04
45◦ 0,11 ± 0,04 0,20 ± 0,04 75◦ 0,44 ± 0,04 0,65 ± 0,04
Com os dados da tabela construa o gráfico adequado e, a partir deste, deter-
mine o ângulo de Brewster θB .
c) Determine o ı́ndice de refração do painel de polietileno utilizado no experi-
mento. Calcule a refletância R da onda quando o ângulo de incidência da
onda corresponde ao ângulo de Brewster e compare com o valor experimental.
d) Como você relaciona este fenômeno de polarização com a utilização de óculos
especı́ficos por pessoas que vão à praia, que esquiam na neve, etc.? Explique!

3
E1. Uma onda plana uniforme incide, do vácuo, em
uma placa de vidro que possui ı́ndice de refração
n2 =1,5. O campo elétrico desta onda é:
~ i = 4 cos 4,0π×106 z − 1,2π×1015 t êx

E

onde todas as grandezas são expressas no SI. As


trajetórias dos raios luminosos correspondentes
(incidente, refletido e transmitido) são mostradas
na figura ao lado.
(a) Transcreva a figura no seu caderno de resposta e, para cada onda, desenhe o
campo magnético B ~ e o vetor de onda ~k correspondentes (preocupe-se apenas
com a direção e sentido dos vetores).
(b) Determine a velocidade de propagação, o comprimento de onda e a freqüência
das ondas incidente, refletida e transmitida.
(c) A partir de uma das equações de Maxwell determine o campo magnético da
onda incidente e também o vetor de Poynting correspondente.

E2. Considere um capacitor plano, de pla-


cas paralelas muito grandes, no vácuo,
com densidades de carga +σ e −σ, que
encontra-se em repouso no referencial do
laboratório.

(a) Qual é o campo elétrico e o campo magnético no interior do capacitor, medidos


por um observador que encontra-se no referencial do laboratório (em repouso
em relação ao capacitor)?
(b) Calcule os vetores campo elétrico e magnético no interior do capacitor, medidos
por um observador que encontra-se em um referencial S0 (ver figura acima),
movendo-se ao longo do eixo x com velocidade V constante.
(c) Supondo agora o observador movendo-se ao longo do eixo z (perpendicular-
mente aos planos das placas), com velocidade constante V , calcule novamente
os vetores campo elétrico e magnético no interior do capacitor.

E3. Em uma experiência de laboratório um estudante,


para determinar a constante de tempo τ = RC
de um capacitor comercial C = 1,0 µF, montou o
circuito da figura abaixo, utilizando um resistor
de 10 MΩ e uma pilha de 1,5 V. Assim, quando
a chave S1 é fechada e a S2 aberta, o capacitor é
carregado até a máxima tensão. Em t = 0 s, a
chave S1 é aberta e a S2 é fechada, descarregando
o capacitor.
(a) Para o processo de descarga, encontre a equação do circuito, resolvendo-a em
Q, e mostre que V = V0 exp(−t/RC) corresponde à expressão que fornece a
tensão no capacitor em função do tempo.

1
(b) Para realizar a medida de τ =RC o estudante optou por medir o tempo (t1/2 )
que leva para que a tensão no capacitor seja exatamente a metade da tensão
inicial. Para isto, ele utilizou um multı́metro e um cronômetro manual. O pro-
cedimento foi realizado 10 vezes e os resultados estão representados na tabela
abaixo.

t1/2 (s) 8,12 8,12 8,15 8,11 8,13 8,14 8,13 8,15 8,12 8,14

Após calcular o valor médio utilizando uma calculadora, o estudante escreveu


em seu relatório que ht1/2 i = 8,131 ± 0,0137 s.
Você concorda com a forma com que ele expressou o valor o valor médio de
t1/2 ? Justifique. Se não concorda, como você expressaria o valor do tempo
médio medido da maneira correta?
(c) A partir do valor arredondado de ht1/2 i=8 s (para uma rápida avaliação dos
resultados), determine o valor experimental da constante de tempo τexp e com-
pare com o seu valor nominal τnominal .
(d) Construa um gráfico da tensão (eixo y) em função do tempo (eixo x) que repre-
sente a descarga do capacitor. Mostre neste gráfico, de uma maneira aproximada,
os dados mais representativos da experiência (V0 , τ , t1/2 ).

Q1. Um feixe de partı́culas com spin 1/2 incide a partir da esquerda no sistema exibido
na figura abaixo. Após passar por A, as partı́culas encontram-se no estado de spin
associado ao autovalor +~/2 de Sz .

A B

Responda justificando:
(a) Se o aparato B mede a componente y do spin, quais são os resultados possı́veis
que B pode fornecer?
(b) Qual a probabilidade de B fornecer como resultado ~/2?

Q2. Considere os operadores associados ao momento angular orbital Lx , Ly e Lz . Supo-


nha que o estado do sistema é dado por
 
Ψ = N (1 − i)Y3,1 + iY2,1 + 2Y1,1 .

onde N é uma constante e Ylm são os harmônicos esféricos propriamente normaliza-


dos.
(a) Encontre N para que Ψ esteja normalizada.
(b) Este estado é autovetor de L2 ? Se é, qual é o autovalor correspondente? Se
não, qual é o valor esperado (médio) de L2 ?
(c) Qual a incerteza de Lz neste estado?
(d) Qual o valor esperado de Ly ?

2
Q3. Considere os estados de um elétron numa molécula diatômica formada pelos átomos
E e D onde a distância ED é igual a d. Denotamos por |ψE i e |ψD i os autovetores
de um observável B o qual corresponde ao elétron estar localizado na vizinhança
dos átomos E ou D respectivamente:
d d
B|ψE i = − |ψE i, B|ψD i = + |ψD i.
2 2
O hamiltoniano do sistema na base {|ψE i, |ψD i} é dado por
 
E0 −a
H=
−a E0

onde a > 0.
(a) Quais os valores possı́veis da energia do sistema?
(b) Qual é a probabilidade de encontrar o elétron na vizinhança de E se o sistema
está no estado fundamental?
(c) Considere que o elétron está no estado |ψE i para t = 0. Obtenha o estado do
sistema como função do tempo.

Q4. Considere um rotor quântico com hamiltoniana

~2 ∂ 2
H=− + λV (ϕ) ,
2I ∂ϕ2

onde V (ϕ) = sin(2ϕ).


(a) Escreva os nı́veis de energia e autofunções não perturbados do sistema (λ = 0).
Discuta suas degenerescências. Use a condição de contorno Ψ(ϕ) = Ψ(ϕ + 2π).
(b) Calcule a correção da energia do estado fundamental na mais baixa ordem não
nula de perturbação.
(c) Qual a correção para a função de onda?

M3. Um átomo de hidrogênio está em seu primeiro estado excitado (n = 2). Usando
o modelo de Bohr para o átomo, calcule as grandezas abaixo, expressando seus
resultados nas unidades que achar conveniente:
(a) o raio da órbita do elétron,
(b) o momento linear do elétron,
(c) o momento angular do elétron,
(d) sua energia cinética,
(e) sua energia potencial.

M4. A vida-média própria de um pı́on é 3,6×10−8 s. Se um feixe de pı́ons tem velocidade


de 0,8c:
(a) qual é a vida-média das partı́culas no referencial do laboratório?
(b) que distância elas percorrem, em média, antes de se desintegrarem?
(c) qual seria a resposta do ı́tem b) se não existissem efeitos relativı́sticos?

3
E1. Considere um anel isolante de raio a com massa M e carga total Q uniformemente
distribuı́das. O anel repousa sobre um plano horizontal sobre o qual pode se mover
livremente sem atrito. Na região do anel há um campo magnético devido a uma
fonte externa dado por:

B0 êz , se t < 0,
B(r,t) = −αt
B0 êz e , se t ≥ 0,
onde B0 e α são constantes positivas. O plano em que repousa o anel é o plano xy
e o centro do anel coincide com a origem do sistema de coordenadas. No instante
t = 0 o anel encontra-se parado.

(a) Determine o campo elétrico sobre o anel como função do tempo.


(b) Encontre a velocidade angular do anel num instante t > 0.
Desconsidere o campo produzido pelo anel em movimento.
(c) Há conservação de momento angular neste processo? Explique.

E2. Uma placa condutora aterrada se encontra no plano xy (z = 0). Uma carga q é
trazida até o ponto dêz , com d > 0. Determine:

(a) o potencial eletrostático em todo o espaço;


(b) o campo elétrico em todo o espaço;
(c) a densidade de carga e a carga total induzida na superfı́cie do condutor;
(d) o trabalho realizado para trazer a carga q do infinito até o ponto dêz .

Q1. Considere, para uma partı́cula de massa m em uma dimensão, a função de onda
 n
x
ψ(x) = A e−x/x0 ,
x0
onde A, n e x0 são constantes. Encontre o potencial V (x) e a energia E tais que essa
função seja uma auto-função da equação de Schrödinger. Considere que V (x) → 0
para x → 0.

Q2. O primeiro estado excitado de um oscilador harmônico isotrópico de freqüência


natural ω0 , massa m e Hamiltoniano H0 , em 3-D, é triplamente degenerado.

(a) Calcule o desdobramento desses nı́veis devido à perturbação


H 0 = b(xy + yz), sendo b uma constante, em primeira ordem de perturbação.
(b) Determine, em função dos autoestados do oscilador harmônico 3-D não per-
turbado, os autoestados associados aos nı́veis perturbados.
Sugestão: Utilize a representação de número de ocupação |nx ,ny ,nz i, tal que

H0 |nx ,ny ,nz i = (nx + ny + nz + 23 )h̄ω0 |nx ,ny ,nz i.

Dado: Para estados de um oscilador harmônico em 1-D, temos a seguinte relação:


s
(n + 1)h̄
hn|xi |n + 1i = , xi = x,y,z.
2mω0

1
M1. Um átomo de deutério, que se encontra inicialmente em repouso no estado n = 3,
faz transição para o estado fundamental, n = 1.

(a) Mostre que a velocidade de recuo do átomo devida à emissão do fóton é dada
aproximadamente por v = 8hR/9M , onde R é a constante de Rydberg e M a
massa do átomo.
(b) Estime a porcentagem da energia da transição 3 → 1 que é carregada pelo
átomo de deutério em recuo. (Dê o resultado com 1 algarismo significativo.)

M2. Uma partı́cula de massa de repouso M0 se move com velocidade V = V x̂ tal que
sua energia cinética é de 41 M0 c2 no referencial do laboratório. Num determinado
3
instante, ela se desintegra em duas partı́culas idênticas de massa m0 = 10 M0 que se
movem paralelamente ao eixo x. Determine:

(a) A velocidade V da partı́cula M0 , antes do decaimento, no sistema do labo-


ratório.
(b) As velocidades u01 e u02 das partı́culas de massa m0 no referencial do seu centro
de massa (CM).
(c) As velocidades u1 e u2 das duas partı́culas no referencial do laboratório.
(d) Repita os cálculos do ı́tem (c), considerando agora que no referencial do CM
as partı́culas são emitidas com velocidades ao longo do eixo y 0 .

C1. Uma partı́cula e uma barra podem se mover livremente sobre a superfı́cie lisa de uma
mesa horizontal. A barra tem comprimento D e massa igual à massa da partı́cula,
M . Inicialmente a barra está em repouso, e a partı́cula se move com velocidade v
perpendicular à barra. Suponha que a partı́cula colide com a barra e que a colisão
é elástica.

(a) Descreva os movimentos subseqüentes da partı́cula e da barra quando a partı́cula


colide com a barra no seu centro de massa (ver figura a) abaixo).
(b) Descreva os movimentos subseqüentes da partı́cula e da barra quando a partı́cula
colide com a barra numa das suas extremidades (ver figura b) abaixo).
(c) No caso do item (b), determine as velocidades da partı́cula, vp , da barra, vb , e
a velocidade angular de rotação da barra em torno do seu centro de massa, w.

Momento de inércia da barra relativo a


Dado: um eixo passando pelo centro de massa
e perpendicular à barra: I = M D2 /12.

1
C2. Um objeto de massa m desliza num tri-
lho liso mostrado na figura ao lado. Ini-
cialmente o objeto está em repouso, a
uma altura h acima do topo do semi-
cı́rculo AC.
(a) Faça um diagrama das forças que agem no objeto quando ele está no ponto B
do semi-cı́rculo. Defina θ como o ângulo do vetor posição medido em relação à
~ Escreva as equações de movimento nas direções radial e tangencial.
direção OA.
(b) Qual é a magnitude e direção da força exercida no objeto pelo trilho, quando
ele passa no ponto A ?
(c) Mostre que h ≥ r/2, para que o objeto atinja o ponto C do trilho.
(d) Para h < r/2 o objeto abandona o trilho antes de atingir o ponto C. Mostre
que isto ocorre na posição tal que −3 cos θ = 2 + 2h/r.
T1. Considere o ciclo de Carnot, composto de duas etapas isotérmicas, 1 → 2 e 3 → 4,
e duas adiabáticas, 2 → 3 e 4 → 1.
(a) O ciclo de Carnot, se realizável, seria o ciclo de maior eficiência possı́vel
(eficiência = trabalho obtido/calor fornecido). Justifique qualitativamente por-
que a escolha de processos adiabáticos e isotérmicos maximiza a eficiência de
uma máquina térmica.
(b) Represente qualitativamente o ciclo em diagramas p-V (pressão-volume) e T -S
(temperatura-entropia), identificando os estados iniciais e finais de cada pro-
cesso. Utilizando o diagrama T -S, obtenha a eficiência do ciclo, em função das
temperaturas dos banhos quente e frio, Tq e Tf .
T2. Considere uma fita de borracha que tem como equações de estado:
bT (L − L0 )
U = cL0 T, e τ = ,
L1 − L 0
onde U é a energia interna, T a temperatura, τ a tensão e L o comprimento da
fita. Os parâmetros b, L0 e L1 são constantes positivas, com L1 > L0 . A tensão
τ desempenha o papel da pressão negativa (τ → −P ) e o comprimento da fita L
desempenha o papel do volume (L → V ), de modo que o trabalho sobre a fita é
dado por τ dL.
(a) Obtenha a equação fundamental para a entropia da fita S(U,L).
(b) Obtenha o coeficiente de expansão térmica (α= L1 ∂T
∂L

τ
) e a capacidade térmica
∂S

da fita, (cτ =T ∂T τ ), ambas a tensão constante.
(c) Compare qualitativamente o comportamento térmico da fita, sob tensão cons-
tante, descrito por seu coeficiente de expansão térmica, determinado no ı́tem
anterior, com o comportamento esperado para um gás, no aquecimento à
pressão constante. Que razão fı́sica você daria para esta diferença de com-
portamento?

2
E3. As propriedades eletromagnéticas da ionosfera terrestre podem ser descritas por
uma permeabilidade magnética µ = µ0 e uma constante dielétrica  dependente da
freqüência (angular) ω na forma

ω02
 
(ω) = 0 1 − 2 .
ω

O parâmetro ω0 é determinado pela composição da ionosfera. Considere uma onda


plana numa determinada região da ionosfera cujo campo elétrico é dado por

E = E0 ei(kz−ωt) .

(a) Obtenha a relação de dispersão k(ω).


(b) Para que valores de ω uma onda eletromagnética propaga neste meio?
(c) Qual a velocidade de fase vf de uma onda eletromagnética neste meio?
(d) É possı́vel que vf seja maior que a velocidade da luz no vácuo c? Explique.
(e) Qual a velocidade de grupo vg desta onda? Esta velocidade pode ser maior que
c?

E4. Considere um cabo coaxial formado por duas cascas cilı́ndricas condutoras de raios
a e b (b > a). Um material de permeabilidade magnética µ ocupa o espaço entre
as cascas cilı́ndricas, que são percorridas por uma corrente constante I ao longo de
seu comprimento como mostra a figura abaixo. Determine:

(a) O campo H em todo espaço;


(b) o campo magnético B em todo espaço;
(c) a magnetização M em todo espaço;
(d) as correntes de magnetização em todo o espaço;
(e) a energia armazenada no cabo por unidade de comprimento.

1
Q3. Considere o átomo de hélio (He).
(a) Escreva o seu hamiltoniano ĤHe , tratando o núcleo como uma carga pontual
de massa infinita.
(b) Considere o hamiltoniano do He sem o termo repulsivo intereletrônico como
um hamiltoniano de ordem zero Ĥ0 . Utilizando resultados para o estado fun-
damental de um átomo hidrogenóide,
 12
Z3 Z 2 e2

ψ1s (r) = exp (−Zr/a0 ) , E1s = −
πa30 2a0
determine a autofunção ψ0 (r1 ,r2 ) e a autoenergia E0 correspondentes ao estado
fundamental de Ĥ0 . Compare a autoenergia com o resultado experimental para
o átomo de He, Eexp = −78,8 eV.
(c) Escreva uma função de estado aproximada para o átomo de He, ψHe (r1 ms1 ,r2 ms2 ),
a partir da autofunção do ı́tem anterior e das funções de spin χ+ e χ− corres-
pondendo às componentes ms = + 21 e ms = − 21 , respectivamente.
(d) Obtenha uma melhor aproximação para a energia do estado fundamental do
He considerando o termo repulsivo descartado acima como uma perturbação e
utilizando teoria de perturbação de primeira ordem.
Dados:
2 Z Z
Z3 e2 3 3 5Ze2

e−2Z(r1 +r2 )/a0 d r1 d r 2 = ; r12 ≡ |r1 − r2 |
πa30 r12 8a0
h̄2 e2
Raio de Bohr: a0 = ; = 13,6 eV
me e2 2a0
Q4. Considere um sistema quântico descrito por um espaço vetorial de duas dimensões
gerado por dois vetores de base ortonormais |1i e |2i. Seja Ĥ o hamiltoniano do
sistema, cujos elementos de matriz são

h1|Ĥ|1i = h2|Ĥ|2i = a
h1|Ĥ|2i = h2|Ĥ|1i = b.

Considere agora um outro observável Ŝ cujos elementos de matriz são

h1|Ŝ|1i = 1
h2|Ŝ|2i = −1
h1|Ŝ|2i = h2|Ŝ|1i = 0.

(a) Quais são os autovalores e autovetores de Ĥ?


(b) Suponha que o estado do sistema seja |ψi = √1 (|1i + |2i). Quais os possı́veis
2
resultados, com suas respectivas probabilidades, das medidas de Ĥ e Ŝ?
(c) Suponha que em t = 0 o sistema esteja no estado |ψ(0)i = |1i. Qual o estado
para um tempo t, |ψ(t)i? Para quais instantes de tempo uma medida de Ŝ
fornece o valor −1 com 100% de probabilidade?
(d) Os operadores Ŝ e Ĥ podem ser diagonalizados simultaneamente? Justifique.

2
M3. Mostre que o comprimento de onda de de Broglie de uma partı́cula de carga e e massa
de repouso m0 , acelerada a partir do repouso e adquirindo velocidades relativı́sticas,
é dada como função do potencial acelerador V pela expressão
 −1/2
h eV
λ= √ 1+ .
2m0 eV 2m0 c2

M4. Um feixe fino de 1,0×106 partı́culas α por segundo, com energia de 5,0 MeV, incide
normalmente num alvo de Cu (Z = 29, A = 60, densidade 9 g/cm3 ) de 1,0×10−4 cm
de espessura. As partı́culas espalhadas coulombianamente são observadas numa tela
fluorescente de 4×4 mm2 , colocada a 10 cm do centro do alvo numa direção fazendo
um ângulo de 60◦ com a do feixe incidente. (Este foi um dos casos estudados
por Geiger e Marsden.) Nos ı́tens abaixo utilize apenas um ou dois algarismos
significativos.

Dados: Espalhamento de Rutherford


2
zZe2 1

nI
dN = dΩ
4π0 4E sen4 (θ/2)

(a) Determine o valor da densidade de átomos de cobre por unidade de área no


alvo.
zZe2
(b) Qual é a dimensão do parâmetro 16π0 E
? Calcule o seu valor.
(c) Calcule o número de cintilações por minuto observadas na tela.

C3. Um plano inclinado de um ângulo α é acelerado horizontalmente. A magnitude da


aceleração aumenta gradualmente até que um bloco de massa m, originalmente em
equilı́brio com respeito ao plano inclinado, começa a subir no plano. O coeficiente
de atrito estático entre o bloco e o plano é µ = 5/4.

(a) Desenhe um diagrama mostrando as forças que atuam no bloco, pouco antes
dele subir no plano, visto de um referencial inercial.
(b) Ache a aceleração do plano quando o bloco começa a subir.
(c) Repita o item (a) visto de um referencial não inercial, fixo no plano.
Dado: cos α = 0,8; sen α = 0,6

1
C4. Uma partı́cula de massa igual a m se move no interior de um cano liso. O cano, por
sua vez, gira num plano horizontal com velocidade angular w constante em torno
de um ponto fixo no cano.

(a) Quantos graus de liberdade tem a partı́cula?


(b) Considere como coordenada generalizada a posição da partı́cul ao longo do
cano, s, com a origem no centro de rotação. Mostre que a lagrageana do
sistema é dada por:
1
L = m(ṡ2 + w2 s2 )
2
(c) Escreva a equação de Lagrange. Existe um ponto de equilı́brio? Ele é estável?
(d) Determine a força de reação do cano.Do ponto de vista de um observador fixo
no cano, qual é a origem da força de reação?

T3. (a) Descreva o modelo de gás ideal para elétrons de condução de um metal e
explique o significado da energia de Fermi. Justifique qualitativamente porque
esta energia deve depender do volume e do número de partı́culas e do spin do
elétron.
(b) Obtenha a expressão para a energia de Fermi a temperatura nula como função
da densidade de elétrons.

T4. (a) Vários sólidos apresentam um calor especı́fico molar à temperatura ambiente
dado por c = 3R, onde R é a constante dos gases. Considere o ensemble
canônico e demonstre que um conjunto de osciladores harmônicos clássicos in-
dependentes constitui um bom modelo para descrever esta propriedade térmica.
(b) Utilize sua dedução para demonstrar o teorema da equipartição da energia
para um sistema qualquer com f graus de liberdade (energia quadrática na
coordenada ou no momento).

2
¥ 
  
   
   
             "!# $
!;:

% &('*)+'(,,-!. / 0 
 1 (2-3547698
< "=> @?  >( 
'
A B C >D "E@C  F &>G HB I =  J CKK @ L@  #  ?  H 
6
 C 7M  FBHND O     P P Q;"R  " "J? C
A  Q;TS 
 "D AU C C7V7"@ ?   XWY3[Z V˜
\ ]^#B> D % & > E@ C      > C?  > 
  
    BK   C  "D  C B" C  =  


H  `_)aW.T3
G
6
N-bKc;d
eHf W
N2g
\ A]"h9 C" @#IKK jk i )mlHnonCR npW.q B C   ( 
  \ C
 $+
 jI r "  
 sC " E@ t A ]
\  ]"    u
Hv      " C B       B
i )mlnConR n7W.t" @B  t " 
 
K C @"IK w>

x C  "y x

z O{* 
HO?=KK  O   t O O  KK?=KK  O
  OI r   =T  
 |     t S}  -˜


f f fq
w>
i ) i~ npW.T3PCZ€H‚B) „
ƒ …*† ‡ˆ
ƒ … wˆ‰ŠƒR .C‹ Œ )+Ž „ ‡ R W.

hK C"  <˜


\ ]"u  % &

? % & "" 
C  BK  
\ A]>{I  t " Bw ‰

z

79L    > "C 
" I">C
 CEmL  " - @L  >  
N A% & "?  I   OO  E@%   B  UV7  C? 
UK   =

 
/3 ~ : n
†
  ~ =" 1   "
 KL    "  B" E@% t

= " 7B

C K I "{P  B E %  7Kr>   @sC # Q;C @9C  > @ @$
 #" C" Q;> "C @9C > r#>  % &>I 79   \ I J?# A  Q; T 
    % &" "?  I     jC BK  
7K R] 

\ ]^# >D * ?  ?  ( (


7KL    (       " @@L  C  
 -         -C @?  0  B> E@%   Q;

R"" C % & "I 79  H=  
H  V˜
q3  )+N : : 
… R : .
f
 R f
)+R : … N : . 
† †
\  
A ]"<  I t D %  t  ?  ? 
 
\  ] j
 CCL I    I " I @ B# C       
    s B  !#"t
 " J  BK I  D H="  @ s  % & "
 
\ ]"h9 C" @>qE@ D M  ( >
 D  " @ %$   "  Cq q I $
 BK    

%h   @$+    O '& CA  |B=  ?   7 rC " jt9

}  @ BtCUK= @   B ( & CA O        B
> C
  & A >?     rC  {P
  L    & A K"  $
  B#=> @? :)+0 * ` | * =(
 CL  # >B % &# -,  # 
C   Q; /.`ˆCE@  TCU=  O
 C & A     ?   $
 rC qE@      qE sCM $  (B I =  > 0 & AH ( @L
;K  ( BVp(
  ?  
0&
\  71 ( > 
C9Q @% & ] r(  A # > 7K   B(
&

?   \  71 "   OE@7 B]t AH ?    rC 

\ ]  &
=  @ s % & "
 "  A t CB= @ 
&
\ A]"h9 C" @ jE@ % &    TA  Q;    A   O $
  B"?     r  
\  ]  =  
"   E  M   
\ ]"h    t  I t
C    `
 CD tB= @ Ktt  BCJ

 % &  A " C  " CL@ 
h  -˜
 3 7KB? C I  +
 3P C"  ` C ,
* ,
[3
 CL  CUK % &   "  C  Q;
¢B_*N?F<G™Z4O9mIn9QdvyPRQGIaF<]IF<G!ONyG-F9N?W*QN?X?ONyF9ePfMamInQYUQNUO9UQN-h
 j @   C     @ C + C0E@ D M       r
     TD      "   ? #     B" 

`  † 
Ž †
A A @     @?  #<T"  E@C%$   K C"  $

 K B 7K   "
7KL      O  E@OSA  D 
@7B W 3ZT"(I  =  ( q
 C   UK   =  : 
 9 C" @ I˜ 
:  "
 !
\ ]"E@ C C D  B  B    " @7B WY3 ZT $
\ A]"OI  `=  O O
  ` UK /)aW.#H   @7B W/D  D  

$ jBt
  r | j 7     
  E@ % & " O "C $
s   ˜ %
)+o`. 3 † '&( : o
)+Ž . :
8 "!
hK C" tO
C A A @    t  B`    `  I   `
CCL I 
 0 ‰B u#"I    
H  )  ‰*"  +  :‰  \ S ?  7&˜#KQ;
C C#
Em % &     C T T E@ %$ C@ s   
 ‰   3 )   ]   
H    ‰q˜
, x C   C jC 7K  
7"
O |? r    U- "  A  @ $
 m /5. rU   BK  #  #C
 CEmL   "r 0 : #{j  
  B$+ F 
H BK  *
\ ]"<  I E@ % & 
7K % & 7C  
K t     B

 O  C? =  <T O     OD I M 
  I  x  BK
\ A]S O =    O @UK ? C j BCC HUK I= t  |
B  
 ! 
) ~ n ~ n ~10 .  C  D C J    >KQ;
 & >E@  % &
:
#
 7K % &  ( J  % &*I  M# 
 C   >q9Q;
 &* 
   ? =  

%
x  ˜
 C   CKQ;
&@ I  -? r    \ >Vp  -H ? rO r

 BK  O  O 
7K %      
  BTCL@?    
   `  B  
 I   " |O
   @?   -Em
 C   @B C "
 C D 
? rtC9Q
 ]
\ ]"h  C I D  BK I  B
C  CH 
B I 7>   C? 
  B
 " ? r 7K J D    B C 
H 
\ A]    
7 B   C?  IO ? rBO
  C 
9Q;
&#C( 
  B K I 
 C    B= C"   " K I   C

 |A   = C"   79 J D C    C" 
" |"z  
F C @1  

 hB*[
  C[  E@UK CCL@UK    (
   q[ 
 KC[ *A $
C C% &  =       J  K - 
  B>  
 3 $    C 
 &
 n  CZ € >  
   ( sI C  #uO
#A % & #
!&  ˜.¥
E B   rA C   B M 
 L I B O I &O  C?  ]T $
!&  $ -
  &E B `z]t I  & -     ]t
  #E B B=9K  


  B H  K @ s % &> C H % &#    B
>CsC 
  C? D  L@  
x C   C * q J  M  (    (
C  Cq #E B L B  \ 
!&
 E UA I      0>  sC  B   C? ] H     
!&
   *   CrC q #E UK"A C CI  "
(D >  Cq    sC $
 B"   C? D  L@   B "n Z ""
 B
¥ x   C   C>D ( q
     qr C q >
H  > #
9Q @  B
 n > : H#  K 
 CUKC
UQ; @   BŽ Z" x    " C

 C " " | 
 ?    m
\ ] x     "
BM   \  UKC]t C  "
t C"
 C

\ A]S
 >D " #
 > 7V7#>  A  B" UV7  
 C$
BKCqVp* (Ž Z  Z "  t UQ;  L D   * #
 q #  C?  
 7 "  rC  \   B ]
%
\  ]"<t- |  \ z C]tD HC C 
7BD BK     C? 

H   "
t % & "C K% &" "   \ A] "! ) )
! x 
    C | @  B% &  † ( Z  
\ # ( Z
]
t 
¥I¥.`  $ & 7& AB &  &%TL C  " K @ s   P
BKQ;
 C ) $
 BHD  @   % &"  9?=9K  " E@ D M  ('3jn Z(q Z
*s    UK C T UV7qE@% & KA  (I   n Z(+ ` C#(  
 $
! /.

 B  K B  H   >E@B C B= KK KK  , &  ?  
D  BK C    " % & "Vp"  B 
\
 ]"} C Cr  I t
C A   K J D 
\ A]S t&  I   D O C ?  7&
D 9Q  E@B C B
 &   


£ 
 
     !#"$!%
#&   #')(+*, -
!./ 
$10
! 2435 - 6&78 0 // -8& 9 8&4
""$
 
 :*!  088
! ;< =*
>    8&4

&""  !$
?
6@  8"
?A CD B * !8& -ˆ
@ QPRB=
F E GH=IKJML+N
O G: PTS = P I/UV P

W &X/ 4
-"

"$ * 
" Y!8&%
%  ;<2
Z   "$ 
-$
8 0!#[ $\ 
 :*]
8$ #^*,_1
 \
.
$  ,7
 //$ \

!$
0!61_ $

?/
 5 -$0!6`82]a,"$"
4 

"Xb6 -
!./6 $10
!

c
dM e 6
A   0 
 ;<=*
f!-   >   ]6&"X 0 
MY
   !
6 0! 4gC 8"
!?/ 8 ?/ hHi b$ \j!DK
B k * - 6 !"  
l 0 8% ;<2Kmn
A
--1
8Y!
A4 - &8 0! A/ -  $10!
2

a,"$& $ > 


60 8
o/!- 
=

£paT;<& 1 > 

!_
! 
?q-r
6Ms r&4M?
-$  7 r!8 0! A/ -

 <!4
 "  "$ 2
t  
/;<& 1 q-r
-Y
 ?
-ˆ uI8   -qr!_
! v 6
! #
!
A 0 A/ 

"&  "$b!Y
! &
! 8
_1
 w
8x!I41 y z ! 88!Y 1
?88s16r2

£-{|  % _1$


4
4
"XO!^6
n""}~
"$ 1 9- |Y!
!  ! 
€"%


-"6i $ 9- ./ >    "
-i  n - 5n"$ 1 z!  n66
"*&
 
‚ !  $
-"8  %7*
60!$   "$ 1 z  * &""$ 8 4
ƒ!- 
! 2
 k4„   6Y
   
 0! 48  %8
-Y 8 -i  > 

% k4„  7&7
-8

!
--0!  !  A/ -  
- ./
?
6!&7†!"n i  > 

k m
//
-$8 
6 Y !8s 16 
Q‡& > 
6  %ˆ6%
j‰  8 $
!88"  z ! 2 k
m9ˆ
K
 "$!
1 -$     % "- "X/
 
6  !$
 "n ! 
! ;<6=8 h
1  
 - 68"4
-""4
8!- 0!
M
-"b8  
 8
-" 9ˆ
 }5^<P
n}5^<P 
@J U }5^<P
n  }5^<P
U 5} ^<P
U
£
|%
A6
""8}
"Xb6&$
-"$ Q
A6


-Y!! 
!"$"}
P
2
3C
!
Y,
-"b"$
4
-"-  -  6& ! &7
 q-r6
n6_1!$q-   "X ?

d* GH} S } I235%X


/ 
""}
"Xb &$
-" ˆ/
/ ?!Y-"
P

6 l   
A; 
 "-s Y
 7

-"b  "Xe  ! 8 -i r8

-Y!2
 k4„   6 -
!
q-r8
!
Y$…
% k4„  7c/
 "r   l 81
/; 
! "$s Y!
! …
k t
 l  > 
%8
-&7
?/1  6
!./
* >   78! 

-qr!88

-Y!!-… „  
8 -
!
q-r8%X
/8
6"" }  …
 k„   ?ˆ
!6&\
Y &!$  %X
/ 
""$} ?/ 0! n -i r 
!
h
Y!!!-…
i!O9UQNVv *}A*} ** 2
 B_*N`F<G‰ZuOmIn9QdvPfQGVaF<]IF<GBONGVF9N?W*QN?X-ONyF9eP MamIn9Q:UQNyUO9UQNVh
P

£ & X/s  


d&7 q-1 l  / 16
! "$    :*"- /  \ ?
Y 

!C'"!C^*
 ?$!h:"$
6 
"X

#G‰'7IKJ%$n'KG:^& 'I('

j‰ k m
A
-$81
   "X ?

 !-6 1 9-!qr! W 2
j:% k „   768
"X
-"X/! !   b$  > 
  "n"$
6&A;)16  _‰ qr!#G‰'7I…
)  "X/ l > 

j‰ k 4a  - ?
6!"4Y! 
"n- !"n&"- qr!"*‰'+
 6 
?"* ,+

- J% "!2

£. W i  8     6


5" ! !4i $6v  -  
 "-    ‰*  !"$"0/

6_1
> z ! 8g4*&

, P

1dJ S43 / g P ' P '
O2/ O
Z !"$"$ - 6
!?
*C
!
-$0 CC
-" _1
? 
~" 1 #i !$h
 v 6  2 „   ?/ 
?/
*
!   r 265K > 


a,"/ 
n
!
--0! 8s 1j‰
"X_‰  6
!? k 
4 " ! !,i $v  
>  e ?/ 8</ 1 9- 8
 q-r
6  
-
9-88s 167'(78,J:;P9 2
£< Z   "$ 
-$
\ ! ! 
# 0b" > 
\ -  "$ "X
\
R$z-"ˆ  "$_1$!q=
-"*n 
ˆ £
"$
0 ?
"
>  z !  j: k -hH
/;<&  "r# "?-- -*4j:  k !hHp\ "%b$ \
#j: 1 k
`£p!h  "!Y 16A$ -2

p
j‰ k
-&
-"$
?
   
|   0!  , 
*!"
!5_1 9$
-K ?/!"!*   > 

" 
&
?/ "4
6

$0 " ?
$ "4"€/z-"n
-"!!"n
M
M 
-"$h

?
*K"$ &   > 
\"$ %"e   \
ˆ% i \0!b!"
,
- 
 ! "nj:!" &X/s  " 1?
- 0!
|&7!4
 
n|&7.
!   .$.A Y6

 0!   -

6
-& "$ Y
6  X8   -
k 2 ) "X/ l > 
2
j:% k W 
$
 !qr\_‰  6
!? K&\
!?& \& ,\0!b"6
, #
-
 

! "6 \& G X<IJ      
G  -I-G S I :234%<
!i \"6
A;<h
&
-"$" =
-"&#\
!
-$0 # ?/
-$ #
\&$
-""$r!*K  6_1 q=
-"R/
h
 
&7
-?/ $6
n6Y 
* 
-"$&
- A/ Y 6
!?
KG 

)IK
,+G )I-2 K
$ l > 
n"

£ `£
"- ",$
-"&7!"",s 
| ?
-$ * >   ?6!" & 
-"$"" -hH
ph *
-"r

A"2

£ Z   "$ 
-$
M"- "
8
 "n&X/s ! !" z!.A !"r! ?
 0!
?
"* 

! *" >    "]&""-
   "7
"X"]
,&X/s ! *
O*
,

$0 "]
"&
 // Y"
" €
" P
2 3

j‰ k a "&
  l > 
4"
-"!" 
-"
4"- "
6&6"T$z-" -"" -ˆ
#HIK%†"$ "!*$#%#HI
_1$   "4
#&#%#
HI &X/s  "4  b""$ -"2
j:% k a " 
-Y!##_‰ q-r#
&XA q-r# -  v  -#\"- "
#&!$\"c$z-" h
"$"* €*('*) ',+ -'*.0/
*
-"$&
- A/ Y 
?
2 Z  
?
"4&$!%%1  
-"4"
7 Y
-$""4
-"!"!2

21%!$*!
5
&"6 n"- "
6 
   434*!&""-  6&$1
?&†&$ 
5
6
"Xb 1 ! 1 ! 
Me 0 76
!M
! q-r8
 ; '\ 
_1
-$
 !  043 2
< 
_1
-$
 !  039 8 !Y!
h:"
6  MY
   !
‡E J ;:$< E #
!M
! q-r 
342
j‰ k „   , \  6&$1
?=5 ] 8 \%$#
- \&8[%"$
-Y!6
!
$
-&  "$
!>39:8 …
j:% k „   ]e 0 =6(7 8 _1$6
?$
%

! ;<'078 
_1
-$
 ! 
3918 *
- 8"$ "X/
398 …
9£ W _  ?

8
!
--0! \ t  ,
-&
! 
%!"- 
?
\ Y
"r\
>  .$
4?  !
!"6
1 4? A@ CB
j‰&†?  " k
![d !

1 jH &†?  "
 z$ " k 2
(D
a  6", i ", !
! " &†?  h:&†? j‰&& k * !_‰ "$r!Q? 

4? E@ =B
&†?  "!* > 
0!
-$ d 
- 
1 *&$ 96ƒ/. ]
6  "
!$1 "2
F 4
!$1 "6%"- 
?
 r#1?
-$0!
![ - [#./-$ \
&X ?\&7
!

" -& t  ]
*
Y
?/  
?
*"$
-
A
- A!"6 ]
2 Z   "- 
  
> 
<ˆ!£ k \Y! 6R?;<\
8
!
-$0 \ $! \&
  t  :*]  11 "$ !
*
5 G-H
Y!
J P JIK (D
w k " 6
!?
n -
! && , - ?$ %&$680!
-qr!

1 L

!
-$0   t  3 ‰w
6p k -
 -
* . |8""

-&7 "$c./ " &†?  "
(D
6 1 %7
-8 - 
!
-$0 8 8 -
! 8&& -* 
"&   ˆ
j‰ k „   ./!"
-  "4"r8& 9 "n  t  7&4"
!0! …
j:% k „   ./!"n -i
0!  ‚ ]
8&4"
!0 …

{
• 
 
  
 ! " #$%!#&' () ()*
+
,) -
+ " .  ( *
)( *( 
0/ 12
3/3
4  ( 05
 ( 657 .//
 
0
  
 8#
 " #9, ( :;<>=?
./3
4" ( $@
.25
 ( 65A$# 5B/
 

•
3
   +#+   #DC ( 
8( 
 <!E>FG4 
..#H 
9D#%3
8

  
JIH$5( #  8K /< L+"  3()M


N!( 4  M O!#%3
33P"
 9D5( #H
3K /G

Q
R8
. 
-S  TVU WX@T
ZY[ z
/
 
%#!   #$
# >
  
#$
/G 3
-
2
BC ( 
0#HIH

Q #(  1%1
B 4( SB1
3/
TVU WXK5
 ./
 
3\3]^R_
Q R8
3 4(
836J:`().
38,)( :;$Da1b3$/
 
33
#33
3C ( 
3
^cedfgDT
z
ZY[
cdfh.i Q #(  8   1  
jA 
 ( 5k #H ( R_
Q R835
  &.
3/.   ( .
.%+#& " #!#H S.#." ( #& 
  
/ ##+&
  
-S$#/%&+/ ##&#( 
+
3/
3
4  ( 83 8g..#<

•l0=m
# 3
 
1/  n(
$.99%3#H 69#1/  IH 
19oZp&O&q.$

/
*3/ *( D/ #9.dfg3r$s 
-S9.dfg./#<t!#(  
%&
 *u v
+./ ( 3  +&
+g!wx9$# Iy
)( 
 .  !/G
 ( *



8( -S8z $%9,)( :;#./ /
43
3( -S8{3.%K v
3#-S8 "

( 
 D 
( 9^ecedfh31<

Q
R8L"  8()$x/
 
Bm/ #$x
|#( 
93(
18 4(
|8|}':;
3G( 8 4(
<
Q R1t!
  +
.
3/)(  +#&
3/#! " ( .3
4" ( .
.
. " j
3
4  ( 
<
Q R1t!
  
./ ##S.":; ( 
./ >/ #.%( ( +# .
.#/ j
,~k ( / ,)( 
3  . }"  
<
Q R8O&# 5

#G":'/ ###x  #/  #x/
 
#G5   #!€ ‚0ƒ {fH„ …€ T ƒ {fy„ …<

†
†=3
./
  k 
.+3
##
. 
 4
.%3+
-

5 H ( 
 3  +
.#/ ,~k ( 
   #H 9 5 ( 
 
|( -S|13
|
 4
s9#()
!/#  

8
3
 4
3%G $ 
]d$F;:;
3
93
3
   
9v9#F(   3  ./%
8/
 

#(  
 3#4.# 
  <
E> 4(
3/  (
G
3/
  k 
8.()4
G
$/
 
3d8]{3
] xM

 

 ƒ {…!c{
{ €
€
Q
R9a|#  B%|#0v0. 1%B?5
  13k)().  U W $
#9
 4
#
   <
#3/ #v
m$()/G  85
  
Q R9a|#  3%.#3v1":; +##.5
  $
#+
 4
##3v
s#3  3#
 ,) .
(  8%.D5
  +8k ( 3 $  U W <
Q R9q+
#1( -#1e(  x$ ":`( #  0?/G  B%() k  (  O& B
#H 65


† 0/  3##/#S8 /Gn 3..
##
m./ ( 3  G#( 


8( -S9v ( * 
G
 8D
3%
-S

z c"!$#&%')( „

t!#(  %8/n 8#.35.D/


85 H ( 
A<

Q
R9a|#  %.
8
4 
4

83#( #  3
33( 4

|M

 ƒ z., - , ,
*c 0/ 1- 2 - / +&#3%' 2 z, 2 … / &#&%' 2
+
 2 544 8%
3G( -S83/n 8,)
*+P
85 H"( 
A<
Q R8O&# 5
3
8%
-S86x
4 
4<
Q R8o   /   3
9%
-S9835().  9*( 
11(  m† B9/   /•
35( #H
|8m# 5
 F;:;||/  18##/#S|1/n <
.

8 ( 4P#@  .##H
8%
-S 

††0= 6J 3+46#& ":; +>/GJ (



 
 ( 5.# +    G$
3
##
0$xS|)( 4
<.O&#H 8/  (
 /3# .
/ ":`().
1/ 1#4()  
/-3./  (
G( ().#( 
AM

9:
8 d /
 
z1]d •
4( S
7 ƒ zG… c 7  /
 
d;
z ;* 4( S9†
€
d /
 
z<* 4( S
 =

   8#H 68#.35(). 


3z9c $8#J"(  / z $P 4(

€?>
  
 ‚ c"7  <

Q
R8O&#|0/-0|/  (
B.#  |
 4(
0   e#
G(
4 
.
<

=
Q R8O  .
9 4(
8( " ( 
8.93/ ( 3  9.
8.L ! 4)(  
3 " 1
#  n#+ 4( #<
Q R8L
8%
8,~-S8
83 " 9
9,  3

8 U
U

 ƒ zG…!c 9:   U  //T  U   /
 

/
 

z1]d
d;z ;*
  U /  U  /
 
z<*|f
; ( )( *9
#3( -#3%95 # 3#
J ( #,)( 
#3/
0,)-S|9
$
#   #&./GJ (
 Q z1cd3+z1c*R$; v
3.JIH  3
 
Nl!e/G0p X;2S>;/C%C5o/<>=@GP;/XS>GXO%<%D/<7GC QkDI@G<L;/CK; S>D/K DPB\ K ;/C%C5;/C
%
-#%.FP
# 
-#J 
## 
  # fZT1f f f >
 <
S2G;2?S2=H;/XO5;/C;2^S>;2O%GMD/<%DD/AB;/I@;/CPAB;EK;jQ*;/\ C5;2<aXBI@GC MNG<
S2GXC%=@K;2<%D/d2“/;/C7F bHC%=@S>DC

Q R 
8%


!#"$%
& &,+ &
 c

U
-
*
' ) &
€(+*
/ & -+ - % . & - 0* /Bf
•
-
; v
8
3":'/ ##S3/
 
88F(    }":;S9
8 4( S <
21
Q R m/G##k 5/
 
e " #35(). 
 $.
e 4( Se/  (

43 *

 8( $#D#,  + }":;S .#()9$#1%.( -#( ##8  

.S$/ %.S

†=9#)v 3#8 65
#( #H .
B c  c ( c g$  ?(>#B


##
.
.,) % n(
.
4
 !+.
./
 H k 
#8
.
-S+ /  (

3
3
7 ( /GB#( 
 9v
  ( B#()./G #$!
#8
' ,~-#8v
3()  (
B
## #/ "( 5#
' 5
  ##SM

7
KW ƒ z…!c
ƒ +…
ƒ …

;: 9< 8 0. = ?>
W ƒ zG… 
ƒ …  9@
‚ W c g
f
. +
/
 > W ƒ z…&#S8#9ACBD E#FHG
IE B
JLKNMOLMPIE#Q M  Q c gf +fRfS S R<
q|()#H
   UT?|1#()
 .v
  3 ( 1#  61 m#H .
1# (  |/

.
#4(   3,)-S8S9 .
)( *

M

V ƒ zxfd…!c ƒ g / z… XW @
Q
R8O&# 5
38# 
2V ƒ z fZd…&   .##/G)( N 3 8( #L+Y  8(  <
Q R8O&# 5
..#H
V ƒ z fZd…    .#&
#&
' ,~-#& .
 ()*

#&
#()
 +v
 L
3 ( 8#()./ #<
Q R8O&# 5
1
9%
-S92  v Z ()4 /  8  3/13  

B,~-S|8
B 3
)( *

6V ƒ z fy„ …/
 
B (  
   „V4 ( |
3#H %
8

3/G
] ] _ ] -b
V ƒ z fy„ …!c \[ R : + +] -  -X^  U_ - / z
 a`
U

c
Q R8L"  8()35
  &#/ 
.
. 

3z9.8,)-S.  3/<
3
Q R1t!#(  %8#( 
 +v
  ( 3#HIH
3/G H  
3/ 8/  (


 c z3f
; v
8
3/ ()3( 
3  -S8S9
8 ‚ W $/
 
3%
 % <
.
c
† Q
R8L

#
#%
-##H
83/k ( 
#/
 
9D46#+( 
~$

 c ^    c
+
R 
^

  
 #S$. #/" ( 5
.  $.5).3
 |0 4(
e3
 $
]
3#H $
./
 H ( &
./ ( 3( 
3 (
.i  3()48( 
$%+
3#
8,)-S
% %
 . `
ƒ f ;…1c ^ ƒ  <L*
0

- `… /
%
  /(
03
 81

0/


3#.
1,~-S$ ƒ f ;…$
9/
 H ( +96  9
1,)-S83/
 H ( -S1
N3
# 
 ( <
43
Q R8=.D3346#.
 3( $  ( 8P ( /(   .5 378$


.
 3/ 
J  
.$,)  
8.
3 :;/
#S3)( 5 +/
 
3 5 3+78<
E>":;/
#S8)( 5   .D( #
3     3( <
(A<+; v
8
35
 (
-S3. 4(
8()   
3  /(
<!r#  ( F%<

( (A< +/ #   #%.


 ( 
.  &e4 6F   /(
.+,~-S
.5 .+(   ( F% +#!#H
#( ( (
+F
346#< Ou:;j
/)( %3
95
 (
-S93  /G(
9s95 3$x' ( )( *
9
1()   j
/ "
-S93(  #C/( 
3/
 
8
3  /(
<
()( (A<   
x.
95
 (
-S9
9/ ##S$
9":;/
#S9)( 5 &b  ( F%3#3

/ # 
-SB4 6F
1|k  m
   (  .#  618
 |m# 

5
 (
-S<


† #3#. 4( #H
#3#(  
 B( 5 #|3B3
|
5( 
9   .
4j
" ( 
8D :;/
#S9( #  C/( 
8  3 
G(
-S8
9.
 ./ 
  
8

+f <


Q
R8=#8
1%
-S1 ( #jALvm/
 
1
|  /(
1/
 
|/ 5
 .%8

/ ##S8
9 
(
-S9.

8/ c  
$5 .
9 4(
857P.
5 .
8
5( 
< `
Q R.
# 68
V ./G 
  
8#H
8 
(
-S$#35). 
 3
 •  
Q R &
 H  ( Q  R+' () ()*9
9†
9 (G
8i  3()48( 
8/
 
8/ 5
 +
9j
##( 
.
8 :`( #H n(
833
8/ ##S8
9 
(
-S8 " 3
4" ( 

Q %|SB 
0C5(
$!
  #9BFG
B#  +o  
R$.
03j
L
#  
-S|' () ()*

1/#H  (  3  8/ ! "*3
x<.EP/ 5
1
#(
jA#

1#4()  8#(  
-SM.m ( /(   88/
 #./ ,)( 
3  8 }j
  
#    n#+3/
 H ().  #(    #+#/
 
#+/ +/
 # 
3j
G / , ( 
3  1 }"  
#8
#3
#8,)
# Q 5HIH
BF4 
B†R<0Es 
j

G(
-S|  ( 
|#../
 H ()3  # 89†1#H SB( ( (
).  8
B3


 #.
 ./G 
  

3  +  c $G3 +9%.
 ./G 
  
9
9 
( j
-

-S1  ( 
|9./
 H ().   =$  <3N
*jA#8 ,) 1
1/
  2
` 
#(). 

3  # 
jA#
3/
 EP( -S  /
  $
 
%3
9
#
#+#3  1<L##
13
( 
$; 
#,) jA#. 4(
9
./
 H ()3  9† Q 3
( #+,) ( R&/
 
833/
 H ().  5  = Q 3
( #%  R<
i 
#,) ( jA#9
  3| #( #H .
03
( #9, ( B/
 
0 #( #H .
03
( #
%  <Ou:;/ ( %.
 
.  ./ %.
9†
1 (G( :;
8.# +5( 

9#
":`( #  ( +.
8/ ##S8
3 4(
3 " .
4 "( 
<

N!( 4 3†M

 " 
† 0L.
# /
 H k 
# +3#.
.3
##
 /#  - c g +b
3()v

" "
#  ( #3/#  #. 5 ( 
#.9df 8df #/G" ( 5
.J <|q+
"  8()
.( # 
  &
# )( >,  3
.
( 
+/
 H k 
+.
##

 /#    5 ( 


  78< L   8( M

Q
R885
    _
Q R885
   78_
Q R8
8 4(
3( " ( 
3
8/
 H k 
8,) 3

9
8 ( #S<


† 
Q
R17 ./!5( 
& /!# G#H ().

/
 H ( G
+3
##
( #/k 5
/
 
8.#H S9 
 +.

:;
8. 4(
9(  
(

91)3()#( j

$ * c
gd - <ED3
##
1( #/k 5 3
|.
##
9|   1
  Q 4( S|  3/ 
J" 
|#F(   3  8
 
|/
 
Bv
5 8,~#S

 
 R$%3%( 5
 .
3s
1.
##
  
 Q  _z _ c + gd
`

4R
2 0
 A< /8%##3
>  8#
 +.,) .
8/ / C  #%.#
,~P
8 ( 4D
   .%#  / ##393
( #  5
  
 2
/
 
38

-8 4  ( $#H ()3V 3/3.5( 
8 A<

Q R8i 
)8( #( 
  
().
$#  ( 3
@ 3/ 
J  
 % ( )k  ( 

3i   
<


– 
  

     !"   $#%&'&
&()*+,
*-./()012-.3 46578  39$:
;4 <  = > &
"& ?#6 3;&
 !@#6
&4AB#6CD
"
E#64  $#%D= "4
4<& FE#% 4
  =G> &
"H2I 6#%
J  / " 7#6K
LNM/ < O E#6P <
C;
FQ<=G &
E#6" 
#6 D;& ?#%N R6#% E#6<4= > 
&DQ S TD  44 

#% QP4<
"&E#69
&U&T7#6T4J O  T 
& DE#6CD

VU
O 7–>€8U  " W#% Q "/
X&E#69
&
 <  / !%#6
Q /  <&AE#6 P< Y E#6&
3Z[\]PC
& U^
SC_  
^&/
F9 &/X`#6  6#6A
E#6 DaN4
#% SC4 !@#6D= 
b#6cd 4O
4&
!@#%
&
D[<\]U e#6/Q !d% 
& TD=G &
E#6D"  &"f@#6
 ;OR R&P4 !%#6
&4F7#@
&2U4  & 4" !%#%

SC8F
g2e#6E57%#%9
;&E#624Hih  ';  8;f@#%

<= 
"@#6
j#%  X  &Xf@?Cb#6'k  l#6
;T4[m;] K/nXJ  P8E#6C 8   !%#6  &'Oo4 
N%#6"
Nj#6 l#6cp "O
"&
!%#%
&"
O
E#6
N
 N   U  N&'Oq/&E#6  U&E#6/& SC/@?P
E#6
$#%/N  &

HGKM /&&
#6 HGK"6#% $#%  = rN"J SCs
H2Q K"N 7#6A"&%#6s
HiI 7#@
J  / X 7#6K

tu//
`#6
!X6#6/J O LvU
NX E#6/Q 
 7#6 P ;7573  "<& ;O!@#%
&<65w<P
= E#6"r< 
D
E#6
B#69
TM 4D
4x#%44  4
46#6A
 4!4= O!%#@
&4& F4<y{z^|{!;6#6DDJ O Dt


&
"&}E#6
3CO@#%
r

L

VU
O L 8U  ""Q Q
j#6



VU
O t v F X

GH K"  
 /J
&E#6/N& FX

&N  X 
H2Q K"Q &"& F"  
"QF4HG% 
 KN&""Q Q
= 
 "W#6" p
"CO!%#@
&C"& ?#6

 7#6 U; j E8<Foo9Q


#6&
&  
#6<< ;&E#6<" PQ4<SC<E#6SC<OF
#%&
3E#6
/Q &E#%/6#6A^  X
7#6_&
E#6X !/
O 
 <

HGK<vU &  /N
N N9QP
#6N&
&  
H2Q K"V S; oOAJ&</9Q
#6nX
 " < > F8 
SC<g
#%  p"
7#6E#6
 @vU

  $
HG&KM  !/" > ""F
E#6"Q
#6 s
H2 KM QFSCX6#6"
&

&
/  N q8&6#%
#% 
E#6U

6#6QP
A
757" pQ &Os

u/ 7#% ?#%X7#%AXF


E57/ F  !  
&XN 

"!83O/ U&
X
U= O @#6  C S8/ #6 

t
#6OE#6FNq
SUP6#6&qF &
  X DN
 E#6/ " SE#6<C<
%#6 E#6 6#6";F

&
 E#6
7#6F"pP  vU
   R

HGKM  "F &


/O  NFP9N SE#6s
H2Q KM /
F&c  " " O E#6
#% s
HG&K 7#6 ?#6 "
E#6PNE#6SC"F S"  57E#6
&/Q 6#%> F8";
O 8
  #6OE#6 ;F"M/ 3F"/
F &
/ SE#6s
H2 K"n/ 9"D SE#6D4&Q 7#%> F2UTF;J&D6#6&
A
sRg
P
 

/ <E#6; 
!&
;;
&; #%;;
;"
%#6
 
PP
&  X"  "P
&  

u/<F&
 #6Y<=  k&

&<
<
E#6 `

@#% nqO
C&
E#6/N
   `!/O
C
& /w#%%#6/FN  =G> &
NN
 Ni /N

npF7#%A< O
C<
F&POF
#6&

PX= 
#6
OF
/ /N
  = r&}  /FO
/&}F &

O    T #6""
?"/
%#6
"&



HGKh" YQ57%#6< R6#6A<; #6%#6<;F3  8!"<


 s

H2Q KhX 
N"Q:5w@#6&
  
C  #6E57%#%9
F
7#6N  U 
QFN }= &
P
&
  }QFX }= &

J:?F! s N "k= rN&A &  
HG&KM $#%l#6P  F"E#%
O
N"i s
/ 3€
 qZ %& $ (
V S&E#6> =G O# z ')& $ *z +$ ,
V SvU

- Z . ('& $ *
 q- z $ / ,

0 !UF/  /23 1 4  y} 1 C


NPN  /& ?#6}E#6/
2&} ?#6g
F /C"OE#6C X  p }&$57 E#6"/ ?#6= S
 

  #6_
5/
HGK /c/"J 
SC" p X'
#6
H2Q Kh757q
8 U'"
#% 6 1  q 1  E#%
= r / SCX
& ?#6SC!76 1 8  1 9);:<^
1 =  < 1 ! #6Q !} q '
#6
 >?3 1 @3 1 =
h Z AB3D1 C  # AB3D1 C !F /!
 23 1 Y 
7#6#E/ /cJ 
SC<F !
" k X 7\PGCWO3<L;
 ! 1 8 > 1 9);: <"1
 7 > 6

HG&K#;6#6/   61  1 E#%
= rp SC"
&7#6r
 k
 
A<1 C  
[
Z A 6D1 C C A ^
A 6 1 ; 1 Z A  1 C C


!U
 
J&   X=G SC& SCE#6 6 1  1 !/   
 }
 657  

6 1 8 >  1
9 C  A > 6 1 > 
A 7>  1 > #1 > 6 1 C  m 
H2 Kh;6 1  1 ;2
 1   1  ' . ,  1 !N/PU 
SC  1 / 
E#6 d;<7#%> &  PQ#6'< SC<"
&6#6r
1 Q   #7 1 8  1  9);:D
E#%
= 
#6" ;6 1  <  1 

XvU
N /7#%> &  /^ /
C  J&
E#6E#6X &
r/
k
E#6F "J 
#6"   
=G SC"/  E-' 8  , / $5w
#6" 
 E#6&
 + 3    

HGK"u/"  SC"h&'
O6#6  QQ



= ED' 8  ,
"!


€   
!&F;HG O7#6C U 
E#6OSC" N7#6K
H2Q Kh  O  X E#6&
   
6#6/
O
A
  "757

# ' ), 
 # $
":% 8
&# &%TC"
N'% !/&7#6E#6 7#%/  
!
 .%
! qZ  
"
 

HG&K X FNN  SCNX


#6Q36#6U  UF
X!
NN6#%> &  
(    ' .% ,
!'

–*) NaN" &


""
&> 
"/&FSC"O
 )<+X  <= E#6 
57AD&'&
TD! &;  ;D&  &> J&DDCD&7#%E#6;!

N  & N &> J&"F /&6#6E#6

HGK 
& ?#6 
&SC
&> 
<O3&FSCO
<
&E#6%#6"&,8
_
&
H2Q KI 7#%
J  XJ
&E#%X   X&  &> J&C&6#6E#6X!

N"  /"& N &> J&"F /&7#6E#6

–>–.- #%r;)"&F;=G SC


#6
_
&<E#6
 3; 

7#%X
 &0/"21.354.6} 6 !UX7NU7#6P&> F


6#6


HGKM 8!;'
 9E#6"7#6E#%> 7#%
&;&
;;7#6;@C;<;g
#%
s
H2Q K"    
 J
&E#6<4 SC4E#6/Q  ""
&&
&4
&
&3 
J   #%

r/  
7#6XP  H2 ?9

 X 6#%> &  &^
6#6NCO&   
@  K
HG&K"  E#%
r/ }OAJ&X
7#6
Q 
SCXQQ

}= SC
 O
 N36#6 3N
7#6NP   NQ  U&
 
&
I 7#%
J  

–Ln}7#6E#%> 7#%
&& A
& <$  g - #%r<P / 
&O


QD<&
SC<"  "<&
E#6<" #6!
&<Q S<
O 
E#6 SC€ #   3
 ' 1    ,     R!4
"6#%> &  "OA !/l#% E#%  #^!/F /  !/"7
7#%> &  

HGK"< SCP/ r


<&YO E#6/
g2  E#@
#6E#%
F
#%

rg2  
6#%
SC/NO
N/&
SC/ 7#6U  g
#@
r/6#%> &  
F/Y &
? Xu`#%

r7#6XO E#6
 `56 7#%
J&Agi 
H2Q KM 
= E#68F&cN&'/  &E#%
Q q/  NOAUN !%#6
%#%P%:
5w  #6E#6E#6"/6#6E#%> 6#%
&  _E#%
&<H2V"
g 
&K
   X"OA/ !&   
#6/  #6$#6E#6&^6#6E#%> 6#%
&
& A
&< H <$
   g - #%rKs<  
 

H X 8
X%#6

 
  Xz<[m  A$#6   
X"OA
 zD[m AE#6  K

–tu/} > F  " pc ?#%"


" ^ !@#6" }9E#6g
#%
 56 $#%/  "E#6SC&  /Q
 Xh  < p
" "<c ?#6
D[m   <U= S4 46#%
E#%
F;4O
;&
!@#%
&;4 !@#6D&g
<&o4O
4 E#6&
 %#66#6AE#%
& v8E#6<< 4
c ?#6<!rAF2H2aNE#6  "F XF &
 " !@#6;
A@
O

7#%
E#%
FN E#%
F
6#%
&X"O
"&
!%#@
&K
–
Nh  ' }&"[mmm"\OE#6F "S"/ AO
"" 
F &
 X\!'<HG  K8h  /  N O U&Q
U
 AO
!X#  "<
O  F
 ^/Or&7#6E#6

 !N &\ <QFN= 
#6X/
= SC"Op&
– NM/ /X&
E#6/X rN
&
E#6%&$"m/"
C
= $#% !@#6N   =G> &
C!/QF 8h/""  =G> &

!<OD

4&R r<&
E#64<4%  &mD<U  
A$:
"O
X= E#6 !%#6
#%
s
–Nh  '"    "[mm\O7#6757f ?#@  "S vU

 4 Rc ?#6T!;$#6> TOF
#6&
 E#6<PD 4F



D= SDOF
#6&
2UJ&4Q
#6; #644 ;R 49Q
#6
&
&  8nXP
 /l#6
"'-U'N"
7#6/&E#6/"O
"
7#6"=G E#6G8N&E#6/"
Q
#6 "7#%=G E#62
–0u/Q/@#6

&
 E#6X  #6 E#%  [m
m q!X& &
S<
E#67#6 M/ Pb#6PE#% 4HG@:
KNJ 3Q
n #6F!X/  P&
"=G> 
&l#6PE#% "!/E#%
O
s

0
}&}… Gs x}…

? l    !#"#%$ & '( "#) "  *+, %# !-$,".*0/%$123$4+*+5 6 2 7, #"#! -
!#"89 ! /4&$ ":*0$;$,&<=?>$@<=6ACB D 2$,"#-E9#$@FG<=?>#HC=?>$@FG<=?AIBDHC=J4>>
KL
!@MN*0O OP2!#"#-Q!Q$4E9#!R,5 Q-$SI!  !7,$&T*+/%$E9#$##/2$,8 5Q$ ".*+/%$! 3- 9#!3  !74!
! /2!UVW>#4 .*+$ "#7, !# $, $0*+/2 +*+X.*0 7,Y-!- /,Z[F =GD >>#B,AIH2<
\ 3]@\^*+$ "_#!!Q$0`^/2$4%25 Q-Qa $b*+/ 74!#& $  Yb*+/2 74Q74/%/%$,% "#-$ ".*+$L
7,@c[!# 7 9 $3!Qd /%R,!%#dO/2 -! /@9&$  Yb*+/%#" E9#!#"#-74# #7,! -fegN=?>#[$,".*+/%$3!
- 9#!@  !74!,LC$4274/%$,a,!Q!h*+/2!.ij$b*+k/2 !-Q$, Y0*+/%#" l$,T*+!m*+/%!.iT$0*0k /4 !Q74/%/%$,% #"#-$!9&
!/%7,-$7, /%7,9" d $,/%n,"#7  !#oqp9T*0 )E9#$r

s lW  M&!q_#!T*+$Q-$Q7,\/%$Q-$&!2%!tuYQ 2 7, #"#!-!%\/%$m*+/4 O_#7,#"#'


- 9v*0 /2$4w*+$,/2&Q "#!- /9&x%$ 8 &$,".*+74#"#- 9v*+/7,#&y/%$,2 T*+n "#7, !z 74#"dO/2&$Q!
) 89#/%! L{C!/%!# $, !&$,".*+$!|*+/4 O_#  I$b`} +*+$9&~)1&9 *+1 #"8,qE9#$Y $4/%7,/%/2 -
  /9&!74/%/2$,".*+$.€bL9# #"_#!E9#$$ &ƒ‚N= >Q!_#!T*+$YO& 9  2 #"#!-!Q7,#&y9&!
a$  #7, -!-$we g $-$, `^!-!QO a#/2$!/%!Q& a $,/%'%$##%$ &„!.*+/2 *+ L

![]@\}*+$,"_#!!3$0`^/2$4%253-37,!#& &!#8"#Y0*b 7,3E9#$;!.*+/%! a$4%2!3!/%$,8 5-!3$,%  /2!


dO/2&! -!Q $  !_#!T*+$$ N*+/2O _#,L
\ C$0*0$4/2&Q "#$32$,".*0 -_#/%X/2 #…,!#".*0 '_ /2X /4 #C$3a !# /;-!74/%/2$,".*+$O"#- 9P4 -!
"#!Q$42  /2!L
7,]@\^*+$," _#!1!1$0`^/%$,%25 1-!1a$  #7  -!-$-!†_#!T*+$$,&ud‡9"#R451-ˆ*0$,&  C$ &


*0$4/2&@-$weg|$-!Q74#"#T*+!#".*+$IZ
Q= A0tz
 g  €    G
2B #H

 l  !/T*b /@-!@$,E9#! R$4@-$!:`;$   "#Qa X 7 9# Z


! ‰ #@]3\^*+$,"_!!Q$4E9#!R45Q-$#"#-!!/%!QQ74!#& $  Yb*+/2 74 Z

      g g  ‚  =?>
 
 O@ ‰ [9# /[- 9#![#"#-![$, $0*+/%#&!#8 "#Yb*0 7,![  !#"#!,-  T*0O".*+!,/%!8 !#"#-'%$
"#!-  /2$4R45$%$,".*0 -Q-$!" $,&x9&y&$  -  $  Yb*+/2 74Q7,#&$# "#-  74$-$/2$,dO/%!R45&%1='
7,#&y&$44&!dO/%$,E n,"#7, !3$&$44&" (&$,/%3-$#"#-!  E9#$ %49#$,&y/%$42 $,70*0 a 
7,!#& @$  Yb*+/2 74@-!-@ /,Z   € =)Ng €+*-,/.0 132
,/4657 8 $    =)wg  *9,/.:0 132:,/465 ,/;<7 =<>

\  27,9v*+!1q$4+*+! -1-$Q  !/2OP2!R45q-$74!-!19&!q-$,T*+!"#-! I$ & dO9"R45


-@  2?# a $  3a,!# /%$,@-!Q74#"#T*+!#".*+$-$dO! 2$<CL
7,Qc[!# 7 9 $174!#&  @~  $!†&Y,-  !ˆ*+$,& /%!#[-1a $b*+/-${CA".*0O"8-!1#"#-!
/2$429 *+!#".*+$74/%/%$,% #"#-$ ".*+$46BQ29# $4/%  4 R,5 Q-$,T*+!@- 9#!@#"#-!,L
†ly# DCEGF3H?I?JLKGM/N?J:O!PQHDRSO TGU<IVO WLXIGJ
UZQM&~$4+*09#-!#".*+$-$47  -$&$4-  /!174!!'
7  *+Y#"#7  !-$Q9&y74!!7, *+/!:*+/2!,aY4-$9&!$b`^ $4/4 n "#7, !7,! /2 &Y0*+/4 7,!LDZ; $! O 7,!
*0$,"#V$,3a !/2 !-!"#74!!7, *+ /  -$4274!/%/%$ 8 !"#-  '   8 $ & 2$,89 -! $ &x9&y/%$44 2'
*0 /Qz„=yJ[ > \@&$4/48 9O_#!-$ &x9&y/2$47    $,".*+$/2$4$ "#74_ -7,&xk $4 LDZ;&y7,!-!
$b*+!! !#9&$ ":*0Q-$h*+$,& $4/2!:*b9#/%!Q#7,/%/2 -"#74#":i+9".*+k $4^]/%$,2 T*+/, ! k3!
-$,%7,! /48,!-7,! !7  *+/, $,/%!7,9 -!-%!#&$ ".*+$@&$4-  -! LI9#/2!".*+$@!$0`^$,7,9#R,5 -$ *+#-!
!$b`^ $4/4 n "#7, ! $4+*09#-!#":*0$|*0!&\ Y,& *+$,a$7 9 -! -- $&!#".*+$4/74#":i+9".*+1‡k# $,
$/%$,2 +*+/,#-$c[!!7, -! - $`_@Y4/4& 74!h*+.*+!#badc =6tfegVh:i|=j' > >3k B-lma *+$4/4& 74!#&$,".*+$
 %# ! -q-1&$, !#&\  $ ".*+$L^ƒ*+!\ $  !q-$-! -  $0`^ $,/2O&$ ":*0! E9#$$, $Q\^*+$,a$QY

n
&T*+/%!-!Q!\! `^ L
J F   g a
 g a

< P‰  P‰
?‰      P‰
‰4<  n  P‰
     P‰
 <    P‰
n     P‰
n < ‰ P‰  P‰
<  ‰    P‰
< ‰    P‰
  #   P‰
!{C!/T*0 "--  /2 "#7<#   -$Q74#"#%$,/%a,!R,5 -$Q$,"#$,/28 ! I&T*+/2$E9#$Q!a,!/2 !R45
#" ! *+$,& $4/2!:*b9#/%!†-†k# $4†Yq/% /%7  #"#!#@!†E9#!-/%!-†-! *+$ "#%5†-$q74!/28 !†-
7,!! 7  *+/%$,,Z6=CF hl2$,"#-†9&!7,#"#T*+!#".*+$L
\ Cc[#"#+*+/29#!  "#$42! R,3E9#!-/2 7,9 ! -74/%/%$,% #"#-$ ".*+$9& 8 /%X)7474#"a$," $,".*+$
E9#$QO_#$ $4/2&Q *+!  !!/T*b /-$4+*+$ \^*+$,/3a !# /3$b`^ $4/4 &$,".*+!#I-!74!!7, *+Y#"#7, !-
7,!! 7  *+/39v*b O P4! -Q $  Q$4+*09#-!#":*0$"#!$b`^ $4/4 n "#7, !L
7,;c[#&!$b`v $4/2 n,"#7  !%$,/2 !!#d $b*+!-!7,! 2$4T*b9#-!".*0$@/%$42 a$42%$@/%$, $0*# '( !9v*0 '
O P4!#"#-19&~/%$,2 +*+/-†&$,2&ˆ*0  1%kqE9#$74#&~/%$,2 T*+n "#7, !1 z  =$[ >>Q\@oyc[#&
E9#!# -$,T*+$,@/%$,2 T*+/%$,@Y %G# a$  /%$,!O P4!/@!$b`^ $4/4 n "#7, !Q&! @/%!  -!#&$ ".*+$,o

 }
x
"!$#x…

%?l C]W_#!#&Q  *+#" !#"#3-$;9& 2 T*+$ &!3E9QY#".*0 7,@- $[-# ["3# a$, IY;/%$4/2$4%$ ".*+!-
  / Z

&x=(' ) )
+*
,@O"#T*+!#".*+$w*.- #Q2 T*+$ &!Q%$$,"#7,".*+/2!Q"#Q$4+*+!-/2$4/%$,%$ ":*0! -Q //' > J * L
0 T21 ]3\^*+$,"_#!Q@!#9v*+#$4+*+! -@$/%$42 $47b*0 a @!#9v*+ a,!#  /2$4@-_#!#&O *0" !#"#L
0 cD1  +*+/%$E9#$!Q/2 \!\ O  -!-$-$$ "#74#".*+/%!/32 T*+$ &!"#&$,2&Q$,T*+!- " '
7  !#‡ !/2!m*mV  a ! /4 !_#!/2&"  74!#&$,".*+$74#& *: 7,&xdO/%$4E n "#7, !E9#$|*+$ "#-$
!P2$,/%E9#!#"#- )3 >L

4l [c[#"#2 -$4/%$@9&!3!/T*m# 7,9 !3-$@&! 2%!5„& a$,"#-'%$;$ & 9&!3- O&$,"#%5 
 /2 !#8 !#"#-'‡2$!Q #"8 -Q$, `^h`  "#2$,9%$ ".*0 -  2 *0 a  $,& 9 &„ .*+$ "#7, ! -
*b  Q-$, *+!Q-$ /%!7

FG76 H[=98;:#G76 H=< G>8V6>#H


T219#!# [![7,"#-  RG$,[-$@74#".*0O"#9 -!-$@E9#$@!dO9"#R,5-$@#"#-!-$4a$@%!.*0 2dO!#P2$,/C$ &
6=?>;o
c 16Z;"#7,#":*0/%$!3d‡9"#RG$43-$#"#-! L
a 1`Z;"#74#".*+/2$@74#$,)7, $,".*+$,@-$w*+/%!#"#4& %%5Q$/%$$0`^5 L
lW M&x$, Y0*0/%#"1"#7,!#& q74#9 #&\  !#"#-$9&x/%k.*+#"1$,"#7,".*+/2! '2$$ &
9& $4+*+!--$,%7,/2 *+Q $  !dO9 "#R45-$"#- !

G U H[=  J J  
 2€2€bG U  H  € , €bG U H  € g G U H
JJ '
#"#- $ hG U  H;= z bG‡UH mG  <%< H;25 ! 3!#9v*+#d‡9"#RG$43-_#!&Q  *+#" !#"#   -Q4 2'
*0$,&!7,9:iT@!#9v*+,a !# /%$,@%5Q-! -@  /
  =  '< v* g  % J  <
#"#-$ * $ vgm25I/2$42 $47b*0 a !#&$ ".*+$I!q74!/28 !q-$  Yb*+/%#"1$Qq/%!# q- $ ;_/,L  Y &
-  %2 mG  <2< HC%5!#9v*+#d‡9"#RG$4;-E9#!-/%!-- &&$,".*+!#"89 !/, ! . $3-!49#!
7,#& #"#$ ".*+" $ !$#.74#&„!#9v*+ a,!#  /2$4 %&G %' J,)H *(  $+ t *I( #/%$,% $,70*0 a !&$,".*+$#L
T21G U HYQ9&!!9v*0d‡9"#R,5 -$,o#$Y# E9#!#IYQ!#9v*+,a !# /7,/%/%$42 #"#-$,".*+$Q-$
o#$"#5 #E9#!# YQa,!# /@$4% $4/%!-Q-" $ o
c 1G U H[Y9 &!!9v*0d‡9"#R,5 -$3S bo$3Y#E9#! Y3!9v*0,a ! /;74/%/%$,% "#-$ ".*+$,oq#$
"#5 #E9#!# YQa ! /@$42 $4/%!-Q-$S bo
a 1-G U  H;Y9&!!#9v*+#dO9"#R,5-$3.S #boq#$Y#E9#!#Y!#9v*+ a,!# /@74/%/%$,% #"#-$ ".*+$,oq#$
"#5 #E9#!# YQa ! /@$42 $4/%!-Q-$.S #bo
[ 1c[!# 7 9 $!@/%\!\  O - ! -$,@-$  Yb*+/%#"Q2$$,"#7,#".*+/%!/@$,& 9& $4T*+!-Q74#/ & %@=J
$/2.ij$,RG$,@ty=;J <%> <?J#/2$4% $47b*0 a !#&$ ":*0$L
K 13-.*+!#"#-q!1d‡9"#R,5 1-$"-!q-!-!q74#&q$4+*+!-1 "  7  !#[!/%! * -C\^*+$ "_#!
G U  <T‚+H4L

0l @M&y/%.*+/3E9QY#".*0 74  7,9:iT! /4 $ ".*+! R,5$4T*+X$,% $,7, )74!-!%#&$,".*+$ $, !
7,# /2-$,"#!-!1;#Y- $4%7,/2 *+ $  _#!#&O *0" !#"#
&x=j
a & 0 g 7  @FG1CH
#"#-$& 0 g 7 
= 2*( 4  3 1  #FG1CH[=5*( 6)7  '1q$`a VV @LC#!\ $,"#- E9#$@[!#9v*+ a,!#  /2$4
3  g -8
$3!#9v*+#$4+*+!- 3-$ & 0 7 %5 /%$42 $47b*0 a,!#&$ ".*+$ <  G1CH;= *9 . $D =a*(  t 
' A
7,#&„ty=?> <:9J <:9D' <);<;;O $4-$,'%$#Z

T21`7, /2/%$4R,5"#5"#9 !Q-$&! 3\!# `^!Q/%-$ & !/%!Q!$,"#$,/28 !-Q$4+*+!-d‡9"#-!'
& $,".*+!#‡L

c 1 74/%/2$4R45Q"#5Q"#9 !- $&!# @\! `^!/%-$,&„!/%!!Qd‡9"#R,5 -$#"#-!-$4+*+!-
d‡9"#-!#&$ ".*+!#‡L

‚
u !m}3!x
‡l I9# #"_#!3E9#$[! k [9&!374#O %5c[&^*+#"$,".*+/%$@9& dOk.*+"$;9 &?$, Y0*+/2"
O " 7  !#O&$ ".*+$;$ & /%$, #9#%  dOk.*+#"$@3$  Yb*+/%#"%!# !#& -$@&!#"#$  /2!32O&Y0*+/2 74!  #9%$+ij! 
7,#& Y#"8 9  -$Qa ! /%$4Q 89#!# q&! T*+$ &x/%$  !R45fB-  /%$4R,5/2O8 O"#!#C-$
O"#7, -n,"#7  !-d k.*+#" L3#$!Q$ "#$4/28 !O" 7  !#I-dOk.*+#"Y-$n< $,KE9#!#  Y#"89  
E9#$374/%/2$42 "-$@!$,%%$3$,%!# _#!#&$,".*+2O&Y0*+/2 74$3E9#!##a !# /;-! ;$,"#$,/28 !;) "#!# 
-dOk:*0"$$  Yb*+/%#"o
B ˆl† " 9#!#"#-q9&!!#& +*+/%!%k#O -!-q&$b*+!#I-$Q%k-  Y O9&Q "#!-!74#&
O9P3-$7,#&/2O&$,".*+- $"-! > '1J4>w"&#Q .*+$,"#7  !# "#$,74$,%%X/2 !/%!QE9#$"#5
 9#!7,/%/%$ ":*0$3"#7  /%7 9 *0! \!# `^Y3-$` L KL 9#!#"#-74#&/4 &$,".*+-$3#"#-!-!
!  > J J4> "&&$42& .*+$ "#7, !##!8 /%!a !# 
O9P;Y@! *+$,/%!-!/%2 $ ‰L 3KL<Z@"#74#".*+/%$
!d‡9"#R45m*+/%!\!O_#Q-2k -  Q$a !# /@-!Q74#"#T*0!".*+$-${; !" 7  L


‚6fLNi RYPTS(K aNWeS0]#Te]#K?S
]„d /2&9 X/2 /2$, !:*0 a B/% a,!-$Z; $0*+/%#&!#8 "#$b*0 4& $,"#7,#":*0/%!'‡2$Q"#! /2E#9 a
Z@ $b*+/%/2&L  - d
!ˆ= !$9
!.l !    G <2< HC=ja     I€:G <%< H
!  = 8 # 
8  ‡  $,/2&9v*+!RG$4@7<# 7   74!@-$E   
  0 € 7 =:% F %   0  7 =  :%0 g 7 F t 0 g  7
       


<  = < 0 g 7  
‡t! " %
g
: & g < 0 g 7  ;;<;
     ! 0 7   0 7
= # € G<6
Q8 H=< % $ = # € G<6&+
Q8 H=<
 
& %qV=   %  % ?J3 V < % $' %1V3=  %,?J %?J3V
( =*+) ,  < ,  = '
> J <  = ' > 
< #N= ' J >
 € J > * ,
> * , > J * >
- =  €
, .021430
5 v= 5 ?G - 6J,7
/
H 648 9  - : ‚
‚=^= - Gf‚ <  6438 0 9 H . 0;:
ty= - t g
=?t^e
 N= 8 e. †t g >e =
!W=)'% *( <L%q=;J <?' <) < >> >
z = , = < , =)[ > @D ? > 1J4> /, A'B BG‡t  €; E CD=:H
*= D <2D 1J4> , =9 k ; = =) <,J? †J4> , * 5 F ;=
* ( =J;<%>Q[1J4> , =9k; = = D <?[@F†J4> , 2€ G * F5; =
e@=j' <4HH@F†J4> A &… 
*be@=JG' <V 1J4> &, Im* 5 F ;,ty=JG' <V J * F ; K
*b( e@=;J <4H?1J4> &, M€ L Im* 5 F ;,ty=;J <4H?@J * F ; K
* =J <%D > 1J4> , c
&!%%!Q-$/2$4 #9#%Q 4€-Q$, Y0*+/%#"$-Q k2 *+/%#" Z
t g N = H <4JJ †J2> , 8=?> <V[ J O * F;B e 
J * F =J <%D > †J2> , € L k


}x 
  
‰O‰ !"!#$%&(' )*!"+-,+/.)01!"&(+%324)56+7"7#89+:7;2=<+>+:?$&(@ A !"& )
%0$B+:7"C0)!:& D:)*%3240@7#)*!#+?$00@1E)0,+)F*+!#G7#+7#+H324!"& 24)EI J$H,*,)
&(%07;240%324+B*!#!"!"+:.:+E+$H&(1E$@ 7#)EK 0,+.$0!424,E$!#*L:M*)B1E+:!#1+%0,E& .$@ !N
*!#!#B%0$O1E)0%324)P,*!#!#Q240@?$0+RSUTWV0)0%0,+XUY).+%324!#),+*7#7"
,B*!#!#EI
0 5)*Z)*1E[*7/)B& 1E$@ 7")0?$00@YF*+@ )0.& ,*,+,).+%324!#),+7"7#,*!#!#0\
E-]$*@YF*+@ )0.& ,*,+0%Z*$@ *!-,B*!#!#B+^24)!:%0),)B.:+%324!#)B,+7"7#0\
.]$00@Y6F*+@ )*.& ,*,+,E+`_*2=!#+B& ,*,+B,*!#!#*& 70'(7424*,,)61E)0%324)
,+&(1.`24)BPa\
,E/]$00@EY@ )0.:0@(& D:LM*),+$O1E)0%324)]>,*!#!#E01*!#)?$00@F+@ )0.& ,*,+
&(%07;240%324b0%0+:BY%0$@ B\
‰a 0%0+& !#,+Q24!#0%07:' +!"& !$c'()0Z$0+`24+,+B$[*!#E& 24.& !#.$@ *!1!"
)0$d24!#Y-'(0D"e:Gf@ )1E+!#.:)*!#!#+!$E[*!#E& 24+@(A 1g2`& .,+h2=!#0%07"'(+:!#e%0.& ?$0+i24*%EZe%0.& 
*7,E$0*7[*!#E& 24*7.& !#.$@ *!#+7Ij )0%07:& ,+!#+?$0+*7[*!#& 24*7,k+!#!#+,E+86*!;24+
7"M)[*!#& 24*7-.& !#.$@ *!#+:7-,+!#0& )7&(Z*$0*& 7-NBlnmo+lnpo!#+7#1E+.=2=& F0+%324++?$0+$
'()0Z$0+`24+/Y@ 0%0L,),k+!#!#324Y-86*!;24+%0$[*!#E& 24+@ A 1g2=& .:,+i24!"*%7"'(+:!#e%0.& 
240@?$0+)7#+$1E+!"&(C0Y@(& ),E& 7424b*%0.& ,+0& )*!/*1!#)`_q&(*LM)0 +:7424<%0[*!#E& 24,+
k+:!#!"+)B7#+$0' Y@ & )6,E& 7424b*%0.& B,+*& )*!-*'(*7;240+%324)0-%0[*!#E& 24,+86*!;2=+*I
0-]$00@YF+@ )0.& ,*,+,)'()0Z$0+`24+!#+@ 32=& FB86*!;2=+?$00%0,)B+@ e77#++%0.)0%0G
24!"*9\
EXU?$0+,+:F*+)*7' 0D:+:!-1*!#B?$0+)' )0Z*$0+=24+1E)0$07#++H86*!;24+\
.-]$00@EY)r24+1E)B,+s24!"*%07:' +!#e%0.& B\
t M*)B.:)0%C0+:.& ,)*7u vBu.:)0%074240%w2=+Z!#F& 24*.& )*%*@+xzyE*7#7",) t )0@{I
‰|a })0& 7132=&(%0,)*!#+7,++:7"~*7#7"+:7424M)7")!"+7:$01E+:!"'{A .& +,+$
@ *Z).:)0%Z+@ ,)EI>,+@ +:7€*+:7424<+>!#+1E)*$7#)E+%0?$00%w2=)?$0+-))*$d2=!#)E *7#+
*1!#)w_q& B,)B1!"&(+& !")B.:)0HF*+@ )0.& ,*,+6ƒ ‚ +1!"b*+=2=!#)B,+& 1*.=24)„+%0)*!
?$0+6).)*1!"&(+%324),)*7B!#*L:)*7B,)*7132=&(%0*,)!"+:7IJ-)& %07424*%32=++…?$0+6
,E& 7;24b0%0.& B+%324!#++@ +:7-YA(%& E +@ +:77#+Z$0!"*G7#+1+@ 7M*)*7I
0]$00@YBF*+@ )0.& ,E,+*%Z*$@ !,E+B!#)324*L:M*)6,)*7132=&(%0*,)!"+:7+†24)*!"%0),)
.+%324!#)B,+*7#7#\
E-]$00@EY)r24+1E)BBA %E& )?$0++@ +:7-,E+:F*+O1+:!"0%0+:.+:!$%& ,E)7-1*!#B?$0+)
132=&(%0,)*!?$0+B+:7424F61!",E)€c7#0& 6'()*!"0%0,)$cb*Z*$@ ),E+B‡ˆ3‰r.:)0c
,E& !#+:LM*),+6ƒ ‚ %0)B7"& 7;24+B,+@ E)*!#324[*!"& )0\
.-]$00@EYr24+%7#M*)B%0)*7-!"L)7,)7132=& %0*,)*!#+:7:\
‰:ŠU }$0*71*!;2=A .$@ 7,+B*7"7#*7& Z*$00& 7‹…+7;24M*)@ &(Z*,*71E)*!$.)!",
&(%0+=_024+%07"A F+@E,+.:)01!"&(+%324)BŒ+*7#7#B,+7#1!#+D:A F*+@0?$0+17"7#B1E)*!$H1E+G
?$0+%0)'{$0!#)7#)*!#+$E+7#C0)*!"& D:)*%32=*@I6,*71*!;2=A .$@ *77#+)F+6%0
7:$01E+:!"'{A .& +,+7#C0)!:& D:)*%3240@+)0$d24!"*)B@ )0%Z),B,E& !"+:LM)BF*+:!;2=& .:0@I
0/k)00%0,)B1*!#B.:)0)*!#,+%0*,7Z+%0+!#0@ &(D"*,*7*7-.)0)!",+%0*,*7-1)*@ *!#+:7EŽ"
,1*!;2=A .$@ B?$0+7#+B)F*+%0)1E@ 0%0),6+7#E %0$97"& 7;24+,+.:)0)*!#,+%0,*7
.$w‘)*!"&(Z+’.:)0& %0.& ,+.:)0’)'($0!")%0+:7" )*7424!#+B?$0+@ 0Z!#0%Z+:0%0,)
7:& 7;2=+Y&(Z*$00@EEu


~Ta‹  p   ‹6 p   p  ‹ 0
E
7".:!#+F*7-+?0$0*L +7-,+50Z!#0%Z+*I
.B86)*7424!#+?$0+))0+%324)~0%Z*$@ *!+ !#+@ *L:M*)~*)'{$0!#),+:7"~Y$
.)*%074240%324+,))F& +%324)EI
,E86)*7;2=!#+?$0+Y1E)*7#7:A F*+@C0F*+!6)F& +%324).& !#.$@ *!6%0)1E@ *%0),W+7#EI
j 0@ .$@ +))*+%324)0%Z$E@ *! +-+%0+:!:Z& ,)7"& 7;24+E*7#+-1!42=A .$@ %0)1E@ 0%0)
,+7#B7#+)F+%0$B[*!#E& 24B.& !#.$@ *!-,+!"0&  ) I
…}B
}O} x  } …   … …
‰a   .:)*!#1E),+.:*1*.& ,*,+ 2=Y:!"B& . U.:)0%07;240%324+Y.:)0@ )0.,)+ .:)0%324324)
.)*c$9!#+:7"+:!#F324[*!"& ) 24Y!"& .:).$w‘ 24+1E+!#32=$0!#   7#+B0%324Y9&(%*F*!"& *%32=+*I
#%& .& *@(+%324+*0)B.:)*!#1E)+%0.:)0%324!#*G7#+NQ24+1+:!#32=$0!# ‰ m 0& )!-,)?$0+ ‰ I}+G
24+!"B& %0+F*!"& LM*) 24)3240@ ,++%324!")1E&   U+*7F!:& *L *+7,+%324!")1E& *7,)
.)!"1E)+-,)!#+:7"+:!#F324[*!"& )EI #%0,& ?$0+-7#+7 F*!"& * L +7 7#M*)1E)*7:& 2=& F*7 )0$B%0+Z32=& F*7I
‰ a -86)*7;2=!#*!-?$0+  
"!$#&% ( T ' )+* -,   .
0) %0,+/+)nT *
10324 2
 . )Q+/,T *  ' . * '  . ) 24
‰ 5a }+=2=+:!"B& %0*!B~+%0+!"Z*& Y,E& ~,+36 )*7#.&(@ *,)*!#+7& %0,+1E+%0,+%324+7B+6,+
+:7:' !"+:?$0e%.& +>.:)0%324324).:)0 $ !#+:7"+:!#F324[*!"& )n24Y!"& .:)Ns2=+1E+:!"w2=$0!"
I€>+%0+:!"Z*& ,+.*,)*7#.& @ *,)*!-Y,*,1)!
7>T849 *;:   . : TaˆEŽ  Ž<Ž>= ==
‰ ?a -}+,E$D"B@ +&,+ t 24+' 0%0Gf )0@ 2=D:0%
% @6B T ACEDnB1*!;2=& !,B,E& 7;24!"& E$& L:M*)
,+/)*7#+:GF
-& %07;2=+&(%I€>,+%07"& ,*,+,++:7;24*,)*7-1*!#'([w24)0%07-Y,*,1E)*!
p
 *FG . HT ' p G8
I .ML *KJ
)0%0,+ G YB+%0+!"Z*& B,+$9' [324)0%+ ' )BF*)0@ $+*I}+=24+!"B& %0+.)*%074240%324+A I
‚  E N!m}-!x
‰ Oa })*& 7B& !"M*)*7Ze+)7€%3M2 P0%& )+6/!"$%0)E,E& D:+’1E)*7#7:$& !B$@(& Z*L:M*)
.[7:& .:6?$0+B'(0D.)*c?$0+B$E97"&(%3246*7,)*!#+:7,)6)*$d24!")E& %07424*%3240%0+*+%32=+*I

7"7#+7,)0& 7B& !"M)*7B+:7424M)+…!#+:1)*$07")+ $…,*,)!#+' +!#+%.& *@ 7#+1!",)*7
1E)*!6$~,E& 7;24b0%0.& ~,+4 QRQTS6-+€%3M2 P0%& ).:)0+L:~.:C0)*!#*!+=_g3240+%324+
%0)& %07424*%324+B+9?$0+B/!"$%0)*.+:!;2=2 ),+,).)*’$c!424+@ )I9.& +%32=& 7424
F& 3‘0%0,)B.)*HBF*+@ )0.& ,,E+,+ *    ‡ . J *)@ )0%Z)B,@ &(%C0B?$0+$%0+)*7,)0& 7
& !"M)*7-& %0,)~,+6€%3M2 P0%& )~1!"~ !:$%0))*7#+!#F~)*7B,)*& 7B+:F*+%324)*7{.C0)!"),+
€%3M2 P0%& )-*!;24+@ *,~,+/!"$%0)0:IU]$00@-~7#+:? e%0.& ~,)*7B+F+%324)*7B7#+Z*$%0,)

|
)~.& +%32=& 7424a'()*!"%0+L:~,E& '(+:!"+%0L~,+ 24+1E)+%324!"++@ +:7-7#++@ +76%0M)W' )*!#+
7:& $E@ 24b0%0+:)*7:# \ W1E)*7#7"A F*+@E.)0%0.@($& !-7"+"@ &(ZLM*).[7:& .:!"+:0@ +%32=++=_q& 7;24+
+%w2=!#+)*7& !"M*)*7"\
4QU  P)*7"& 24<! P0%& )Y$ <324)0)'()*!"*,)1E)*!/$E +@ Y=24!")*%B+$ 1[7:& 24!")*%{0%32=& G
+@ Y`24!#)0%:T I
/@ +6Y7"++@(C00%324+*)<324)0),+C& ,!#)0Ze%& ) .)*…)1[7:& 24!")*%W%0)
@($Z*!,)1!#[324)0%Io2=&(@ &(D"0%0,))6)0,+@ ),+B )0C0!!#+:7"1E)0%0,Eu-0
/%0.)0%w2=!#+)
!#0& )6,+B/)*C!1*!#6+:7"7#+B7"& 7;24+EIE]$0*@)6.)*1!"&(+%324)6,+)0%0,6,+$
'([324)*%*7#7")0.& ,E)Mr24!#0%07"& L:M*)% 1E!"B% /‰\

You might also like